FINAL Chapters

Ace your homework & exams now with Quizwiz!

Which feature would alert the nurse that the client is in the transition phase of labor? a. enthusiasm in the client b. decrease in the bloody show c. beginning urge to bear down d. reduction of rectal pressure

c. beginning urge to bear down Starting of the urge to bear down is a feature associated with the transition phase of labor. The transition phase is the last phase of the first stage of labor. In this phase the process of cervical dilatation is completed. During this phase the client experiences an increase in rectal pressure, an increase in the bloody show, and an urge to bear down. The contractions are stronger and hence the client feels irritable, restless, and nauseous. The client feels enthusiastic during the latent phase and not the transition phase.

When providing nutritional counseling to a pregnant woman with diabetes, the nurse would urge the client to obtain most of her calories from which source? a. saturated fats b. unsaturated fats c. complex carbohydrates d. protein

c. complex carbohydrates The pregnant woman with diabetes is encouraged to eat three meals a day plus three snacks, with 40% of calories derived from good-quality complex carbohydrates, 35% of calories from protein sources, and 35% of calories from unsaturated fats. The intake of saturated fats should be limited during pregnancy, just as they should be for any person to reduce the risk of heart disease.

A nurse is admittig a pregnant woman with sickle cell anemia to the emergency department. Which findings would lead the nurse to suspect the client is in crisis? Select all that apply. a. increased skin turgor b. joint pain c. fever d. pallor e. fatigue

b. joint pain c. fever e. fatigue Signs and symptoms of a sickle cell crisis commonly include severe abdominal pain, muscle spasm, leg pains, joint pain, fever, stiff neck, nausea and vomiting, and seizures. Skin turgor would most likely be poor because the client would probably be dehydrated. The client may also be fatigued during the crisis. Pallor would be the result of the anemia but not necessarily indicative of a crisis.

A client in labor has administered an epidural anesthesia. Which assessment findings should the nurse prioritize? a. maternal hypotension and fetal tachycardia b. maternal hypertension and fetal bradycardia c. maternal hypertension and fetal tachycardia d. maternal hypotension and fetal bradycardia

b. maternal hypertension and fetal bradycardia Epidural anesthesia conveys the risk of hypotension, especially if the client has not received an adequate amount of fluid before the procedure is performed. A sudden drop in maternal blood pressure can cause uterine hypoperfusion, which may result in fetal bradycardia. The other choices are not an adverse effect of epidural anesthesia.

A nursing student correctly identifies the most desirable position to promote an easy birth as which position? a. breech b. occiput anterior c. shoulder dystocia d. face and brow

b. occiput anterior Any presentation other than occiput anterior or a slight variation of the fetal position or size increases the probability of dystocia.

The nurse is caring for a pregnant woman is determined to be at high risk for gestational diabetes. The nurse prepares to rescreen this client at which time frame? a. 24 to 28 weeks b. 28 to 32 weeks c. 16 to 20 weeks d. 20 to 24 weeks

a. 24 to 28 weeks A woman identified as high risk for gestational diabetes would undergo rescreening between 24 and 28 weeks, however, some health care providers can choose to conduct this screening earlier.

A woman is to undergo labor induction. The nurse determines that the woman requires cervical ripening if her Bishop score is: a. 5. b. 7. c. 9. d. 6.

a. 5. A Bishop score less than 6 usually indicates that a cervical ripening method should be used before labor induction.

A client gave birth to a child 3 hours ago and noticed a triangular-shaped gap in the bones at the back of the head of her newborn. The attending nurse informs the client that it is the posterior fontanelle. The client is anxious to know when the posterior fontanelle will close. Which time span is the normal duration for the closure of the posterior fontanelle? a. 8 to 12 weeks b. 4 to 6 weeks c. 14 to 8 weeks d. 12 to 14 weeks

a. 8 to 12 weeks The posterior fontanelle is a triangular-shaped area at the back of the skull. The nurse should inform the client that the posterior fontanelle normally closes by 8 to 12 weeks after birth, and if there is delay the primary health care provider should be notified.

Which neonate is at highest risk for developing neonatal herpes following birth? a. A newborn who was a vaginal delivery to a mother who had her initial outbreak during the third trimester of pregnancy and has active lesions b. A newborn who was a vaginal delivery to a mother with no active lesions but a history of herpes c. A newborn who was delivered by cesarean section following prolonged rupture of membranes to a herpes positive mother with no active lesions d. A newborn who was delivered by cesarean section to a mother with genital herpes

a. A newborn who was a vaginal delivery to a mother who had her initial outbreak during the third trimester of pregnancy and has active lesions The newborn most likely to develop a herpetic infection is the one delivered vaginally to a mother who is experiencing her first outbreak, may or may not know she has herpes, and has active lesions. Delivery by cesarean section reduces the chance of the newborn developing herpes, even if there is premature rupture of membranes prior to delivery. Vaginal deliveries are recommended for mothers with a history of herpes but no current active lesions.

A pregnant client is screened for tuberculosis during her first prenatal visit. An intradermal injection of purified protein derivative (PPD) of the tuberculin bacilli is given. Which sign would indicate a positive test result? a. An indurated wheal over 10 mm in diameter appears in 48 to 72 hours. b. An indurated wheal under 10 mm in diameter appears in 6 to 12 hours. c. A flat circumscribed area over 10 mm in diameter appears in 48 to 72 hours. d. A flat, circumscribed area under 10 mm in diameter appears in 6 to 12 hours.

a. An indurated wheal over 10 mm in diameter appears in 48 to 72 hours. A positive PPD result would be an indurated wheal over 10 mm in diameter that appears in 48 to 72 hours. The area must be a raised wheal, not a flat, circumscribed area.

With which findings would the nurse anticipate a diagnosis of false labor? a. Cervical dilation of 1 cm b. Softening of the cervix c. A feel of pressure in the pelvic region d. Regular contractions 8 minutes apart

a. Cervical dilation of 1 cm To be in true labor, there needs to be cervical dilation and effacement. Cervical dilation of 1 cm does not show progression in dilation as the contractions are not effective in producing further dilation. The other options could possibly be signs of true labor with cervical dilation.

A nurse is caring for a young woman who is in her 10th week of gestation. She comes into the clinic reporting vaginal bleeding. Which assessment finding best correlates with a diagnosis of hydatidiform mole? a. Dark red, "clumpy" vaginal discharge b. Painful uterine contractions and nausea c. Brisk deep tendon reflexes and shoulder pain d. Bright red, painless vaginal bleeding

a. Dark red, "clumpy" vaginal discharge If a complete molar pregnancy continues into the second trimester undetected, other signs and symptoms appear. The woman often presents with complaints of dark to bright red vaginal bleeding and pelvic pain. Infrequently, she will report passage of grapelike vesicles.

The client in active labor overhears the nurse state the fetus is ROA. The nurse should explain this refers to which component when the client becomes concerned? a. Fetal position b. Fetal attitude c. Fetal station d. Fetal size

a. Fetal position When documenting the ROA, this is the right occiput anterior or the relationship of the fetal position to the mother using the maternal pelvis as the point of reference. Fetal station refers to the relationship of the presenting part of the fetus to the ischial spines of the pelvis. Fetal attitude refers to the relationship of the fetal parts to one another. Fetal size refers the actual size of the developing fetus.

The nurse is assisting a client through labor, monitoring her closely now that she has received an epidural. Which finding should the nurse prioritize to the anesthesiologist? a. Inability to push b. Dry, cracked lips c. Rapid progress of labor d. Urinary retention

a. Inability to push If the client is not able to push, her epidural dose may need to be adjusted to decrease the impact on the sensory system. Dry lips indicate that she may need fluids, so the nurse should give her some ice chips or a drink of water. Urinary retention and rapidly progressing labor should be directly reported to the obstetrician, not the anesthesiologist.

A young mother has tested positive for HIV. When discussing the situation with the client, the nurse should advise the mother that she should avoid which activity? a. future pregnancies b. cesarean birth c. handling the infant with open sores d. breastfeeding

d. breastfeeding Breastfeeding is a major contributing factor for mother-to-child transmission of HIV. Cesarean birth before the onset of labor and/or rupture of membranes can greatly reduce the chance of transmitting the infection to the infant. Future pregnancies should be discussed and decided on an individual basis. Proper treatment of any open wounds and education should be provided to the mother to ensure she reduces the chance of transmitting HIV to her infant.

A new dad is alarmed at the shape of his newborn's head. When responding to the dad, the nurse reminds him this is due to: a. a congenital defect. b. extreme pressure in the vaginal vault. c. prolonged labor. d. cranial bones overlapping at the suture lines.

d. cranial bones overlapping at the suture lines. This is due to molding, which is the result of overlapping of the cranial bones at the suture lines. It is a temporary situation that will correct itself. It is due to the fetus passing through the pelvis. Molding is not the result of extreme pressure, a congenital defect, or prolonged labor.

The nurse assesses a client in labor and finds that the fetal long axis is longitudinal to the maternal long axis. How should the nurse document this finding? a. presentation b. attitude c. position d. lie

d. lie The nurse is assessing fetal lie, the relationship of the fetal long axis to the maternal long axis. When the fetal long axis is longitudinal to the maternal long axis, the lie is said to be longitudinal. Presentation is the portion of the fetus that overlies the maternal pelvic inlet. Attitude is the relationship of the different fetal parts to one another. Position is the relationship of the fetal denominator to the different sides of the maternal pelvis.

Part of the assessment of the first prenatal visit includes screening for rubella antibodies. The nurse determines that a client with which titer shows evidence of immunity against rubella? a. 1:8 b. 1:6 c. 1:4 d. 1:0

a. 1:8 A rubella antibody titer of 1:8 or greater proves evidence of immunity. Women with titers of less than 1:8 should be immunized

The nurse is assisting a client in labor and delivery and notes the placenta is now delivered. Which documentation should the nurse prioritize? a. The completion of the third stage of labor b. The transition phase c. The client's vital signs d. The end of recovery

a. The completion of the third stage of labor The third stage of labor ends with the expulsion of the placenta. Transition precedes the second stage and recovery follows later. The client's vital signs should be monitored throughout the entire process but are not the priority with the expulsion of the placenta.

A woman in active labor has just had her membranes ruptured to speed up labor. The nurse is concerned the woman is experiencing a prolapse of the umbilical cord when the nurse notices which pattern on the fetal heart monitor? a. variable deceleration pattern b. early deceleration with each contraction c. late deceleration with late recovery following contraction d. fetal heart rate (FHR) increase to 200 beats/min

a. variable deceleration pattern Umbilical cord prolapse can be seen after the membranes have ruptured, when the FHR is displaying a sudden variable deceleration FHR pattern on a fetal monitor. It is not uncommon for FHR to increase following a procedure. Early deceleration with each contraction is seen when the fetal head is being compressed through the pelvic opening. Late deceleration with late recovery following contraction is associated with uteroplacental insufficiency (UPI).

A nursing instructor is teaching students about preexisting illnesses and how they can complicate a pregnancy. The instructor recognizes a need for further education when one of the students makes which statement? a. "A pregnant woman needs to be careful of and cautious about accidents and illnesses during her pregnancy." b. "A pregnant woman does not have to worry about contracting new illnesses during pregnancy." c. "A pregnant woman with a chronic condition can put herself at risk." d. "A pregnant woman with a chronic illness can put the fetus at risk."

b. "A pregnant woman does not have to worry about contracting new illnesses during pregnancy." When a woman enters a pregnancy with a chronic illness, it can put both her and the fetus at risk. She needs to be cautious about developing a new illness during her pregnancy as well as having an accident during the pregnancy.

The nurse is assessing a pregnant client who has a long history of asthma. She states, "I'm trying not to use my asthma medications because I certainly don't want my baby exposed to them." What is the nurse's best response? a. "In fact, most modern asthma medications are categorized as safe for use in pregnancy." b. "Actually, having uncontrolled asthma is much riskier for your baby than the medication." c. "Your health care provider will likely agree with your decision." d. "I'm glad to hear that you're focused on ensuring your baby's health."

b. "Actually, having uncontrolled asthma is much riskier for your baby than the medication." It is important for pregnant clients with asthma to keep taking their medications because the risks of exacerbations exceed the risks of the medications.

A woman in active labor with a history of two previous cesarean births is being monitored frequently as she tries to have a vaginal birth. Suddenly, the woman grabs the nurse's hand and states, "Something inside me is tearing." The nurse notes her blood pressure is 80/50 mm Hg, pulse rate is 130 bpm and weak, the skin is cool and clammy, and the fetal monitor shows bradycardia. The nurse activates the code team because the nurse suspects the client may be experiencing which complication? a. compression on the inferior vena cava b. uterine rupture c. an amniotic embolism to the lungs d. an undiagnosed abdominal aorta aneurysm

b. uterine rupture If a uterus should rupture, the woman experiences a sudden, severe pain during a strong labor contraction, which she may report as a "tearing" sensation. Because the uterus at the end of pregnancy is such a vascular organ, uterine rupture is an immediate emergency. Signs of hypotensive shock begin, including a rapid, weak pulse, falling blood pressure, cold and clammy skin, and dilation of the nostrils from air starvation. Fetal heart sounds fade and then are absent.

A nurse is assessing the following antenatal clients. Which client is at highest risk for having a multiple gestation? a. the 27-year-old client who gave birth to twins two years ago b. the 38-year-old client whose spouse is a triplet c. the 41-year-old client who conceived by in vitro fertilization d. the 19-year-old client diagnosed with polycystic ovarian syndrome

c. the 41-year-old client who conceived by in vitro fertilization The nurse should assess infertility treatment as a contributor to increased probability of multiple gestations. Multiple gestations do not occur with an adolescent birth; instead, chances of multiple gestations are known to increase due to the increasing number of women giving birth at older ages.

A woman of 16 weeks' gestation telephones the nurse because she has passed some "berry-like" blood clots and now has continued dark brown vaginal bleeding. Which action would the nurse instruct the woman to do? a. "Come to the health care facility if uterine contractions begin." b. "Maintain bed rest, and count the number of perineal pads used." c. "Continue normal activity, but take the pulse every hour." d. "Come to the health facility with any vaginal material passed."

d. "Come to the health facility with any vaginal material passed." This is a typical time in pregnancy for gestational trophoblastic disease to present. Asking the woman to bring any material passed vaginally would be important so it can be assessed for this.

The nurse is assessing a woman with Class III heart disease who is in for a prenatal visit. What would be the first recognizable sign that this client is in heart failure? a. Low blood pressure b. Audible wheezes c. Elevated blood pressure d. Persistent rales in the bases of the lungs

d. Persistent rales in the bases of the lungs The earliest warning sign of cardiac decompensation is persistent rales in the bases of the lungs.

A client has just given birth to a healthy baby boy, but the placenta has not yet delivered. What stage of labor does this scenario represent? a. Fourth b. Second c. First d. Third

d. Third Stage three begins with the birth of the baby and ends with delivery of the placenta.

The nurse plays a major role in assessing the progress of labor. The nurse integrates understanding of the typical rule for monitoring labor progress. Which finding would the nurse correlate with this rule? a. Cervix dilates 1 cm per hour. b. Fetus descends 1 cm per hour. c. Fetus descends 2 cm per hour. d. Cervix dilates 2 cm per hour.

a. Cervix dilates 1 cm per hour. A simple rule for evaluating the progress of labor is expecting 1 cm per hour of cervical dilation. If the cervix fails to respond to uterine contractions by dilating and effacing, then dysfunctional labor must be ruled out.

The nurse is instructing on maternal hormones which may impact the onset of labor. Which hormones are included in the discussion? Select all that apply. a. Progesterone b. Prostaglandins c. Insulin d. Testosterone e. Thyroxin e. Oxytocin

a. Progesterone b. Prostaglandins e. Oxytocin There are several hypotheses regarding what triggers labor to begin. Progesterone is the hormone of pregnancy and elimination may cause the uterus to contract. Oxytocin also causes the uterus to contract. Prostaglandins causes the cervix to soften and also causes the uterus to contract. Testosterone, thyroxin and insulin are not one of the main factors in the onset of labor theories.

Which neonatal assessment is the highest priority if the mother received meperidine during labor? a. Respiratory rate b. Time of first meconium c. Temperature regulation d. Lung sounds

a. Respiratory rate The fetal respiratory rate immediately after delivery is assessed as respiratory depression is a side effect of the maternal dose of meperidine. Meperidine crosses the placenta and can remain in the fetal system. The other options are assessed and monitored but for rationale not related to the opioid medication administration.

The nurse notes that the fetal head is at the vaginal opening and does not regress between contractions. The nurse interprets this finding as which process? a. crowning b. restitution c. descent d. engagement

a. crowning Crowning occurs when the top of the fetal head appears at the vaginal orifice and no longer regresses between contractions. Engagement occurs when the greatest transverse diameter of the head passes through the pelvic inlet. Descent is the downward movement of the fetal head until it is within the pelvic inlet. Restitution or external rotation occurs after the head is born and free of resistance. It untwists, causing the occiput to move about 45 degrees back to its original left or right position.

A nursing instructor identifies which factor as increasing the chances of infection when coupled with prolonged labor? a. premature rupture of membranes b. multiple births c. maternal age d. number of previous pregnancies

a. premature rupture of membranes The risk for infection increases during prolonged labor particularly in association with premature rupture of membranes. The other options do not increase the risk of infection during labor.

A pregnant client has been diagnosed with gestational diabetes. Which are risk factors for developing gestational diabetes? Select all that apply. a. previous large for gestational age (LGA) infant b. maternal age less than 18 years c. hypertension d. obesity e. genitourinary tract abnormalities

a. previous large for gestational age (LGA) infant c. hypertension d. obesity Obesity, hypertension, and a previous infant weighing more than 9 lb (4 kg) are risk factors for developing gestational diabetes. Maternal age less than 18 years and genitourinary tract abnormalities do not increase the risk of developing gestational diabetes.

Assessment of a woman in labor reveals that the scapula of the fetus is the presenting part. The nurse interprets this finding as indicating which fetal presentation? a. shoulder b. vertex c. breech d. cephalic

a. shoulder The three main fetal presentations are cephalic or vertex, with the head as the presenting part, breech, with the pelvis as the presenting part, and shoulder, with the scapula as the presenting part.

A woman states that she does not want any medication for pain relief during labor. Her primary care provider has approved this for her. What the nurse's best response to her concerning this choice? a. "That's wonderful. Medication during labor is not good for the baby." b. "I respect your preference whether it is to have medication or not." c. "Your health care provider is a man and has never been in labor; he may be underestimating the pain you will have." d. "Let me get you something for relaxation if you don't want anything for pain."

b. "I respect your preference whether it is to have medication or not." Individualizing care to meet women's specific needs is a nursing responsibility.

A client with a history of cervical insufficiency is seen for reports of pink-tinged discharge and pelvic pressure. The primary care provider decides to perform a cervical cerclage. The nurse teaches the client about the procedure. Which client response indicates that the teaching has been effective? a. "Staples are put in the cervix to prevent it from dilating." b. "Purse-string sutures are placed in the cervix to prevent it from dilating." c. "The cervix is glued shut so no amniotic fluid can escape." d. "A cervical cap is placed so no amniotic fluid can escape."

b. "Purse-string sutures are placed in the cervix to prevent it from dilating." The cerclage, or purse string suture is inserted into the cervix to prevent preterm cervical dilatation and pregnancy loss. Staples, glue, or a cervical cap will not prevent the cervix from dilating.

A pregnant woman with type 2 diabetes is scheduled for a laboratory test of glycosylated hemoglobin (HbA1C). What does the nurse tell the client is a normal level for this test? a. 12% b. 6% c. 8% d. 14%

b. 6% The upper normal level of HbA1C is 6% of total hemoglobin.

A primagravida has an office appointment in her 39th week of pregnancy. Which assessment data is most definitive of the onset of labor? a. The fetal head is engaged in the pelvis. b. Cervical ripening is noted on examination. c. The mother reports frequent urination. d. Expulsion of the mucous plug.

b. Cervical ripening is noted on examination. Clinical signs that labor is approaching include ripening or softening of the cervix with effacement and dilation. Frequent urination is common during engagement where the fetal head is in the pelvis. This is common up to 2 weeks before true labor begins. Expulsion of the mucous plug also is common a week or two before labor begins.

A client's membranes rupture. The nurse observes the fetal heart rate drop from 156 to 110. The nurse inspects the client's perineum and sees a loop of umbilical cord. What is the nurse's priority concern in this situation? a. Increased risk for placental abruption b. Decreased fetal oxygenation c. Decreased strength of uterine contractions d. Increased risk for infection

b. Decreased fetal oxygenation When there is a cord prolapse the cord becomes compressed, blood flow is interrupted, and there is decreased oxygen available to the fetus resulting in fetal distress. There is a slight increased risk for postbirth infection, but it is not the priority at this time. A cord prolapse does not increase the risk for placental abruption nor does it decrease the strength of uterine contractions.

The nurse is assessing a mother who just delivered a 7 lb (3136 g) baby via cesarean delivery. Which assessment finding should the nurse prioritize if the mother has a history of controlled atrial fibrillation? a. Abdominal cramps b. Jugular distention c. Urinary retention d. Nausea and vomiting

b. Jugular distention A woman who has a cardiac condition is at increased risk in the postpartum period. The most important nursing action is to monitor for signs of cardiac decompensation. The nurse should monitor for and report jugular distention, clubbing, and slow capillary refill time. If an irregular pulse is noted, compare it to the apical pulse. The abdominal cramps may be related to the uterus involution. The nausea and vomiting and urinary retention may be related to the surgical procedure and not necessarily the cardiac issue.

The nurse is preparing an educational event for pregnant women on the topic of labor pain and birth. The nurse understands the need to include the origin of labor pain for each stage of labor. What information will the nurse present for the first stage of labor? a. Pain is focal in nature. b. Pain originates from the cervix and lower uterine segment. c. Diffuse abdominal pain signals a complication with progression of labor. d. It is reported as the worst pain a woman will ever feel.

b. Pain originates from the cervix and lower uterine segment. Pain sensations associated with labor originate from different places depending on the stage of labor. During the first stage of labor, the stretching required to efface and dilate the cervix stimulates pain receptors in the cervix and lower uterine segment.

A 24-year-old primigravida client at 39 weeks' gestation presents to the OB unit concerned she is in labor. Which assessment findings will lead the nurse to determine the client is in true labor? a. After walking for an hour, the contractions have not fully subsided. b. The client reports back pain, and the cervix is effacing and dilating. c. The contraction pains are 2 minutes apart and 1 minute in duration. d. The contraction pains have been present for 5 hours, and the patterns are regular.

b. The client reports back pain, and the cervix is effacing and dilating. True labor is indicated when the cervix is changing. Contractions occur for weeks before true labor, and may occur close together. Contractions may also occur for a long time before true labor begins.

A client has just given birth to a healthy baby boy, but the placenta has not yet delivered. What stage of labor does this scenario represent? a. Fourth b. Third c. First d. Second

b. Third Stage three begins with the birth of the baby and ends with delivery of the placenta.

Immediately after giving birth to a full-term infant, a client develops dyspnea and cyanosis. Her blood pressure decreases to 60/40 mm Hg, and she becomes unresponsive. What does the nurse suspect is happening with this client? a. placental separation b. amniotic fluid embolism c. congestive heart failure d. aspiration

b. amniotic fluid embolism With amniotic fluid embolism, symptoms may occur suddenly during or immediately after labor. The woman usually develops symptoms of acute respiratory distress, cyanosis, and hypotension.

When explaining to a class of pregnant women why labor begins, the nurse will include the fact that there are several theories that have been proposed to explain why labor begins, although none have been proven scientifically. Which idea is one of those theories? a. decrease in the level of estrogen b. change in estrogen-to-progesterone ratio c. decrease in number of oxytocin receptors d. decrease in prostaglandins, leading to myometrium contractions

b. change in estrogen-to-progesterone ratio One of the theories suggests that labor is initiated by a change in the estrogen-to-progesterone ratio. The number of oxytocin receptors have been noted to increase. Estrogen levels also increase, which in turn increases myometrial sensitivity to oxytocin. Prostaglandin levels also increase, which in turn leads to myometrial contractions.

A client has been admitted with abruptio placentae. She has lost 1,200 mL of blood, is normotensive, and ultrasound indicates approximately 30% separation. The nurse documents this as which classification of abruptio placentae? a. grade 1 b. grade 2 c. grade 4 d. grade 3

b. grade 2 The classifications for abruptio placentae are: grade 1 (mild) - minimal bleeding (less than 500 mL), 10% to 20% separation, tender uterus, no coagulopathy, signs of shock or fetal distress; grade 2 (moderate) - moderate bleeding (1,000 to 1,500 mL), 20% to 50% separation, continuous abdominal pain, mild shock, normal maternal blood pressure, maternal tachycardia; grade 3 (severe) - absent to moderate bleeding (more than 1,500 mL), more than 50% separation, profound shock, dark vaginal bleeding, agonizing abdominal pain, decreased blood pressure, significant tachycardia, and development of disseminated intravascular coagulopathy. There is no grade 4.

A pregnant woman has just found out that she is having twin girls. She asks the nurse the difference between fraternal and identical twins. The nurse explains that with one set of twins there is fertilization of two ova, and with the other set one fertilized ovum splits. What type of twins result from the split ovum? a. neither type results from a split ovum b. identical c. both types can result from the split ovum d. fraternal

b. identical The incidence of twins is about 1 in 30 conceptions, with about 2/3 being from the fertilization of two ova (fraternal) and about 1/3 from the splitting of one fertilized ovum (identical).

A nursing student has learned that precipitous labor is when the uterus contracts so frequently and with such intensity that a very rapid birth will take place. This means the labor will be completed in which span of time? a. less than 4 hours b. less than 3 hours c. less than 5 hours d. less than 8 hours

b. less than 3 hours Precipitous labor is completed in less than 3 hours.

During the second stage of labor, a woman is generally: a. very aware of activities immediately around her. b. turning inward to concentrate on body sensations. c. anxious to have people around her. d. no longer in need of a support person.

b. turning inward to concentrate on body sensations. Second-stage contractions are so unusual that most women are unable to think of things other than what is happening inside their body.

A pregnant client with a history of asthma since childhood presents for a prenatal visit. What statement by the client would the nurse prioritize? a. "Certain substances make me sneeze." b. "I sometimes get a feeling of euphoria." c. "I sometimes get a bit wheezy." d. "I have trouble getting comfortable in bed."

c. "I sometimes get a bit wheezy." Wheezing is a classic symptom of asthma. This statement should alert the nurse to the possibility that the woman's asthma is not being well-controlled and needs further evaluation and possible intervention. The other statements do not relate to the typical presentation of this disease in pregnancy.

A primigravida 28-year-old client is noted to have Rh negative blood and her husband is noted to be Rh positive. The nurse should prepare to administer RhoGAM after which diagnostic procedure? a. Nonstress test b. Biophysical profile c. Amniocentesis d. Contraction test

c. Amniocentesis Amniocentesis is a procedure requiring a needle to enter into the amniotic sac. There is a risk of mixing of the fetal and maternal blood which could result in blood incompatibility. A contraction test, a nonstress test, and biophysical profile are not invasive, so there would be no indication for Rho(D) immune globulin to be administered.

During which time is the nurse correct to document the end of the third stage of labor? a. Following fetal birth b. When pushing begins c. At the time of placental delivery d. When the mother is moved to the postpartum unit

c. At the time of placental delivery The third stage of labor concludes with the delivery of the placenta. The nurse is correct to document that time in the medical record. The beginning of the third stage of labor is the documented time of birth. Neither the time when the woman begins to push nor when she is moved to the postpartum unit are notable.

A nurse places an external fetal monitor on a woman in labor. Which instruction would be best to give her? a. Lie supine so the tracing does not show a shadow. b. Avoid using her call bell to reduce interference. c. Lie on her side so she is comfortable. d. Avoid flexing her knees so her abdomen is not tense.

c. Lie on her side so she is comfortable. The best position for all women during labor is on their side.

The nurse is caring for a client who is late in her pregnancy. What assessment finding should the nurse attribute to the role of prostaglandins? a. The perineum is relaxing b. The uterus is relaxing c. The cervix is softening d. The cervix is dilating

c. The cervix is softening The prostaglandin theory is another theory of labor initiation. Prostaglandins influence labor in several ways, which include softening the cervix and stimulating the uterus to contract. However, evidence supporting the theory that prostaglandins are the agents that trigger labor to begin is inconclusive.

The nurse discovers that the FHM is now recording late decelerations in a client who is in labor. The nurse predicts this is most likely related to which event? a. Maternal hypotension b. Maternal fatigue c. Uteroplacental insufficiency d. Cord compression

c. Uteroplacental insufficiency Late decelerations are associated with uteroplacental insufficiency. They typically indicate decreased blood flow to the uterus during the contractions. Maternal hypotension and fatigue would not be observed on the fetal heart monitor. Cord compression would be marked by fetal tachycardia.

A nurse notes a pregnant woman has just entered the second stage of labor. Which interaction should the nurse prioritize at this time to assist the client? a. completing the identification process of the newborn with the mother b. palpating the woman's fundus for position and firmness c. encouraging the woman to push when she has a strong desire to do so d. alleviating perineal discomfort with the application of ice packs

c. encouraging the woman to push when she has a strong desire to do so During the second stage of labor, nursing interventions focus on motivating the woman, encouraging her to put all her efforts toward pushing. Alleviating perineal discomfort with ice packs and palpating the woman's fundus would be appropriate during the fourth stage of labor. Completing the newborn identification process would be appropriate during the third stage of labor.

A client receives an epidural anesthetic. Which medication would the nurse anticipate the primary care provider will prescribe if the client develops moderate hypotension? a. atropine b. methylergonovine c. ephedrine d. betamethasone

c. ephedrine A hypotensive agent such as ephedrine is given to elevate blood pressure if hypotension occurs.

To give birth to her infant, a woman is asked to push with contractions. Which pushing technique is the most effective and safest? a. lying on side, arms grasped on abdomen b. lying supine with legs in lithotomy stirrups c. head elevated, grasping knees, breathing out d. squatting while holding her breath

c. head elevated, grasping knees, breathing out An important point is to be certain the woman does not hold her breath, as this puts pressure on the vena cava, reducing blood return.

When assessing fetal heart rate patterns, which finding would alert the nurse to a possible problem? a. accelerations b. variable decelerations c. prolonged decelerations d. early decelerations

c. prolonged decelerations Prolonged decelerations are associated with prolonged cord compression, abruptio placentae, cord prolapse, supine maternal position, maternal seizures, regional anesthesia, or uterine rupture. Variable decelerations are the most common deceleration pattern found. They are usually transient and correctable. Early decelerations are thought to be the result of fetal head compression. They are not indicative of fetal distress and do not require intervention. Fetal accelerations are transitory increases in FHR and provide evidence of fetal well-being.

When teaching possible differences in labor between the first labor experience and all other labors, which statement is most beneficial to assist a woman's psyche? a. "The labor process is typically shorter for subsequent pregnancies." b. "The intensity of contractions are much greater throughout the labor." c. "You can have input into the labor plan as you know what to expect." d. "You had a successful labor and vaginal delivery with your first pregnancy."

d. "You had a successful labor and vaginal delivery with your first pregnancy." Reminding the client of her successful labor and birth best provides confidence, which strengthens a woman's psyche. It is true that subsequent pregnancies are typically shorter in length, and input in the labor plan by the multipara client can be expressed (since the woman has already experienced the process), but these are not as important as successfully completing the process. Depending upon the client's past experiences, the intensity of the contractions may or may not be more intense.

The nurse has been monitoring a multipara client for several hours. She cries out that her contractions are getting harder and that she cannot do this. The nurse notes the client is very irritable, nauseated, annoyed, and doesn't want to be left alone. Based on the assessment the nurse predicts the cervix to be dilated how many centimeters? a. 5 to 7 b. 0 to 2 c. 3 to 4 d. 8 to 10

d. 8 to 10 The reaction of the client is indicative of entering or being in the transition phase of labor, stage 1. The dilation would be 8 cm to 10 cm. Before that, when dilation is 0 to 7 cm, the client has an easier time using positive coping skills.

A woman is going to have labor induced with oxytocin. Which statement reflects the induction technique the nurse anticipates the primary care provider will prescribe? a. Administer oxytocin in two divided intramuscular sites. b. Administer oxytocin diluted in the main intravenous fluid. c. Administer oxytocin in a 20 cc bolus of saline. d. Administer oxytocin diluted as a "piggyback" infusion.

d. Administer oxytocin diluted as a "piggyback" infusion. Oxytocin is always infused in a secondary or "piggyback" infusion system so it can be halted quickly if overstimulation of the uterus occurs.

How does a woman who feels in control of the situation during labor influence her pain? a. There is no association between the two factors. b. Feeling in control shortens the overall length of labor. c. Decreased feeling of control helps during the third stage. d. Feelings of control are inversely related to the client's report of pain.

d. Feelings of control are inversely related to the client's report of pain. Studies reveal that women who feel in control of their situation are apt to report less pain than those who feel they have no control.

A client has presented in the early phase of labor, experiencing abdominal pain and signs of growing anxiety about the pain. Which pain management technique should the nurse prioritize at this stage? a. Immersing the client in warm water in a pool or hot tub b. Administering a sedative such as secobarbital or pentobarbital c. Administering an opioid such as meperidine or fentanyl d. Practicing effleurage on the abdomen

d. Practicing effleurage on the abdomen In early labor, the less medication use the better; allow use of nonpharmacologic management and control the pain with effleurage. Sitting in a warm pool of water is relaxing and may lessen the pain, but it does not control the pain. Sedatives are not indicated as they may slow the birthing process. Opioids should be limited as they too may slow the progression of labor.

A young mother gives birth to twin boys who shared the same placenta. What serious complication are they at risk for? a. ABO incompatibility b. HELLP syndrome c. TORCH syndrome d. Twin-to-twin transfusion syndrome (TTTS)

d. Twin-to-twin transfusion syndrome (TTTS) When twins share a placenta, a serious condition called twin-to-twin transfusion syndrome (TTTS) can occur.

A woman experiences an amniotic fluid embolism as the placenta is delivered. The nurse's first action would be to: a. increase her intravenous fluid infusion rate. b. tell the woman to take short, catchy breaths. c. put firm pressure on the fundus of her uterus. d. administer oxygen by mask.

d. administer oxygen by mask. An amniotic embolism quickly becomes a pulmonary embolism. The woman needs oxygen to compensate for the sudden blockage of blood flow through her lungs.

A pregnant client with type I diabetes asks the nurse about how to best control her blood sugar while she is pregnant. The best reply would be for the woman to: a. limit weight gain to 15 pounds during the pregnancy. b. exercise for 1 to 2 hours each day to keep the blood glucose down. c. begin oral hyperglycemic medications along with the insulin she is currently taking. d. check her blood sugars frequently and adjust insulin accordingly.

d. check her blood sugars frequently and adjust insulin accordingly. The goal for a mother who has type I diabetes mellitus is to keep tight control over her blood sugars throughout the pregnancy. Therefore, she needs to test her blood sugar frequently during the day and make adjustments in the insulin doses she is receiving.

The nurse is providing care to a neonate. Review of the maternal history reveals that the mother is suspected of heroin use disorder. The nurse would be alert for which finding when assessing the neonate? a. low, feeble cry b. easy consolability c. vigorous sucking d. hypertonicity

d. hypertonicity Newborns of mothers with heroin or other narcotic use disorder display irritability, hypertonicity, a high-pitched cry, vomiting, diarrhea, respiratory distress, disturbed sleeping, sneezing, diaphoresis, fever, poor sucking, tremors, and seizures.

Place the following stages of labor in order from what occurs first to last. All options must be used. 1. third stage 2. active stage 3. transition stage 4. latent stage 5. second stage

4. latent stage 2. active stage 3. transition stage 5. second stage 1. third stage The latent stage is the beginning of labor from approximately 0 cm dilated to 3 to 4 cm dilated. The active stage is from when the cervix is 4 cm dilated to 8 cm dilated. Active labor begins to become more uncomfortable for the client, and the contractions are usually stronger, longer, and closer together. Transition is approximately 8 to 10 cm. The second stage of labor is from when the client is 10 cm dilated to birth of the baby. The third stage of labor is from the birth of the baby to the delivery of the placenta.

The nursing student demonstrates an understanding of dystocia with which statement? a. "Dystocia is diagnosed after labor has progressed for a time." b. "Dystocia is not diagnosed until after the birth." c. "Dystocia cannot be diagnosed until just before birth." d. "Dystocia is diagnosed at the start of labor."

a. "Dystocia is diagnosed after labor has progressed for a time." Nursing management of the woman with dystocia, regardless of etiology, requires patience. The nurse needs to provide physical and emotional support to the client and family. Dystocia is diagnosed not at the start of labor, but rather after it has progressed for a time.

Which change in insulin is most likely to occur in a woman during pregnancy? a. less effective than normal b. enhanced secretion from normal c. not released because of pressure on the pancreas d. unavailable because it is used by the fetus

a. less effective than normal Somatotropin released by the placenta makes insulin less effective. This is a safeguard against hypoglycemia.

A nurse is monitoring a female client with an epidural block. Which complication would be the most important for the nurse to monitor in the client? a. respiratory depression b. a failed block c. postdural puncture headache d. accidental intrathecal block

a. respiratory depression Respiratory depression is a complication of epidural anesthesia and should be closely monitored in laboring clients. A failed block, accidental intrathecal block, and a postdural headache are all side effects of a spinal epidural block.

The client is being rushed into the labor and delivery unit. At which station would the nurse document the fetus immediately prior to birth? a. +1 b. +4 c. -5 d. 0

b. +4 As the fetus is being born, the station is +4. The fetus is floating and not engaged in the pelvis at -5. The fetus is at the level of the ischial spines and engaged at station 0. The fetus is progressing down the birth canal below the ischial spines at +1.

A shoulder dystocia situation is called in room 4. The nurse enters the room to help and the health care provider says to the nurse, "McRoberts maneuver." What does the nurse do next? a. Push the fetal head back into the uterus and prepare the client for cesarean birth b. Bring the client's knees back toward the shoulders, causing hyperflexion of the hips and rotation of the pubic symphysis c. Apply downward pressure above the pubic bone of the client, in an attempt to rotate the anterior shoulder d. Move the client into a hands-and-knees position, to straighten the sacral curve and release the posterior shoulder

b. Bring the client's knees back toward the shoulders, causing hyperflexion of the hips and rotation of the pubic symphysis To implement McRoberts maneuver, the nurse brings the client's knees back toward the shoulders, causing hyperflexion of the hips and rotation of the pubic symphysis. This maneuver enlarges the space for delivery of the fetal shoulders. Applying pressure above the pubic bone is suprapubic pressure. Pushing the fetal head back into the vagina is a Zavanelli maneuver. Since the fetal head has been delivered, it is not safe to move the client to a hands-and-knees position.

The nurse is caring for a pregnant client with severe preeclampsia. Which nursing intervention should a nurse perform to institute and maintain seizure precautions in this client? a. Provide a well-lit room. b. Keep the suction equipment readily available. c. Place the client in a supine position. d. Keep head of bed slightly elevated.

b. Keep the suction equipment readily available. The nurse should institute and maintain seizure precautions such as padding the side rails and having oxygen, suction equipment, and call light readily available to protect the client from injury. The nurse should provide a quiet, darkened room to stabilize the client. The nurse should maintain the client on complete bed rest in the left lateral lying position and not in a supine position. Keeping the head of the bed slightly elevated will not help maintain seizure precautions.

The nurse is monitoring a client in labor who has had a previous cesarean section and is trying a vaginal birth with an epidural. The nurse observes a sudden drop in blood pressure, increased heart rate, and deep variable deceleration on the fetal monitor. The client reports severe pain in her abdomen and shoulder. What should the nurse prepare to do? a. Turn the client on her left side. b. Prepare the client for a cesarean birth. c. Place the client in a knee-chest position. d. Bolus the client with another dose of medication through the epidural.

b. Prepare the client for a cesarean birth. The findings are consistent with uterine rupture. An abrupt change in the fetal heart rate pattern is often the most significant finding associated with uterine rupture. Others are reports of pain in the abdomen, shoulder, or back in a laboring woman who had previous good pain relief from epidural anesthesia. Falling blood pressure and rising pulse may be associated with hypovolemia caused by occult bleeding. The treatment is immediate cesarean birth.

The nurse is caring for a client with preeclampsia and understands the need to auscultate this client's lung sounds every 2 hours. Why would the nurse do this? a. Pulmonary hypertension b. Pulmonary edema c. Pulmonary emboli d. Pulmonary atelectasis

b. Pulmonary edema In the hospital, monitor blood pressure at least every 4 hours for mild preeclampsia and more frequently for severe disease. In addition, it is important to auscultate the lungs every 2 hours. Adventitious sounds may indicate developing pulmonary edema.

The pain of labor is influenced by many factors. What is one of these factors? a. The woman has a high threshold for pain. b. The woman is prepared for labor and birth. c. The woman has a high tolerance for pain. d. The woman has lots of visitors during labor.

b. The woman is prepared for labor and birth. The woman who enters labor with realistic expectations usually copes well and reports a more satisfying labor experience than does a woman who is not as well prepared.

A novice nurse asks to be assigned to the least complex antepartum client. Which condition would necessitate the least complex care requirements? a. placenta previa b. gestational hypertension c. abruptio placenta d. preecalmpsia

b. gestational hypertension Hypertensive disorders represent the most common complication of pregnancy. Gestational hypertension is elevated blood pressure without proteinuria, other signs of preeclampsia, or preexisting hypertension. Abruptio placenta (separation of the placenta from the uterine wall), placenta previa (placenta covering the cervical os), and preeclampsia are high-risk, potentially life-threatening conditions for the fetus and mother during labor and birth.

A nurse is caring for a pregnant client who is in labor. Which maternal physiologic responses should the nurse monitor for in the client as the client progresses through birth? Select all that apply. a. increase in gastric emptying and pH b. increase in respiratory rate c. slight decrease in body temperature d. increase in heart rate e. increase in blood pressure

b. increase in respiratory rate d. increase in heart rate e. increase in blood pressure When caring for a client in labor, the nurse should monitor for an increase in the heart rate by 10 to 20 bpm, an increase in blood pressure by as much as 35 mm Hg, and an increase in respiratory rate. During labor, the nurse should monitor for a slight elevation in body temperature as a result of an increase in muscle activity. The nurse should also monitor for decreased gastric emptying and gastric pH, which increases the risk of vomiting with aspiration.

A woman with systemic lupus erythematosus is interested in preconception counseling to discuss her desire to get pregnant. The nurse explains that it would be best if she is symptom-free or in remission for how long before getting pregnant? a. 12 months b. 9 months c. 6 months d. 3 months

c. 6 months If the woman is considering pregnancy, it is recommended that she postpone conception until the disease has been stable or in remission for six months. Active disease at the time of conception and history of renal disease increase the likelihood of a poor pregnancy outcome.

If the monitor pattern of uteroplacental insufficiency were present, which action would the nurse do first? a. Ask her to pant with the next contraction. b. Administer oxygen at 3 to 4 L by nasal cannula. c. Turn her or ask her to turn to her side. d. Help the woman to sit up in a semi-Fowler's position.

c. Turn her or ask her to turn to her side. The most common cause of uteroplacental insufficiency is compression of the vena cava; turning the woman to her side removes the compression.

A client is 33 weeks pregnant and has had diabetes since age 21. When checking her fasting blood glucose level, which value would indicate the client's disease is controlled? a. 45 mg/dL b. 120 mg/dL c. 136 mg/dL d. 85 mg/dL

d. 85 mg/dL Recommended fasting blood glucose levels in pregnant clients with diabetes are 60 to 95 mg/dL. A fasting blood glucose level of 45 g/dL is low and may result in symptoms of hypoglycemia. A blood glucose level below 120 mg/dL is recommended for 2-hour postprandial values. A blood glucose level above 136 mg/dL in a pregnant client indicates hyperglycemia.

Which cardinal movement of delivery is the nurse correct to document by station? a. Extension b. Internal rotation c. Flexion d. Descent

d. Descent Descent is documented by station, which is the relationship of the fetal presenting part to the maternal ischial spines. Descent continues throughout labor until the fetus reaches the fetal station of +4. The other options represent fetal movements to accommodate the passage of the fetus.

A pregnant woman has been admitted to the hospital due to severe preeclampsia. Which measure will be important for the nurse to include in the care plan? a. Institute NPO status. b. Plan for immediate induction of labor. c. Admit the client to the middle of ICU where she can be constantly monitored. d. Institute and maintain seizure precautions.

d. Institute and maintain seizure precautions. The woman with severe preeclampsia should be maintained on complete bed rest in a dark and quiet room to avoid stimulation. The client is at risk for seizures; therefore, institution and maintenance of seizure precautions should be in place.

The nurse is monitoring a client who is in labor and notes the client is happy, cheerful, and "ready to see the baby." The nurse interprets this to mean the client is in which stage or phase of labor? a. Transition phase b. Stage three c. Stage two d. Latent phase

d. Latent phase The woman in labor undergoes numerous psychological adaptations during labor. During the latent phase, she is often talkative and happy, and yet anxious. During transition, the client may show fear and anger. During stage two she may remain positive, but the work of labor is very intense.

The nurse is caring for a client who is sent to the obstetric unit for evaluation of fetal well-being. At which location is the nurse correct to place the tocodynamometer? a. On the right side of the abdomen b. At the level of the umbilicus c. Midline but low on the abdomen d. On the uterine fundus

d. On the uterine fundus The nurse is correct to place the tocodynamometer on the fundus with the sensor facing downward and then strap it securely to the abdomen. The other positions will relay information as well.

The student nurse is learning about normal labor. The teacher reviews the cardinal movements of labor and determines the instruction has been effective when the student correctly states the order of the cardinal movements as follows: a. internal rotation, flexion, descent, extension, external rotation, expulsion. b. descent, flexion, external rotation, extension, internal rotation, expulsion. c. internal rotation, flexion, descent, extension, external rotation, expulsion. d. descent, flexion, internal rotation, extension, external rotation, expulsion.

d. descent, flexion, internal rotation, extension, external rotation, expulsion. The six cardinal movements of the fetus are descent, flexion, internal rotation, extension, external rotation, and expulsion.

A woman is being closely monitored and treated for severe preeclampsia with magnesium sulfate. Which finding would alert the nurse to the development of magnesium toxicity in this client? a. elevated liver enzymes b. seizures c. serum magnesium level of 6.5 mEq/L d. diminished reflexes

d. diminished reflexes Diminished or absent reflexes occur when a client develops magnesium toxicity. Elevated liver enzymes are unrelated to magnesium toxicity and may indicate the development of HELLP syndrome. The onset of seizure activity indicates eclampsia. A serum magnesium level of 6.5 mEq/L would fall within the therapeutic range of 4 to 7 mEq/L.

The nurse is monitoring the uterine contractions of a woman in labor. The nurse determines the woman is experiencing hypertonic uterine dysfunction based on which contraction finding? a. poor in quality. b. well coordinated. c. brief. d. erratic.

d. erratic. Hypertonic contractions occur when the uterus never fully relaxes between contractions, making the contractions erratic and poorly coordinated because more than one uterine pacemaker is sending signals for contraction. Hypotonic uterine contractions are poor in quality, brief, and lack sufficient intensity to dilate and efface the cervix.

A client in active labor is given spinal anesthesia. Which information would the nurse include when discussing with the client and family about the disadvantages of spinal anesthesia? a. increased frequency of micturition b. excessive contractions of the uterus c. passage of the drug to the fetus d. headache following anesthesia

d. headache following anesthesia The nurse should inform the client and her family about the possibility of headache after spinal anesthesia. The drug is retained in the mother's body and not passed to the fetus. There may be uterine atony, and not excessive uterine contractions, following spinal anesthesia. Spinal anesthesia may lead to bladder atony, and not an increased frequency of micturition.

During the assessment of a woman in labor, the nurse explains that certain landmarks are used to determine the progress of the birth. The nurse identifies which area as one of these landmarks? a. ischial tuberosity b. pubic symphysis c. cervical os d. ischial spine

d. ischial spine Station is assessed in relation to the maternal ischial spines and the presenting fetal part. These spines are not sharp protrusions but rather blunted prominences at the midpelvis. The ischial spines serve as landmarks and have been designated as zero station.

A nurse is caring for a newborn with fetal alcohol spectrum disorder. What characteristic of the fetal alcohol spectrum disorder should the nurse assess for in the newborn? a. wide eyes b. decreased blood glucose level c. poor breathing pattern d. small head circumference

d. small head circumference The nurse should assess for small head circumference in a newborn being assessed for fetal alcohol spectrum disorder. Fetal alcohol spectrum disorder does not cause decreased blood glucose level, a poor breathing pattern, or wide eyes.

A pregnant woman has arrived to the office reporting vaginal bleeding. Which finding during the assessment would lead the nurse to suspect an inevitable abortion? a. no passage of fetal tissue b. slight vaginal bleeding c. closed cervical os d. strong abdominal cramping

d. strong abdominal cramping Strong abdominal cramping is associated with an inevitable spontaneous abortion. Slight vaginal bleeding early in pregnancy and a closed cervical os are associated with a threatened abortion. With an inevitable abortion, passage of the products of conception may occur. No fetal tissue is passed with a threatened abortion.

Assessment reveals that the fetus of a client in labor is in the vertex presentation. The nurse determines that which part is presenting? a. brow b. buttocks c. occiput d. shoulders

c. occiput

A pregnant client with cystic fibrosis (CF) comes to the office for a prenatal visit. She asks the nurse for information on breast-feeding. The best response by the nurse is: a. "Breast-feeding is not a good idea. Because your breast milk is high in sodium due to CF, there is a risk of the infant receiving too much sodium." b. "Breast-feeding is a great idea. It will help put your CF into remission." c. "Breast-feeding is a great idea. Let me get you some information." d. "Breast-feeding is not a good idea for you since you have a comorbidity."

a. "Breast-feeding is not a good idea. Because your breast milk is high in sodium due to CF, there is a risk of the infant receiving too much sodium." The milk of a nursing mother with cystic fibrosis is high in sodium. This potentially places the infant at risk for hypernatremia, that is, too much sodium. Provide the client with as much correct information as possible, and explain medical terms in layperson's language.

The nurse is is giving discharge instructions to a client who experienced a complete spontaneous abortion. Which question should the nurse prioritize at this time? a. "Do you have someone to talk to, or may I give you the names and numbers for some possible grief counselors?" b. "May I give you some resources that you can use to try to prevent this from happening again?" c. "Are you going to wait a while before you try to get pregnant again?" d. "Did you know that 75 percent of women who are trying to get pregnant experience this same thing?"

a. "Do you have someone to talk to, or may I give you the names and numbers for some possible grief counselors?" When a woman has a spontaneous abortion, or miscarriage, one important consideration is the emotional needs of the woman once she is home. She may not want to talk about the loss for a period of time, but the nurse needs to determine her support system for the future. Asking the woman if she is "going to try again" is an inappropriate question for the nurse to ask and diminishes the experience of having a spontaneous abortion. It would be inappropriate to point out the woman is not the only one to have this experience or to offer ways to prevent it from happening in the future. The woman needs to deal with this situation first before moving on to a possible "next" time.

The nurse is caring for a client after experiencing a placental abruption. Which finding is the priority to report to the health care provider? a. 45 ml urine output in 2 hours b. hematocrit of 36% c. platelet count of 150,000 mm3 d. hemoglobin of 13 g/dl

a. 45 ml urine output in 2 hours The nurse knows a placental abruption places the client at high risk of hemorrhage. A decreased urine output indicates decreased perfusion from blood loss. The hematocrit, hemoglobin, and platelet counts are all within expected levels.

At 24 weeks' gestation, a client's 1-hour glucose tolerance test is elevated. The nurse explains that, based on this finding, the client will need to take which action? a. A 3-hour glucose tolerance test for follow-up b. Daily insulin injections for gestational diabetes c. Daily fingersticks for a fasting blood glucose level d. Monthly hemoglobin A1C levels to rule out diabetes

a. A 3-hour glucose tolerance test for follow-up The 1-hour glucose tolerance test is a screening procedure. If the results are elevated, the client needs a 3-hour glucose tolerance test, which is diagnostic of gestational diabetes. Since this is only a screening test, no treatment for gestational diabetes, such as finger-sticks or insulin, is implemented until the 3-hour glucose tolerance test result determines if the client has gestational diabetes. A hemoglobin A1C level does not rule out diabetes; it monitors average blood glucose level over an extended period of time.

Why is it important for the nurse to thoroughly assess maternal bladder and bowel status during labor? a. A full bladder or rectum can impede fetal descent. b. A full rectum can cause diarrhea. c. If the woman's bladder is distended, it may rupture. d. If the woman has a full bladder, labor may be uncomfortable for her.

a. A full bladder or rectum can impede fetal descent. Throughout labor the nurse needs to assess the woman's fluid balance status as well as check skin turgor and mucous membranes. In addition she needs to monitor the bladder and bowel status. A full bladder or rectum can impede fetal descent.

A G4P3 client with a history of controlled asthma is upset her initial prenatal appointment is taking too long, making her late for another appointment. What is the nurse's best response when the client insists she knows how to handle her asthma and needs to leave? a. Acknowledge her need to leave but ask her to demonstrate the use of inhaler and peak flow meter before she goes; remind her to take regular medications. b. Schedule an appointment for her to return to discuss her asthma management. c. Remind her to continue taking asthma medications, to monitor peak flow daily, and to monitor the baby's kicks in the second and third trimesters. d. Note in the chart that the woman was not counseled about her asthma.

a. Acknowledge her need to leave but ask her to demonstrate the use of inhaler and peak flow meter before she goes; remind her to take regular medications. Management of asthma during pregnancy is very important; the nurse must document that the client has the proper ability to manage her asthma for her health and the health of the fetus. Reminding the client to continue taking her prescribed medication and to monitor her peak flow daily is not enough. It is the nurse's responsibility to know that the client knows how to take her medications. Monitoring the baby's kicks in the second and third trimester is an appropriate action. Scheduling a return appointment to discuss asthma management is not appropriate. She could have an asthma attack between the time the nurse sees her and the time of the return appointment. Noting in the chart that the woman was not counseled does not relieve the nurse of their obligation to ensure that the woman knows how to use her inhaler and her peak flow meter.

During an admission assessment of a client in labor, the nurse observes that there is no vaginal bleeding yet. What nursing intervention is appropriate in the absence of vaginal bleeding when the client is in the early stage of labor? a. Assess amount of cervical dilation. b. Monitor vital signs. c. Obtain urine specimen for urinalysis. d. Monitor hydration status.

a. Assess amount of cervical dilation. If vaginal bleeding is absent during admission assessment, the nurse should perform vaginal examination to assess the amount of cervical dilation. Hydration status is monitored as part of the physical examination. A urine specimen is obtained for urinalysis to obtain a baseline. Vital signs are monitored frequently throughout the maternal assessment.

The nurse is reviewing the medication administration record (MAR) of a client at 39 weeks' gestation and notes that she is ordered an opioid for pain relief. Which is an assessment priority after administering? a. Assess fetal heart rate. b. Assess for constipation. c. Assess for dry mouth. d. Assess maternal blood pressure.

a. Assess fetal heart rate. After administering an opioid to a laboring mother, the priority is to assess the impact on the fetus. Opioid administration can cross the placental barrier with symptoms including assessing heart rate and variability. After birth, there may be a decrease in alertness. Maternal factors of a decreased blood pressure, constipation and dry month are of a lower priority.

A nurse is caring for a client with cardiovascular disease who has just given birth. What nursing interventions should the nurse perform when caring for this client? Select all that apply. a. Assess for shortness of breath. b. Auscultate heart sounds for abnormalities. c. Assess for a moist cough. d. Assess for edema and note any pitting. e. Monitor the client's hemoglobin and hematocrit.

a. Assess for shortness of breath. b. Auscultate heart sounds for abnormalities. c. Assess for a moist cough. d. Assess for edema and note any pitting. The nurse should assess for possible fluid overload in a client with cardiovascular disease who has just given birth. Signs of fluid overload in the client who has just labored include cough, progressive dyspnea, edema, palpitations, and crackles in the lung bases. Hemoglobin and hematocrit levels are not affected by laboring of the client with cardiovascular disease.

A nurse in the maternity triage unit is caring for a client with a suspected ectopic pregnancy. Which nursing intervention should the nurse perform first? a. Assess the client's vital signs. b. Administer oxygen to the client. c. Provide emotional support to the client and significant other. d. Obtain a surgical consent from the client.

a. Assess the client's vital signs. A suspected ectopic pregnancy can put the client at risk for hypovolemic shock. The assessment of vital signs should be performed first, followed by any procedures to maintain the ABCs. Providing emotional support would also occur, as would obtaining a surgical consent, if needed, but these are not first steps.

A nurse is providing care to a couple who have experienced intrauterine fetal demise. Which action would be least effective in assisting a couple at this time? a. Avoid any discussion of the situation with the couple. b. Give the parents a lock of the infant's hair. c. Allow the couple to spend as much time as they want with their stillborn infant. d. Assist the family in making arrangements for their stillborn infant.

a. Avoid any discussion of the situation with the couple. The nurse should encourage discussion of the loss and allow the couple to vent their feelings of grief and guilt. The nurse should allow the parents to spend unlimited time with their stillborn infant so that they can validate the death. Providing the parents and family with mementos of the infant helps validate the reality of the death. Assisting the family with arrangements is helpful to reduce the stress of coping with the situation and making decisions at this difficult time.

A 38-year-old woman comes into the obstetrician's office for prenatal care, stating that she is about 12 weeks pregnant with her first child. What questions would the nurse ask this client, considering her age and potential sensitivity to being labeled an "older" primipara? a. Be non-judgmental in your history gathering and offer her pregnancy resources to read and explore. b. Ask the mother if she has any chronic illnesses that the doctor needs to know about due to her being older. c. Offer genetic counseling and an early amniocentesis to determine if termination is needed. d. Inquire about any family history of chromosomal abnormalities since older women are more likely to have infants with a chromosomal defect.

a. Be non-judgmental in your history gathering and offer her pregnancy resources to read and explore. Women are having babies later in life and nurses must be supportive of their choices to postpone pregnancy. Most women realize the increased risks for having a baby after 35 years of age and don't need constant reminding of all the potentially bad outcomes that can occur. The majority of pregnancies to women over 35 years of age end up with healthy babies and mothers.

The nurse is preparing a birthing care plan for a pregnant client. Which factor should the nurse prioritize to achieve adequate pain relief during the birthing process? a. Client priorities and preferences are incorporated into the plan. b. The health care provider decides the best pain relief for the mother and family. c. The client has the baby without any analgesic or anesthetic. d. The nurse suggests alternative methods of pain relief.

a. Client priorities and preferences are incorporated into the plan. The nurse and the client should work together with the nurse seeking information on the desires of the client and work to achieve the desired level of pain control for the labor and birth experience. The nurse can suggest various options for the client to choose from, but that is not the priority. The health care provider will also work with the client and prescribe medications as necessary and/or desired. The health care professionals should respect the client's wishes for pain control and not insist the client follow any specific format.

The nursing instructor is illustrating the various positions the fetus may utilize during the passage through the vaginal canal at birth. The instructor determines the session is successful when the students correctly identify the ROA position, indicating which presentation by the fetus? a. Facing the right anterior pelvic quadrant b. Presenting with the face as the presenting part c. In a common breech birth position d. In a longitudinal lie facing the left posterior

a. Facing the right anterior pelvic quadrant ROA (right occiput anterior) means the occiput of the fetal head points toward the mother's right anterior pelvis; the head is the presenting part.

The nursing instructor is preparing a class discussing the role of the nurse during the labor and birthing process. Which intervention should the instructor point out has the greatest effect on relieving anxiety for the client? a. Continuous labor support b. Pharmacologic pain management c. Massage therapy d. Prenatal classes

a. Continuous labor support Continuous labor support by a caring nurse or doula can help decrease a woman's anxiety during labor. Anxiety causes the release of catecholamines, which slow down the labor process. The continuous support helps keep the woman focused on what is important as well as provide necessary guidance and education as needed. The massage therapy, prenatal classes, and pharmacologic pain management are all tools that the nurse can use to help the woman.

What special interventions would the nurse implement in a client who is carrying twin fetuses? a. Demonstrate to the client how to perform fetal movement counts after 32 weeks. b. Assist the physician on doing uterine ultrasounds every 2 weeks to monitor fetal size and placental information. c. Remind the client to monitor her intake since she does not need any more food for a multiple pregnancy than she would ingest for a singleton pregnancy. d. Schedule non-stress tests (NST) starting at 16 weeks.

a. Demonstrate to the client how to perform fetal movement counts after 32 weeks. A woman carrying a multiple gestation needs to keep up with how her fetuses are doing, and an excellent way to do that is by doing fetal movement counts, or "kick counts" as they are sometimes called. This starts at around 32 weeks' gestation for an uncomplicated pregnancy and continues until delivery. Weekly or bi-weekly NSTs begin after 32 weeks. Obstetrical ultrasounds are done every 4 to 6 weeks after confirmation of a multiple fetal pregnancy. The client needs to increase her intake, along with her iron and folic acid intake, to provide adequate nutrition for both fetuses.

The nurse is teaching a pregnant woman about how to prevent contracting cytomegalovirus (CMV) during pregnancy. What tips would the nurse share with this client? Select all that apply. a. Do not share food or drinks with young children, especially if they are in daycare. d. If you contract CMV, your doctor will give you some oral medicine to treat it. c. If you have CMV, it is suggested that you not breast-feed your infant. d. If you develop any flu-like symptoms, notify your physician immediately to be evaluated for CMV. d. Wash your hands thoroughly with soap and water after touching saliva or urine.

a. Do not share food or drinks with young children, especially if they are in daycare. d. If you develop any flu-like symptoms, notify your physician immediately to be evaluated for CMV. d. Wash your hands thoroughly with soap and water after touching saliva or urine. Cytomegalovirus (CMV) is a mild infection and women may not know they have contracted it. The problem arises when a pregnant woman contracts it during the first 20 weeks of gestation. Prevention is the key, so the nurse would reinforce handwashing, not eating or drinking from a container after a small child has done so, and notifying the physician if the client develops mild flu-like symptoms so she can be tested to rule out CMV.

A 28-year-old client with a history of endometriosis presents to the emergency department with severe abdominal pain and nausea and vomiting. The client also reports her periods are irregular with the last one being 2 months ago. The nurse prepares to assess for which possible cause for this client's complaints? a. Ectopic pregnancy b. Healthy pregnancy c. Placenta previa d. Molar pregnancy

a. Ectopic pregnancy The most commonly reported symptoms of ectopic pregnancy are pelvic pain and/or vaginal spotting. Other symptoms of early pregnancy, such as breast tenderness, nausea, and vomiting, may also be present. The diagnosis is not always immediately apparent because many women present with complaints of diffuse abdominal pain and minimal to no vaginal bleeding. Steps are taken to diagnose the disorder and rule out other causes of abdominal pain. Given the history of the client and the amount of pain, the possibility of ectopic pregnancy needs to be considered. A healthy pregnancy would not present with severe abdominal pain unless the client were term and she was in labor. With a molar pregnancy the woman typically presents between 8 to 16 weeks' gestation reporting painless (usually) brown to bright red vaginal bleeding. Placenta previa typically presents with painless, bright red bleeding that begins with no warning.

The nurse is monitoring a client who is in the second stage of labor, at +2 station, and anticipating birth within the hour. The client is now reporting the epidural has stopped working and is begging for something for pain. Which action should the nurse prioritize? a. Encourage her through the contractions, explaining why she cannot receive any pain medication. b. Call the anesthetist from the nurse's station to retry the epidural. c. Call the primary care provider, and obtain a reduced dose of meperidine. d. Give the meperidine because she needs pain relief now.

a. Encourage her through the contractions, explaining why she cannot receive any pain medication. At this point, any medication would be contraindicated as it would pass to the fetus and may cause respiratory depression. The nurse will have to work with the mother through the contractions and pushing. The client has progressed too far to retry the epidural medication. No meperidine should be given due to the risk to the fetus.

A client at 35 weeks' gestation is now in stable condition after being admitted for vaginal bleeding. Which assessment should the nurse prioritize? a. Fetal heart tones b. Signs of shock c. Infection d. Uterine stabilization

a. Fetal heart tones When a client is admitted for vaginal bleeding and is stable, the next priority assessment is to determine if the fetus is viable. The other options are not a higher priority than fetal heart tones.

The nurse is educating a client with type 1 diabetes about the complications associated with diabetes and pregnancy. Which problems would the nurse include in her teaching? Select all that apply. a. Increased risk of spontaneous abortion b. Cystic fibrosis c. Hypertension d. Decreased birth weight e. Polyhydramnios

a. Increased risk of spontaneous abortion c. Hypertension e. Polyhydramnios Women with pregestational diabetes, which is type 1 diabetes, are at a higher risk of having an infant with complications during the pregnancy and at delivery. Spontaneous abortion is higher in women who have pregestational diabetes. Also, they run a higher risk of having a pregnancy with polyhydramnios, and of developing maternal hypertension. The birth weight of an infant born to a mother with diabetes is increased, not decreased. Cystic fibrosis is not associated with maternal diabetes.

A 25-year-old client at 22 weeks' gestation is noted to have proteinuria and dependent edema on her routine prenatal visit. Which additional assessment should the nurse prioritize and alert the RN or health care provider? a. Initial BP 100/70 mm Hg; current BP 140/90 mm Hg b. Initial BP 120/80mm Hg; current BP 130/88 mm Hg c. Initial BP 110/60 mm Hg; current BP 112/86 mm Hg d. Initial BP 140/85 mm Hg; current BP 130/80 mm Hg

a. Initial BP 100/70 mm Hg; current BP 140/90 mm Hg A proteinuria of trace to 1+ and a rise in blood pressure to above 140/90 mm Hg is a concern the client may be developing preeclampsia. The blood pressures noted in the other options are not indicative of developing preeclampsia. The edema would not necessarily be indicative of preeclampsia; however, edema of the face and hands would be a concerning sign for severe preeclampsia.

A pregnant client has opted for hydrotherapy for pain management during labor. Which measure should the nurse consider when assisting the client during the birthing process? a. Initiate the technique only when the client is in active labor. b. Ensure that the water temperature exceeds body temperature. c. Allow the client into the water only if her membranes have ruptured. d.Do not allow the client to stay in the bath for long.

a. Initiate the technique only when the client is in active labor. The recommendation for initiating hydrotherapy is that women be in active labor (>5 cm dilated) to prevent the slowing of labor contractions secondary to muscular relaxation. Women are encouraged to stay in the bath or shower as long as they feel they are comfortable. The water temperature should not exceed body temperature. The woman's membranes can be intact or ruptured.

Which nursing intervention offered in labor would probably be the most effective in applying the gate control theory for relief of labor pain? a. Massage the woman's back. b. Give the prescribed medication. c. Encourage the woman to rest between contractions. d. Change the woman's position.

a. Massage the woman's back. Gate-control is based on the idea of distraction or redirection of the conduction of impulses up the neural pathways. Massage redirects the paths of sensation away from the pain to the other area. Encouragement is a form of psychological support. Position change will only distract the client. Medication should be withheld until all nonpharmacological treatments have been exhausted.

The nurse is monitoring a client who has given birth and is now bonding with her infant. Which finding should the nurse prioritize and report immediately for intervention? a. Maternal tachycardia and falling blood pressure b. Dark red lochia c. Placental separation 15 minutes after birth d. The mother is unable to void after 4 hours.

a. Maternal tachycardia and falling blood pressure Maternal tachycardia and falling blood pressure may indicate fluid volume deficit or hemorrhage and require immediate assistance. The client needs further assessment and intervention. The final stages of labor may include uterine contractions, the delivery of the placenta, and the discharge of lochia as the uterus heals. These are normal and not an emergency.

A 25-week-gestation client presents with a blood pressure of 152/99, pulse 78, no edema, and urine negative for protein. What would the nurse do next? a. Notify the health care provider b. Document the client's blood pressure c. Assess the client for ketonuria d. Provide health education

a. Notify the health care provider The client is exhibiting a sign of gestational hypertension, elevated blood pressure greater than or equal to 140/90 mm Hg that develops for the first time during pregnancy. The health care provider should be notified to assess the client. Without the presence of edema or protein in the urine, the client does not have preeclampsia.

A nurse caring for a pregnant client suspected substance use during pregnancy. What is the priority nursing intervention for this client? a. Obtain a urine specimen for a drug screening. b. Determine if the client has emotional support. c. Determine how long the client has been using drugs. d. Provide education material on cessation of substance use.

a. Obtain a urine specimen for a drug screening. Substance use during pregnancy is associated with preterm labor, abortion, low birth weight, central nervous system and fetal anomalies, and long-term childhood developmental consequences. It is most important to know what the client is taking in order to provide the best care for the client and newborn.

The fetus of a woman in labor is determined to be in a persistent occiput posterior position. Which intervention would the nurse prioritize? a. Pain relief measures b. Oxytocin administration c. Side-lying position d. Immediate cesarean birth

a. Pain relief measures Intense back pain is associated with persistent occiput posterior position. Therefore, a priority is to provide pain relief measures. Counterpressure and backrubs may be helpful. Position changes that can promote fetal head rotation are important and can help to relieve some of the pain. Additionally, the woman's ability to cooperate and participate in these position changes is enhanced when she is experiencing less pain. Immediate cesarean birth is not indicated unless there is evidence of fetal distress. Oxytocin would add to the woman's already high level of pain.

The nurse is admitting a client at 23 weeks' gestation in preparation for induction and delivery after it was determined the fetus had died secondary to trauma. When asked by the client to explain what went wrong, the nurse can point out which potential cause for this loss? a. Placental abruption b. Premature rupture of membranes c. Preeclampsia d. Genetic abnormality

a. Placental abruption The most common cause of fetal death after a trauma is placental abruption, where the placenta separates from the uterus, and the fetus is not able to survive. Genetic abnormalities typically cause spontaneous abortion in the first trimester. Trauma does not cause preeclampsia (which is related to various issues in the mother) nor does trauma usually cause PROM.

Which nursing action prevents a complication associated with the lithotomy position for the birth of the fetus? a. Placing a wedge under the hips b. Massaging the client's lower back c. Providing a paper bag d. Rubbing the client's legs

a. Placing a wedge under the hips Due to the lithotomy position, the nursing action of placing a wedge under the hips is correct to avoid supine hypotension. Rubbing the legs or massaging the back can relax the client between intense contractions but those actions do not prevent a complication. Providing a paper bag prevents hyperventilation typically caused by pattern breathing.

The nurse cared for a client who delivered. The duration of labor from the onset of contractions until the birth of the baby was 2 hours. How will the nurse document the client's labor in the health record? a. Precipitous labor b. Prodromal labor c. False Labor d. Prolonged labor

a. Precipitous labor A labor that is less than 3 hours in duration is a precipitous labor. Prolonged labor, also known as failure to progress, occurs when labor lasts for approximately 20 hours or more in a first-time mother. Prodromal labor is labor that starts and stops before fully active labor begins. The contractions are real, but they come and go, and labor does not progress. False labor is intermittent nonproductive or practice contractions, which most commonly occur in the last 2 months before a full-term delivery.

A woman is pregnant and has asthma. Her primary care provider has told her to continue taking prednisone during pregnancy, but she is concerned the drug may be teratogenic. What advice would be best to give her regarding this? a. Prednisone is considered safe in the doses prescribed by her care provider. b. She should half her dose during the first 3 months of pregnancy. c. She should omit the drug during pregnancy. d. Prednisone is a teratogenic drug, but she may need it to control her asthma symptoms.

a. Prednisone is considered safe in the doses prescribed by her care provider. Women should take no medication during pregnancy except that prescribed by their primary care provider. Prednisone may be prescribed safely because, although it may be teratogenic in animal models, it does not appear to be teratogenic in humans.

The nurse is assessing a multipara woman who presents to the hospital after approximately 2 hours of labor and notes the fetus is in a transverse lie. After notifying the RN and primary care provider, which action should the LPN prioritize? a. Prepare to assist with external version. b.. Assist with Nitrazine and fern tests. c. Include a set of piper forceps when the table is prepped. d. Apply pressure to the woman's lower back with a fisted hand.

a. Prepare to assist with external version. Transverse lie is a fetal malposition and is a cause for labor dystocia. The fetus would need to be turned to the occipital position or be born via a cesarean birth. Piper forceps are used in the birth of a fetus that is in the breech position. Nitrazine and fern tests are done to assess if amniotic fluid is leaking from the sac into the vagina. Counterpressure applied to the lower back with a fisted hand sometimes helps the woman to cope with the "back labor" that is characteristic of occiput posterior positioning.

A 17-year-old primigravida with type 1 diabetes is at 37 weeks gestation comes to the clinic for an evaluation. The nurse notes her blood sugar has been poorly controlled and the health care provider is suspecting the fetus has macrosomia. The nurse predicts which step will be completed next? a. Preparing for amniocentesis and fetal lung maturity assessment b. Allowing her to continue without plans for delivery. c. Scheduling the woman for induction of labor today. d. Scheduling a cesarean delivery at 39 weeks.

a. Preparing for amniocentesis and fetal lung maturity assessment If the infant has macrosomia, is large for gestation age, and the mother has had poor blood-sugar control, the provider will want further information on the fetus and readiness for delivery before making any decisions on delivery. After determining the readiness of the fetus, then plans for delivery can be determined and scheduled.

abor dystocia is an abnormal progression of labor. It is the most common cause of primary cesarean birth. When is it most common for labor dystocia to occur? a. Second stage of labor b. First stage of labor c. Third stage of labor d. Fourth stage of labor

a. Second stage of labor Labor dystocia can occur in any stage of labor, although it occurs most commonly once the woman is in active labor or when she reaches the second stage of labor.

The nurse is caring for a woman at 32 weeks gestation with severe preeclampsia. Which assessment finding should the nurse prioritize after the administration of hydralazine to this client? a. Tachycardia b. Gastrointestinal bleeding c. Sweating d. Halos around lights

a. Tachycardia Hydralazine reduces blood pressure but is associated with adverse effects such as palpitation, tachycardia, headache, anorexia, nausea, vomiting, and diarrhea. It does not cause gastrointestinal bleeding, blurred vision (halos around lights), or sweating. Magnesium sulfate may cause sweating.

Which client outcome during active and transitional labor is best? a. The client will practice breathing techniques during contractions. b. The client will tolerate 8 oz (224 g) of clear liquids during labor process. c. The client will walk in the hall for 15 minutes every 2 hours. d. The client will state a pain level of 7 and under during contractions.

a. The client will practice breathing techniques during contractions. The nurse identifies a priority during the active and transitional stage of labor as working with the contractions to give birth. Being tense works against cervical dilation and fetal descent. For that reason, the client is encouraged to practice breathing techniques. It may be unrealistic to state that the pain level is under 7 in the active and transitional phase. Walking in the hall and tolerating liquids also depends on the client.

What assessment finding would suggest to the care team that the pregnant client has completed the first stage of labor? a. The client's cervix is fully dilated. b. The client has contractions once every two minutes. c. The client experiences her first full contraction. d. The infant is born.

a. The client's cervix is fully dilated. The first stage of labor ends with the client's cervix being fully dilated at 10 cm. The onset of contractions signals the beginning of the first stage and birth occurs at the end of the second stage.

A client experiencing contractions presents at a health care facility. Assessment conducted by the nurse reveals that the client has been experiencing Braxton Hicks contractions. The nurse has to educate the client on the usefulness of Braxton Hicks contractions. Which role do Braxton Hicks contractions play in aiding labor? a. These contractions help in softening and ripening the cervix. b. These contractions make maternal breathing easier. c. These contractions increase the release of prostaglandins. d. These contractions increase oxytocin sensitivity.

a. These contractions help in softening and ripening the cervix. Braxton Hicks contractions assist in labor by ripening and softening the cervix and moving the cervix from a posterior position to an anterior position. Prostaglandin levels increase late in pregnancy secondary to elevated estrogen levels; this is not due to the occurrence of Braxton Hicks contractions. Braxton Hicks contractions do not help in bringing about oxytocin sensitivity. Occurrence of lightening, not Braxton Hicks contractions, makes maternal breathing easier.

The nursing instructor is teaching the students the basics of the labor and delivery process. The instructor determines the session is successful when the students correctly choose which action will best help to prevent infections in their clients? a. Thoroughly wash the hands before and after client contact. b. Strictly follow universal precautions. c. Replace soiled drapes and linen as needed. d. Clean the woman's perineum with a Betadine scrub.

a. Thoroughly wash the hands before and after client contact. The most important infection control technique in any health care setting is thoroughly washing hands on a routine basis. Keeping the area clean is secondary, but is also important.

There has been much research done on pain and the perception of pain. What is the result of research done on levels of satisfaction with the control of labor pain? a. Women report higher levels of satisfaction when they felt they had a high degree of control over the pain experience. b. Women report higher levels of satisfaction when the primary care provider makes the decision on what type of pain control to use. c. Women report higher levels of satisfaction when different types of relaxation techniques are used to control pain. d. Women report higher levels of satisfaction when regional anesthetics are used to control pain.

a. Women report higher levels of satisfaction when they felt they had a high degree of control over the pain experience. Research has shown that women report higher levels of satisfaction with their labor experience when they feel a high degree of control over the experience of pain (Stuebe & Barbieri, 2005).

The nurse is preparing the plan of care for a woman hospitalized for hyperemesis gravidarum. Which interventions would the nurse most likely include? Select all that apply. a. administering antiemetic agents b. maintaining NPO status for the first day or two c. obtaining baseline blood electrolyte levels d. monitoring intake and output e. preparing the woman for insertion of a feeding tube

a. administering antiemetic agents b. maintaining NPO status for the first day or two c. obtaining baseline blood electrolyte levels d. monitoring intake and output When hospitalization is necessary, oral food and fluids are withheld to allow the gut to rest. Antiemetic agents are ordered to help control nausea and vomiting. The woman is likely to be dehydrated, so the nurse would obtain baseline blood electrolyte levels and administer intravenous fluid and electrolyte replacement therapy as indicated. Once the nausea and vomiting subside, oral food and fluids are gradually reintroduced. Total parenteral nutrition or a feeding tube is used to prevent malnutrition only if the client does not improve with these interventions.

When teaching a group of soon-to-be parents about the structures of the fetal skull, the nurse describes the anterior fontanelle. Which description would the nurse include? a. approximately 2 to 3 cm in size b. closes 8 to 12 weeks after birth c. located at the back of the fetal head d. triangular shape

a. approximately 2 to 3 cm in size The anterior fontanelle measures about 2 to 3 cm in size, is diamond-shaped, and closes 12 to 18 months after birth. The posterior fontanelle is triangular and located at the back of the fetal head. The posterior fontanelle closes about 8 to 12 weeks after birth.

A pregnant woman is admitted to the hospital with a diagnosis of placenta previa. Which action would be the priority for this woman on admission? a. assessing fetal heart tones by use of an external monitor b. performing a vaginal examination to assess the extent of bleeding c. assessing uterine contractions by an internal pressure gauge d. helping the woman remain ambulatory to reduce bleeding

a. assessing fetal heart tones by use of an external monitor Not disrupting the placenta is a prime responsibility. An internal monitor, a vaginal examination, and remaining ambulatory could all do this and thus are contraindicated.

A nurse is caring for a pregnant client with gestational diabetes. Which meal should the nurse recommend for this client? a. baked turkey, brown rice, and strawberries b. pizza, corn, and orange slices c. baked chicken, green beans, and chocolate cake d. steak, baked potato with butter, and ice cream

a. baked turkey, brown rice, and strawberries The nurse should recommend foods high in whole-grain, lean meats, high fiber, and naturally low in fat and sodium.

A client is experiencing shoulder dystocia during birth. The nurse would place priority on performing which assessment postbirth? a. brachial plexus assessment b. monitor for a cardiac anomaly c. assess for cleft palate d. extensive lacerations

a. brachial plexus assessment The nurse should identify nerve damage as a risk to the fetus in cases of shoulder dystocia. Other fetal risks include asphyxia, clavicle fracture, central nervous system injury or dysfunction, and death. Extensive lacerations is a poor maternal outcome due to the occurrence of shoulder dystocia. Cleft palate and cardiac anomalies are not related to shoulder dystocia.

A 16-year-old client has been in the active phase of labor for 14 hours. An ultrasound reveals that the likely cause of delay in dilatation is cephalopelvic disproportion. Which intervention should the nurse most expect in this case? a. cesarean birth b. administration of morphine sulfate c. administration of oxytocin d. darkening room lights and decreasing noise and stimulation

a. cesarean birth If the cause of the delay in dilatation is fetal malposition or cephalopelvic disproportion (CPD), cesarean birth may be necessary. Oxytocin would be administered to augment labor only if CPD were ruled out. Administration of morphine sulfate (an analgesic) and darkening room lights and decreasing noise and stimulation are used in the management of a prolonged latent phase caused by hypertonic contractions. These measures would not help in the case of CPD.

Which finding would lead the nurse to suspect that the fetus of a woman in labor is in hypertonic uterine dysfuction? a. contractions most forceful in the middle of uterus rather than the fundus b. reports of severe back pain c. fetal buttocks as the presenting part d. lack of cervical dilation past 2 cm

a. contractions most forceful in the middle of uterus rather than the fundus Contractions that are more forceful in the midsection of the uterus rather than in the fundus suggest hypertonic uterine dysfunction. Reports of severe back pain are associated with a persistent occiput posterior position due to the pressure of the fetal head on the woman's sacrum and coccyx. Cervical dilation that has not progressed past 2 cm is associated with dysfunctional labor. A breech position is one in which the fetal presenting part is the buttocks or feet.

A client is giving birth when shoulder dystocia occurs in the fetus. The nurse recognizes that which condition in the client is likely to increase the risk for shoulder dystocia? a. diabetes b. preterm birth c. nullipara d. pendulous abdomen

a. diabetes Shoulder dystocia is most apt to occur in women with diabetes, in multiparas, and in postdate pregnancies. A pendulous abdomen is associated with the transverse lie fetal position not with shoulder dystocia.

A 29-year-old client has gestational diabetes. The nurse is teaching her about managing her glucose levels. Which therapy would be most appropriate for this client? a. diet b. glucagon c. long-acting insulin d. oral hypoglycemic drugs

a. diet Clients with gestational diabetes are usually managed by diet alone to control their glucose intolerance. Long-acting insulin usually is not needed for blood glucose control in the client with gestational diabetes. Oral hypoglycemic drugs are contraindicated in pregnancy. Glucagon raises blood glucose and is used to treat hypoglycemic reactions.

Certain pharmaceuticals can be used to attain cervical ripening in women who need assistance in cervical ripening. They have also often continued into labor without further agents to stimulate uterine contractions. The nurse is aware that the FDA has approved the use of which medication as a cervical ripening agent? a. dinoprostone b. oxytocin c. misoprostol d. magnesium sulfate

a. dinoprostone Dinoprostone is approved by the FDA as the only cervical ripening agent to be used; however, ACOF acknowledges the apparent safety and effectiveness of misoprostol for this purpose as well. It is contraindicated in women with prior uterine scars. It is also known to cause hyperstimulation of the uterus, which can lead to other complications. Magnesium sulfate is used in hygroscopic dilators to assist in a mechanical method of cervical dilation.

When teaching a group of nursing students about the stages of labor, the nurse explains that softening, thinning, and shortening of the cervical canal occur during the first stage of labor. Which term is the nurse referring to in the explanation? a. effacement b. molding c. crowning d. dilatation

a. effacement The nurse is explaining about effacement, which involves softening, thinning, and shortening of the cervical canal. Dilatation refers to widening of the cervical os from a few millimeters in size to approximately 10 cm wide. Crowning refers to a point in the maternal vagina from where the fetal head cannot recede back after the contractions have passed. Molding is a process in which there is overriding and movement of the bones of the cranial vault, so as to adapt to the maternal pelvis.

A woman is lightly stroking her abdomen in rhythm with her breathing during contractions. The nurse identifies this technique as: a. effleurage. b. patterned breathing. c. therapeutic touch. d. acupressure.

a. effleurage. Effleurage is a light, stroking, superficial touch of the abdomen in rhythm with breathing during contractions. Acupressure involves the application of a finger or massage at a trigger point to reduce the pain sensation. Patterned breathing involves controlled breathing techniques to reduce pain through a stimulus-response conditioning. Therapeutic touch involves light or firm touch to the energy field of the body using the hands to redirect the energy fields that lead to pain.

A nurse is conducting a refresher program for a group of perinatal nurses. Part of the program involves a discussion of HELLP. The nurse determines that the group needs additional teaching when they identify which aspect as a part of HELLP? a. elevated lipoproteins b. low platelet count c. hemolysis d. liver enzyme elevation

a. elevated lipoproteins The acronym HELLP represents hemolysis, elevated liver enzymes, and low platelets. This syndrome is a variant of preeclampsia/eclampsia syndrome that occurs in 10% to 20% of clients whose diseases are labeled as severe.

A client in her 20th week of gestation develops HELLP syndrome. What are features of HELLP syndrome? Select all that apply. a. elevated liver enzymes b. hyperthermia c. hemolysis d. leukocytosis e. low platelet count

a. elevated liver enzymes c. hemolysis e. low platelet count The HELLP syndrome is a syndrome involving hemolysis (microangiopathic hemolytic anemia), elevated liver enzymes, and a low platelet count. Hyperthermia and leukocytosis are not features of HELLP syndrome.

A woman is in the fourth stage of labor. During the first hour of this stage, the nurse would assess the woman's fundus at which frequency? a. every 15 minutes b. every 5 minutes c. every 10 minutes d. every 20 minutes

a. every 15 minutes During the first hour of the fourth stage of labor, the nurse would assess the woman's fundus every 15 minutes and then every 30 minutes for the next hour.

A nurse is preparing to teach a class to pregnant women about the signs of preterm labor and what to do if these occur. Which signs of preterm labor should the nurse include in the presentation? Select all that apply. a. feeling of pelvic pressure or fullness b. nausea, vomiting, and diarrhea c. leaking of fluid from the vagina d. feelings of stress e. increase in vaginal discharge f. uterine contractions, cramping, low back pain

a. feeling of pelvic pressure or fullness b. nausea, vomiting, and diarrhea c. leaking of fluid from the vagina e. increase in vaginal discharge f. uterine contractions, cramping, low back pain Signs and symptoms of preterm labor include uterine contractions, cramping, or low back pain; feeling of pelvic pressure or fullness; increased vaginal discharge; nausea, vomiting, and diarrhea; and leaking of fluid from the vagina.

A nurse is conducting a presentation about prenatal care and preexisting maternal conditions. When discussing the various risks to the mother and infant, the nurse would include information about which condition as the leading cause of intellectual disability in the United States? a. fetal alcohol spectrum disorder b. maternal drug addiction c. pregnancy category X medications d. genetic anomalies

a. fetal alcohol spectrum disorder Fetal alcohol spectrum disorder is a lifelong yet completely preventable set of physical, mental, and neurobehavioral birth defects. It is the leading cause of intellectual disability in the United States.

A nurse is monitoring a client with PROM who is in labor and observes meconium in the amniotic fluid. What does the observation of meconium indicate? a. fetal distress related to hypoxia b. cord compression c. central nervous system (CNS) involvement d. infection

a. fetal distress related to hypoxia When meconium is present in the amniotic fluid, it typically indicates fetal distress related to hypoxia. Meconium stains the fluid yellow to greenish brown, depending on the amount present. A decreased amount of amniotic fluid reduces the cushioning effect, thereby making cord compression a possibility. A foul odor of amniotic fluid indicates infection. Meconium in the amniotic fluid does not indicate CNS involvement.

A nurse is providing care to a client who has been diagnosed with a common benign form of gestational trophoblastic disease. The nurse identifies this as: a. hydatidiform mole. b. ectopic pregnancy. c. placenta accrete. d. hydramnios.

a. hydatidiform mole. Gestational trophoblastic disease comprises a spectrum of neoplastic disorders that originate in the placenta. The two most common types are hydatidiform mole (partial or complete) and choriocarcinoma. Ectopic pregnancy, placenta accreta, and hydramnios fall into different categories of potential pregnancy complications.

Which changes in pregnancy would the nurse identify as a contributing factor for arterial thrombosis, especially for the woman with atrial fibrillation? a. hypercoagulable state b. elevation of diaphragm c. increase in blood volume d. increased cardiac output

a. hypercoagulable state The nurse should identify that the increased risk of arterial thrombosis in atrial fibrillation is due to hypercoagulable state of pregnancy. During pregnancy there is a state of hypercoagulation. This increases the risk of arterial thrombosis in clients having atrial fibrillation and artificial valves. Increased cardiac output and blood volume do not cause arterial thrombosis. Elevation of the diaphragm is due to the uterine distension, and it causes a shift in the QRS axis and is not a associated with arterial thrombosis.

A client visits a health care facility reporting amenorrhea for 10 weeks, fatigue, and breast tenderness. Which assessment findings should the nurse prioritize for immediate intervention? Select all that apply. a. hyperemesis gravidarum b. whitish discharge from the vagina c. dyspareunia d. elevated hCG levels e. absence of fetal heart sound

a. hyperemesis gravidarum d. elevated hCG levels e. absence of fetal heart sound This client presents with signs and symptoms suspicious for hydatidiform mole. The signs and symptoms of molar pregnancy include an elevated hCG level, absence of fetal heart sounds, and hyperemesis gravidarum. Whitish discharge from the vagina and dyspareunia (painful sexual intercourse) are seen in cases of infection. In molar pregnancy, a brownish vaginal bleeding is often seen.

A pregnant woman diagnosed with cardiac disease 4 years ago is told that her pregnancy is a high-risk pregnancy. The nurse then explains that the danger occurs primarily because of the increase in circulatory volume. The nurse informs the client that the most dangerous time for her is when? a. in weeks 28 to 32 b. in weeks 20 to 28 c. in weeks 12 to 20 d. in weeks 8 to 12

a. in weeks 28 to 32 The danger of pregnancy in a woman with cardiac disease occurs primarily because of the increase in circulatory volume. The most dangerous time for a woman is in weeks 28 to 32, just after the blood volume peaks.

A nursing instructor is teaching students about fetal presentations during birth. The mostcommon cause for increased incidence of shoulder dystocia is: a. increasing birth weight. b. increased number of overall pregnancies. c. poor quality of prenatal care. d. longer lengths of labor.

a. increasing birth weight. Shoulder dystocia is the obstruction of fetal descent and birth by the axis of the fetal shoulders after the fetal head has emerged. The incidence of shoulder dystocia is increasing because of increasing birth weights, with reports of it in up to 2% of vaginal births.

The nurse is monitoring a client's uterine contractions. Which factors should the nurse assess to monitor uterine contraction? Select all that apply. a. intensity of contractions b. frequency of contractions c. change in blood pressure d. change in temperature e. uterine resting tone

a. intensity of contractions b. frequency of contractions e. uterine resting tone The nurse should assess the frequency of contractions, intensity of contractions, and uterine resting tone to monitor uterine contractions. Monitoring changes in temperature and blood pressure is part of the general physical examination and does not help to monitor uterine contraction.

What would be the physiologic basis for a placenta previa? a. low placental implantation b. a placenta with multiple lobes c. a loose placental implantation d. a uterus with a midseptum

a. low placental implantation The cause of placenta previa is usually unknown, but for some reason the placenta is implanted low instead of high on the uterus.

The nurse is assessing a client in labor for pain and notes she is currently not doing well handling the increased pain. Which opioid can the nurse offer to the client to assist with pain control? a. meperidine b. secobarbital c. thiopental d. hydroxyzine hydrochloride

a. meperidine Meperidine is an opioid that is commonly used during labor and birth. Secobarbital and thiopental are barbiturates. Hydroxyzine hydrochloride is a tranquilizer which can be used to supplement the narcotic or reduce anxiety.

Over the past 20 weeks, the following blood pressure readings are documented for a pregnant client with chronic hypertension: week 16 - 124/86 mm Hg; week 20 - 138/90 mm Hg; week 24 - 140/92 mm Hg; and week 28 - 142/94 mm Hg. The nurse interprets these findings as indicating which classification of her blood pressure? a. mild hypertensive b. severe hypertensive c. prehypertensive d. normotensive

a. mild hypertensive Chronic hypertension exists when the woman has high blood pressure before pregnancy or before the 20th week of gestation, or when hypertension persists for more than 12 weeks. It has been classified as normotensive (systolic less than 120 mm Hg, diastolic less than 80 mm Hg); prehypertension (systolic 120 to 139 mm Hg, diastolic 80 to 89 mm Hg); mild hypertension (systolic 140 to 159 mm Hg, diastolic 90 to 99 mm Hg); and severe hypertension (systolic 160 mm Hg or higher, diastolic 100 mm Hg or higher).

During a routine prenatal visit, a client is found to have proteinuria and a blood pressure rise to 140/90 mm Hg. The nurse recognizes that the client has which condition? a. mild preeclampsia b. eclampsia c. gestational hypertension d. severe preeclampsia

a. mild preeclampsia A woman is said to have gestational hypertension when she develops an elevated blood pressure (140/90 mm Hg) but has no proteinuria or edema. If a seizure from gestational hypertension occurs, a woman has eclampsia, but any status above gestational hypertension and below a point of seizures is preeclampsia. A woman is said to be mildly preeclamptic when she has proteinuria and a blood pressure rise to 140/90 mm Hg, taken on two occasions at least 6 hours apart. A woman has passed from mild to severe preeclampsia when her blood pressure rises to 160 mm Hg systolic and 110 mm Hg diastolic or above on at least two occasions 6 hours apart at bed rest (the position in which blood pressure is lowest) or her diastolic pressure is 30 mm Hg above her prepregnancy level. Marked proteinuria, 3+ or 4+ on a random urine sample or more than 5 g in a 24-hour sample, and extensive edema are also present. A woman has passed into eclampsia when cerebral edema is so acute a grand-mal seizure (tonic-clonic) or coma has occurred.

A client is in the first stage of labor and asks the nurse what type of pain she should expect at this stage. What is the nurse's most appropriate response? a. pain from the dilation or stretching of the cervix b. hypoxia of the contracting uterine muscles c. distention of the vagina and perineum d. pressure on the lower back, buttocks, and thighs

a. pain from the dilation or stretching of the cervix In the first stage of labor, the primary source of pain is the dilation of the cervix. Hypoxia of the contracting uterine muscles, distension of the vagina and perineum, and pressure on the lower back, buttocks, and thighs may occur in the first stage but are more significantly associated with the second stage of labor.

A pregnant woman with preeclampsia is to receive magnesium sulfate IV. Which assessment should the nurse prioritize before administering a new dose? a. patellar reflex b. anxiety level c. blood pressure d. heart rate

a. patellar reflex A symptom of magnesium sulfate toxicity is loss of deep tendon reflexes. Assessing for the patellar reflex or ankle clonus before administration is assurance the drug administration will be safe. Assessing the blood pressure, heart rate, or anxiety level would not reveal a potential magnesium toxicity.

A 40-year-old woman comes to the clinic reporting having missed her period for two months. A pregnancy test is positive. What is she and her fetus at increased risk for? a. placental abnormalities b. type 1 diabetes mellitus c. postterm birth d. type 2 diabetes mellitus

a. placental abnormalities A woman older than 35 years is more likely to conceive a child with chromosomal abnormalities, such as Down syndrome. She is also at higher risk for spontaneous abortion (miscarriage), preeclampsia-eclampsia, gestational diabetes, preterm birth, bleeding and placental abnormalities, and other intrapartum complications.

A 44-year-old client has lost several pregnancies over the last 10 years. For the past 3 months, she has had fatigue, nausea, and vomiting. She visits the clinic and takes a pregnancy test; the results are positive. Physical examination confirms a uterus enlarged to 13 weeks' gestation; fetal heart tones are heard. Ultrasound reveals that the client is experiencing some bleeding. Considering the client's prenatal history and age, what does the nurse recognize as the greatest risk for the client at this time? a. pregnancy loss b. hypertension c. preterm labor d. premature birth

a. pregnancy loss The client's advanced maternal age (pregnancy in a woman 35 years or older) increases her risk for pregnancy loss. Hypertension, preterm labor, and prematurity are risks as this pregnancy continues. Her greatest risk at 13 weeks' gestation is losing this pregnancy.

A woman in week 35 of her pregnancy with severe hydramnios is admitted to the hospital. The nurse recognizes that which concern is greatest regarding this client? a. preterm rupture of membranes followed by preterm birth b. development of eclampsia c. hemorrhaging d. development of gestational trophoblastic disease

a. preterm rupture of membranes followed by preterm birth Even with precautions, in most instances of hydramnios, there will be preterm rupture of the membranes because of excessive pressure, followed by preterm birth. The other answers are not as big of concerns as preterm birth, in this situation.

A 24-year-old client presents in labor. The nurse notes there is an order to administer RhoGAM after the birth of her infant. When asked by the client the reason for this injection, which reason should the nurse point out? a. prevent maternal D antibody formation. b. stimulate maternal D immune antigens. c. prevent fetal Rh blood formation. d. promote maternal D antibody formation.

a. prevent maternal D antibody formation. Because RhoGAM contains passive antibodies, the solution will prevent the woman from forming long-lasting antibodies which may harm a future fetus. The administration of RhoGAM does not promote the formation of maternal D antibodies; it does not stimulate maternal D immune antigens or prevent fetal Rh blood formation.

A nurse is reviewing the FHR and notes it to be in the range of 100 to 106 bpm over the past 10 minutes. Which conditions might the nurse suspect as the cause? Select all that apply. a. prolonged umbilical cord compression b. effect of maternal analgesia c. prematurity d. fetal hypoxia d. maternal fever

a. prolonged umbilical cord compression b. effect of maternal analgesia d. fetal hypoxia Fetal bradycardia occurs when the FHR is below 110 bpm and lasts 10 minutes or longer (Maso, Piccoli, De Seta, et al., 2015). It can be the initial response of a healthy fetus to asphyxia. Causes of fetal bradycardia might include fetal hypoxia, prolonged maternal hypoglycemia, fetal acidosis, administration of analgesic drugs to the mother, hypothermia, anesthetic agents (epidural), maternal hypotension, fetal hypothermia, prolonged umbilical cord compression, and fetal congenital heart block. Maternal fever and prematurity are associated causes of fetal tachycardia.

A woman is experiencing dystocia that appears related to psyche problems. Which intervention would be most appropriate for the nurse initiate? a. providing a comfortable environment with dim lighting b. administering oxytocin c. encouraging the women to change positions frequently d. preparing the woman for an amniotomy

a. providing a comfortable environment with dim lighting Comfort measures minimize the woman's stress and promote relaxation so that she can work more effectively with the forces of labor. This action is consistant with assisting a women experiencing problems with the psyche. Oxytocin would be appropriate for the woman experiencing hypotonic uterine dysfunction (problem with the powers). An amniotomy may be used with hypertonic uterine dysfunction to augment labor. Frequent position changes would be appropriate for a woman with persistent occiput posterior position (problem with the passenger).

When dealing with a pregnant adolescent, the nurse assists the client to integrate the tasks of pregnancy while at the same time fostering development of which trait? a. self-identity b. dependence c. trust d. autonomy

a. self-identity The nurse assists the pregnant adolescent to integrate the tasks of pregnancy, bonding, and preparing to care for another with the tasks of developing self-identity and independence. Trust is a developmental task of infancy. Autonomy is a developmental task of toddlerhood. Independence, not dependence, is fostered.

To assess the frequency of a woman's labor contractions, the nurse would time: a. the beginning of one contraction to the beginning of the next. b. the end of one contraction to the beginning of the next. c. how many contractions occur in 5 minutes. d. the interval between the acme of two consecutive contractions.

a. the beginning of one contraction to the beginning of the next. Measuring from the beginning of one contraction to the next marks the time between contractions.

A multigravid client has been in labor for several hours and is becoming anxious and distressed with the intensity of her frequent contractions. The nurse observes moderate bloody show and performs a vaginal examination to assess the progress of labor. The cervix is 9 cm dilated. The nurse knows that the client is in which phase of labor? a. transition phase b. early phase c. latent phase d. active phase

a. transition phase The woman is likely in transitional labor (first stage, transition phase) as evidenced by her increasing anxiety and distress, intense frequent contractions, and cervical dilation of 9 cm. The amount of bloody show indicates remarkable cervical changes. Cervical dilation in the transition phase is 8 to 10 cm. The latent and early phase is the same phase of labor in the first stage and characterized by positive coping, mild contractions, and cervical dilation of 1 to 4 cm. The active phase is characterized by increased anxiety but cervical dilation of 4 to 7 cm.

Which possible outcome would be a major disadvantage of any pain relief method that also affects awareness of the mother? a. The infant may show increased drowsiness. b. The mother may have difficulty working effectively with contractions. c. The father's coaching role may be disrupted at times. d. The mother may have continued memory loss postpartum.

b. The mother may have difficulty working effectively with contractions. Breathing and relaxation techniques can speed labor. An inability of the woman to do these as a result of pain relief measures can slow labor.

A woman's husband expresses concern about risk of paralysis from an epidural block being given to his wife. Which would be the most appropriate response by the nurse? a. "The injection is given at the third or fourth thoracic vertebrae so paralysis is not a problem." b. "The injection is given in the space outside the spinal cord." c. "An injury is unlikely because of expert professional care given." d. "I have never read or heard of this happening."

b. "The injection is given in the space outside the spinal cord." An epidural block, as the name implies, does not enter the spinal cord but only the epidural space outside the cord.

The nurse encourages a woman with gestational diabetes to maintain an active exercise period during pregnancy. Prior to this exercise period, the nurse would advise her to take which action? a. Inject a bolus of insulin. b. Add a bolus of long-acting insulin. c. Eat a sustaining-carbohydrate snack. d. Eat a high-carbohydrate snack.

c. Eat a sustaining-carbohydrate snack. Because exercise uses up glucose, women with diabetes should take a sustaining-carbohydrate snack before hard exercise to prevent hypoglycemia.

A client is placed on bed rest and home care for preeclampsia. The client's spouse is very concerned about the condition progressing to eclampsia and what do if she experiences a seizure at home. Which is the priority statement by the nurse? a. "Time the seizure from start to finish." b. "To reduce injury, you should move furniture away from her." c. "You should begin CPR breathing." d. "Call 911 immediately."

b. "To reduce injury, you should move furniture away from her." The priority teaching would be for the nurse to instruct the spouse to move furniture away from the client to reduce the risk of injury. The spouse should be instructed to time the seizure, and any lasting over 5 minutes should prompt a call to 911. CPR breathing is never indicated in seizures.

A nurse is performing an assessment on a client in early labor who is discouraged about the seemingly slow progress of her labor. Which response should the nurse prioritize for this client after noting the effacement is progressing even though the cervix is still only 2 cm for the past 2 hours? a. "You haven't dilated any further, but hang in there; it will happen eventually." b. "You are still 2 cm dilated, but the cervix is thinning out nicely." c. Don't mention anything to the client yet; wait for further dilatation to occur. d. "There has been no further dilatation; effacement is progressing."

b. "You are still 2 cm dilated, but the cervix is thinning out nicely." Women are anxious to have frequent reports during labor, to reassure them everything is progressing well. If giving a progress report, remember most women are aware of the word dilatation but not effacement. Just saying, "no further dilatation", therefore, is a depressing report. "You're not dilated a lot more, but a lot of thinning is happening and that's just as important" is the same report given in a positive manner.

A client with asthma is confused by her primary care provider continuing her medication while she is pregnant, since she read online the medications can cause birth defects. What is the nurse's best response? a. "They won't cause any major defects." b. "Your primary care provider will order safe doses of your medication." c. "I'll let your primary care provider know how you feel about it." d. "It's OK to not use them if you would feel more comfortable."

b. "Your primary care provider will order safe doses of your medication." Women should take no medication during pregnancy except that prescribed by their primary care provider. The PCP will work with the mother to ensure the safest amount is given to adequately handle the mother's health issues and not injure the fetus. The PCP must weigh the risks against the benefits for both the mother and her fetus. The nurse should not encourage the client to stop her asthma medication as that may result in the client having an asthma attack, which could result in injury to the fetus or even miscarriage. The nurse should not tell the client a drug will not cause any defects, especially if it is known that it can. That could make the nurse liable for damages. The nurse should inform the PCP of the client's concerns; however, it is more important for the nurse to calm the client's anxiety and offer positive reinforcement that the PCP is working hard to protect the mother and infant from harm.

A nurse has been assigned to assess a pregnant client for abruptio placenta. For which classic manifestation of this condition should the nurse assess? a. generalized vasospasm b. "knife-like" abdominal pain with vaginal bleeding c. increased fetal movement d. painless bright red vaginal bleeding

b. "knife-like" abdominal pain with vaginal bleeding The classic manifestations of abruption placenta are painful dark red vaginal bleeding, "knife-like" abdominal pain, uterine tenderness, contractions, and decreased fetal movement. Painless bright red vaginal bleeding is the clinical manifestation of placenta previa. Generalized vasospasm is the clinical manifestation of preeclampsia and not of abruptio placenta.

The experienced labor and birth nurse knows to evaluate progress in active labor by using which simple rule? a. 1/2 cm/hour for cervical dilation b. 1 cm/hour for cervical dilation c. 2 cm/hour for cervical dilation d. 1/4 cm/hour for cervical dilation

b. 1 cm/hour for cervical dilation In evaluating the progress in active labor, the nurse uses the simple rule of 1 cm/hour for cervical dilation.

A pregnant woman with diabetes is having her glycosylated hemoglobin level evaluated. The nurse determines that the woman's glucose is under control and continues the woman's plan of care based on which result? a. 8.0% b. 6.5% c. 7.5% d. 8.5%

b. 6.5% A glycosylated hemoglobin level of less than 7% indicates good control; a value of more than 8% indicates poor control and warrants intervention. A glycosylated hemoglobin level of more than 8.0% indicates poor blood glucose control and the need for intervention, necessitating a revision in the woman's plan of care.

A primipara at 36 weeks' gestation is being monitored in the prenatal clinic for risk of preeclampsia. Which sign or symptom should the nurse prioritize? a. A systolic blood pressure increase of 10 mm Hg b. A dipstick value of 2+ for protein c. Pedal edema d. Weight gain of 1.2 lb (0.54 kg) during the past 1 week

b. A dipstick value of 2+ for protein The increasing amount of protein in the urine is a concern the preeclampsia may be progressing to severe preeclampsia. The woman needs further assessment by the health care provider. Dependent edema may be seen in a majority of pregnant women and is not an indicator of progression from preeclampsia to eclampsia. Weight gain is no longer considered an indicator for the progression of preeclampsia. A systolic blood pressure increase is not the highest priority concern for the nurse, since there is no indication what the baseline blood pressure was.

A nurse is caring for a client at 38 weeks gestation who is diagnosed with chorioamnionitis. On which intervention should the nurse place priority? a. Assess temperature. b. Administer oxytocin. c. Monitor WBC count. d. Assess amniotic fluid.

b. Administer oxytocin. Chorioamnionitis is an indication for labor induction. The WBC, temperature, and amniotic fluid are not priority to assess because the nurse already knows the client has chorioamnionitis.

At 37 weeks' gestation, a woman presents to labor and delivery complaining of intense, knife-like abdomen pain that started suddenly about 1 hour ago and has not subsided. On palpation, the abdomen is rigid and board-like and no vaginal bleeding is evident. What should the nurse do next? a. Insert a Foley catheter b. Assess fetal heart rate c. Prepare the client for an epidural d. Administer oxygen by face mask

b. Assess fetal heart rate The presence of intense, knife-like abdomen pain with a sudden onset, a rigid and board-like abdomen, and no vaginal bleeding is evidence of a placental abruption. The next action by the nurse is to assess the fetal heart rate to determine the fetus's status. The priority is saving the life of the fetus and the mother. Inserting a urinary catheter and administering oxygen can be done once the status of the fetus is known. This client is not an appropriate candidate for an epidural at this time.

The nursing instructor is conducting a class discussion on the various agents used during labor and delivery to assist the client. The instructor determines the class is successful after the students correctly choose which factor as true about the use of systemic analgesia? a. Opioids are more effective if given PO. b. Benzodiazepines enhance pain relief attained with opioids and cause sedation. c. Barbiturates are used primarily in active labor and during transition. d. Ataractics are used for pain relief but may cause nausea and vomiting.

b. Benzodiazepines enhance pain relief attained with opioids and cause sedation. Barbiturates are used in latent labor for their minor tranquilizng and sedative effects. They can also be used just before general anesthesia, if required. Ataractics are opioid agonists used to decrease anxiety, nausea, and vomiting. Opioids may be given IV, intrathecally, or epidurally.

A nurse is teaching a group of pregnant women about the signs that labor is approaching. When describing these signs, which sign would the nurse explain as being essential for effacement and dilation to occur? a. Bloody show b. Cervical ripening and softening c. Lightening d. Braxton Hicks contractions

b. Cervical ripening and softening The ripening and softening of the cervix which result from the effects of prostaglandins and pressure from Braxton Hicks contractions are essential for effacement and dilation of the cervix. Lightening occurs when the fetal presenting part begins to descend into the true pelvis. Bloody show occurs as the mucous plug is expelled as a result of cervical softening and increased pressure of the presenting part.

A nurse is teaching a group of pregnant women about the signs that labor is approaching. When describing these signs, which sign would the nurse explain as being essential for effacement and dilation to occur? a. Braxton Hicks contractions b. Cervical ripening and softening c. Lightening d. Bloody show

b. Cervical ripening and softening The ripening and softening of the cervix which result from the effects of prostaglandins and pressure from Braxton Hicks contractions are essential for effacement and dilation of the cervix. Lightening occurs when the fetal presenting part begins to descend into the true pelvis. Bloody show occurs as the mucous plug is expelled as a result of cervical softening and increased pressure of the presenting part.

Before calling the primary care provider to report a slow progression or an arrest of labor, several assessments need to be made. What other maternal assessment does the nurse need to make prior to calling the care provider? a. Make sure the client is lying on her left side. b. Check for a full bladder. c. Make sure the epidural medication is turned down. d. Assess vital signs every 30 minutes.

b. Check for a full bladder. A full bladder can interfere with the progress of labor, so the nurse must be sure that the client has emptied her bladder.

A woman arrives in the L & D unit in the beginning early phase with her contractions 5 to 8 minutes apart and dilated 1 cm. Thirty minutes later the nurse finds the woman in hard, active labor and 8 cm dilated. The nurse calls for assistance, prepares for a precipitate birth, and monitors the woman for which priority assessment caused by a rapid birth? a. Assess and administer pain medication as needed. b. Check perineal area frequently for bleeding. c. Assess the woman's breathing and intervene if necessary. d. Assess bladder for fullness.

b. Check perineal area frequently for bleeding. Precipitate dilatation is cervical dilatation that occurs at a rate of 5 cm or more per hour in a primipara or 10 cm or more per hour in a multipara. Contractions can be so forceful they lead to premature separation of the placenta or lacerations of the perineum, placing the woman at risk for hemorrhage. The other interventions are appropriate, but the priority is assessing for bleeding/hemorrhage.

A client has been admitted to the hospital with a diagnosis of severe preeclampsia. Which nursing intervention is the priority? a. Administer oxygen by face mask. b. Confine the client to bed rest in a darkened room. c. Keep the client on her side so that secretions can drain from her mouth. d. Check for vaginal bleeding every 15 minutes.

b. Confine the client to bed rest in a darkened room. With severe preeclampsia, most women are hospitalized so that bed rest can be enforced and a woman can be observed more closely than she can be on home care. The nurse should darken the room if possible because a bright light can also trigger seizures. The other interventions listed pertain to a client who has experienced a seizure and has thus progressed to eclampsia.

A woman with cardiac disease at 32 weeks' gestation reports she has been having spells of light-headedness and dizziness every few days. Which instruction should the nurse prioritize? a. Increase fluids and take more vitamins. b. Decrease activity and rest more often. c. Discuss induction of labor with the primary care provider. d. Bed rest and bathroom privileges only until birth.

b. Decrease activity and rest more often. If the client is developing symptoms associated with her heart condition, the first intervention is to monitor activity levels, decrease activity, and treat the symptoms. At 32 weeks' gestation, the suggestion to induce labor is not appropriate, and without knowledge of the type of heart condition one would not recommend increase of fluids or vitamins. Total bed rest may be required if the symptoms do not resolve with decreased activity.

The nurse assesses that the fetus of a woman is in an occiput posterior position. The nurse predicts the client will experience which situation related to this assessment? a. Necessity for vacuum extraction for birth b. Experience of additional back pain c. Shorter dilatational stage of labor d. Need to have the baby manually rotated

b. Experience of additional back pain Most women whose fetus is in a posterior position experience back pain while in labor. Pressure against the back by a support person often reduces this type of pain. An occiput posterior position does not make for a shorter dilatational stage of labor, it does not indicate the need to have the baby manually rotated, and it does not indicate a necessity for a vacuum extraction birth.

The nurse is assessing a woman who had a forceps-assisted birth for complications. Which condition would the nurse assess in the fetus? a. cervical lacerations b. caput succedaneum c. infection of episiotomy d. perineal hematoma

b. caput succedaneum Caput succedaneum is a complication that may occur in the newborn of a woman who had a forceps- assisted birth. Maternal complications include tissue trauma, such as lacerations of the cervix, vagina, and perineum, hematoma, extension of episiotomy into the anus, hemorrhage, and infection.

A multiparous woman at 39 weeks' gestation arrives at the labor and delivery unit stating that she is in labor. Upon pelvic examination, the nurse documents a softening of the cervix and 3 cm dilation. Which nursing action is best? a. Have the client rest in bed on her left side. b. Have the client ambulate in the hall and recheck. c. Send the client home and return if contractions increase d. Admit the client directly to the labor and delivery area

b. Have the client ambulate in the hall and recheck. To determine if the client is in true labor, the nurse is most correct to have her walk in the hall for approximately an hour. At that point, the client is rechecked to identify if labor has progressed. If labor has progressed, the client is admitted. Having the client rest in bed is not helpful to assist in labor progression.

Which nursing action is essential if the laboring client has the urge to push but she is not fully dilated? a. Have the client lightly push to meet the need. b. Have the client pant and blow through the contraction. c. Have the client divert the energy to squeezing a hand. d. Assist the client to a Fowler's position.

b. Have the client pant and blow through the contraction. The essential nursing action does not allow the client to push. The action is to have the client pant at the beginning of the contraction and then have the client blow through the peak of the contraction. Pushing efforts before the cervix is fully dilated may result in cervical lacerations or cause edema of the cervix, slowing delivery of the fetus. No pushing should be accomplished at this time. It is difficult to divert energy but not push. Assuming a Fowler's position places weight on the perineum.

Many women develop iron-deficient anemia during pregnancy. What diagnostic criteria would the nurse monitor for to determine anemia in the pregnant woman? a. Blood pressure of 100/68 b. Hematocrit of 32% or less c. Hemoglobin of 13 or lower d. Heart rate of 84

b. Hematocrit of 32% or less Iron-deficiency anemia is diagnosed in a pregnant woman if the hematocrit is less that 33% or the hemoglobin is less than 11 g/dL. Tachycardia, hypotension and tachypnea are all symptoms of iron-deficiency anemia but are not diagnostic criteria.

A pregnant client with sickle cell anemia is admitted in crisis. Which nursing intervention should the nurse prioritize? a. antibiotics b. I.V. fluids c. antihypertensive drugs d. diuretic drugs

b. I.V. fluids A sickle cell crisis during pregnancy is usually managed by exchange transfusion, oxygen, and I.V. fluids. Antihypertensive drugs usually aren't necessary. Diuretics would not be used unless fluid overload resulted. The client would be given antibiotics only if there were evidence of an infection.

An infant is born to a mother with gestational diabetes. Which long-term maternal complication is associated with this diagnosis? a. Heart disease b. Increased risk of development of type 2 diabetes c. Weight gain that is not lost after the pregnancy d. Development of long-term hypertension

b. Increased risk of development of type 2 diabetes A mother who had gestational diabetes is at a 30% to 50% higher risk of developing type 2 diabetes mellitus than the general population. Long-term hypertension and heart disease are not associated with gestational diabetes, nor is weight gain following pregnancy. There is no data that validates long-term weight gain as a complication of gestational diabetes.

A pregnant client is admitted to a maternity clinic for birth. The client wishes to adopt the kneeling position during labor. The nurse knows that which to be an advantage of adopting a kneeling position during labor? a. It facilitates external belt adjustment. b. It helps to rotate fetus in a posterior position. c. It helps the woman in labor to save energy. d. It facilitates vaginal examinations.

b. It helps to rotate fetus in a posterior position. The advantage of adopting a kneeling position during labor is that it helps to rotate the fetus in a posterior position. Facilitating vaginal examinations, facilitating external belt adjustment, and helping the woman in labor to save energy are advantages of the back-lying maternal position.

The nurse is preparing to assist with a pudendal block. The nurse predicts the client is at which point in the labor process? a. Just after birth b. Just before birth c. Before dilation only d. Early stage labor

b. Just before birth Pudendal block is a local block in the perineal area and is used to numb for birth. Application before labor begins or while labor is in its early stages would be counterproductive, as the client would not have proper feeling and would have a harder time pushing. After birth it is pointless; the most painful part is over.

The nurse is caring for a client suspected to have a uterine rupture. The nurse predicts the fetal monitor will exhibit which pattern if this is true? a. Variable decelerations b. Late decelerations c. Mild decelerations d. Early decelerations

b. Late decelerations When the fetus is being deprived of oxygen the fetus will demonstrate late decelerations on the fetal monitoring strip. This is an indication the mother is in need of further assessment. Early decelerations are a normal finding. Variable decelerations usually coincide with cord compression.

A nurse is teaching a 42-week nulliparous pregnant woman about labor induction which is being recommended by her health care provider. The nurse determines that the woman needs additional teaching when she identifies which assessment as being done before induction? a. amniotic fluid studies b. Leopold's maneuver c. Bishop scoring d. fetal dating

b. Leopold's maneuver Before labor induction is started, fetal maturity (dating, ultrasound, amniotic fluid studies) and cervical readiness (vaginal examination, Bishop scoring) must be assessed. Both need to be favorable for a successful induction. Leopold's maneuver is a technique for determining the position of the fetus as it moves through the labor process.

A pregnant client is admitted to a maternity clinic for birth. Which assessment finding indicates that the client's fetus is in the transverse lie position? a. Long axis of fetus is at 60° to that of client. b. Long axis of fetus is perpendicular to that of client. c. Long axis of fetus is at 45° to that of client. d. Long axis of fetus is parallel to that of client.

b. Long axis of fetus is perpendicular to that of client. If the long axis of the fetus is perpendicular to that of the mother, then the client's fetus is in the transverse lie position. If the long axis of the fetus is parallel to that of the mother, the client's fetus is in the longitudinal lie position. The long axis of the fetus being at 45° or 60° to that of the client does not indicate any specific position of the fetus.

A woman in labor is having very intense contractions with a resting uterine tone >20 mm Hg. The woman is screaming out every time she has a contraction. What is the highest priority fetal assessment the health care provider should focus on at this time? a. Monitor heart rate for tachycardia. b. Look for late decelerations on monitor, which is associated with fetal anoxia. c. Monitor fetal movements to ensure they are neurologically intact. d. Monitor fetal blood pressure for signs of shock (low BP, high FHR).

b. Look for late decelerations on monitor, which is associated with fetal anoxia. A danger of hypertonic contractions is that the lack of relaxation between contractions may not allow optimal uterine artery filling; this can lead to fetal anoxia early in the latent phase of labor. Applying a uterine and a fetal external monitor will help identify that the resting phase between contractions is adequate and that the FHR is not showing late deceleration.

The nurse is admitting a client in early labor and notes: FHR 120 bpm, blood pressure 126/84 mm Hg, temperature 98.8oF, contractions every 4 to 5 minutes lasting 30 seconds, and greenish-color fluid in the vaginal vault. Which finding should the nurse prioritize? a. Fetal heart rate b. Meconium in the fluid c. Irregular contractions d. Possible maternal infection

b. Meconium in the fluid Green tinted fluid with ROM is indicative of meconium in the amniotic sac, or the infant having a bowel movement in utero. Infection would be shown by pus or cloudy fluid and possibly an elevated temperature. The FHR is within normal range. Irregular contractions is expected at this stage of labor.

A woman at 28 weeks' gestation has been hospitalized with moderate bleeding that is now stabilizing. The nurse notes on routine assessment the client sleeping, lying on her side and EFM showing gradually increasing baseline with late decelerations. Which action should the nurse prioritize? a. Decrease the rate of IV fluids. b. Notify the primary care provider or RN STAT. c. Reposition the mother in a semi-recumbent position on her back. d. Administer oxygen to the mother.

b. Notify the primary care provider or RN STAT. The fetus is showing signs of fetal distress. The immediate treatment is putting the client in a side lying position if not already in it, and alerting the RN and/or health care provider immediately. The health care provider may order oxygen and also increase the rate of IV fluids based on the client's assessment; however, this is not within the scope of the LPN to administer without an order.

A nurse is describing the use of Rho(D) immune globulin as the therapy of choice for isoimmunization in Rh-negative women and for other conditions to a group of nurses working at the women's health clinic. The nurse determines that additional teaching is needed when the group identifies which situation as an indication for Rho(D) immune globulin? a. maternal trauma b. STIs c. amniocentesis d. molar pregnancy

b. STIs Indications for Rho(D) immune globulin include isoimmunization, ectopic pregnancy, chorionic villus sampling, amniocentesis, prenatal hemorrhage, molar pregnancy, maternal trauma, percutaneous umbilical sampling, therapeutic or spontaneous abortion, fetal death, or fetal surgery.

Which order by the health care provider would the nurse question if given in the active stage of labor? a. Out of bed with assistance b. Secobarbital for relaxation c. Positioning with pillows d. Ice chips as requested.

b. Secobarbital for relaxation A sedative, such as secobarital, is given in early labor to promote sleep or a hypertonic contraction pattern. This class of barbiturates can cause respiratory and central nervous system depression if given within 12 to 24 hours of birth. Ice chips may be provided to decrease symptoms of dry mouth. Providing client positioning with the use of pillows for support may allow the client to relax between contractions. For safety reasons, primarily during an intense contraction, the client needs assistance while out of bed.

The nurse is caring for a client in active labor. Which assessment finding should the nurse prioritize and report to the team? a. Bradypnea b. Sudden shortness of breath c. Unrelieved pain d. Bradycardia

b. Sudden shortness of breath Sudden shortness of breath can be a sign of amniotic fluid embolism and requires emergent intervention. This can occur suddenly during, or immediately after, labor. The woman usually develops symptoms of acute respiratory distress, cyanosis, and hypotension. It must be reported to the care team so proper intervention may be accomplished. Other symptoms can include hypotension, cyanosis, hypoxemia, uterine atony, seizures, tachycardia, coagulation failure, DIC, pulmonary edema,

The client may spend the latent phase of the first stage of labor at home unless which occurs? a. The contractions vary in length and intensity b. The client experiences a rupture of membranes c. The client begins back labor d. The client passes the bloody show

b. The client experiences a rupture of membranes Once the client experiences a rupture of membranes, the client is instructed to report to the health care facility. When the rupture of membranes occurs, there is a potential for infection. Also, assessment of the client is required as this is the time of greatest threat of a prolapsed cord. The client may remain at home for all other options.

The nurse is assisting a client in labor and delivery and notes the placenta is now delivered. Which documentation should the nurse prioritize? a. The client's vital signs b. The completion of the third stage of labor c. The transition phase d. The end of recovery

b. The completion of the third stage of labor The third stage of labor ends with the expulsion of the placenta. Transition precedes the second stage and recovery follows later. The client's vital signs should be monitored throughout the entire process but are not the priority with the expulsion of the placenta.

A pregnant client arrives to the clinic for a prenatal visit appearing uncomfortable. During the assessment, the nurse determines the client is experiencing fairly strong contractions at 12:05 p.m., 12:10 p.m., 12:15 p.m., and 12:20 p.m. What can the nurse conclude from these findings? a. The client is in active labor. b. The frequency of the contractions is every 5 minutes. c. The duration of the contractions is every 5 minutes. d. The client can be sent home.

b. The frequency of the contractions is every 5 minutes. Based on the information, the nurse knows the contractions are regular and every 5 minutes apart. This is the only data gathered based on the information given, but it is very useful to the provider. A change in the cervix is necessary for active labor. This client will need further assessment to determine whether the client can go home or should be prepared for active labor. There is no information providing the duration of the contractions.

A woman in active labor suddenly experiences a sharp, excruciating low abdominal pain, which the nurse suspects may be a uterine rupture since the shape of the abdomen has changed. The nurse calls a code, and a cesarean birth is performed stat, but the infant does not survive the trauma. A few hours later, after the woman has stabilized, she asks to hold and touch her infant, and the nurse arranges this. Later, the nurse's documentation should include which outcome statement? a. The parents are exhibiting dysfunctional coping mechanisms related to the death of their newborn. b. The parents are beginning to demonstrate positive grieving behaviors. c. The parents continue to mourn the loss of their infant. d. The parents just cannot believe their perfect infant died.

b. The parents are beginning to demonstrate positive grieving behaviors. An evaluation of proposed outcomes may reveal unhappiness because not every woman who experiences a deviation from the normal in labor and birth will be able to give birth to a healthy child. Some infants will die. Outcome achievement might include the client begins positive grieving behaviors (touching, counting toes/fingers, etc.) in response to the loss of the newborn. The other statements are probably accurate but are not written as outcome statements.

The nurse is assessing a primigravida woman who reports vaginal itching, a great deal of foamy yellow-green discharge, and pain during intercourse. The nurse suspects the woman has contracted which disorder? a. Gonorrhea b. Trichomoniasis c. Simple yeast infection d. Chlamydia

b. Trichomoniasis

A G2P1 woman is in labor attempting a VBAC, when she suddenly complains of light-headedness and dizziness. An increase in pulse and decrease in blood pressure is noted as a change from the vital signs obtained 15 minutes prior. The nurse should investigate further for additional signs or symptoms of which complication? a. Umbilical cord compression b. Uterine rupture c. Hypertonic uterus d. Placenta previa

b. Uterine rupture The client with any prior history of uterus surgery is at increased risk for a uterine rupture. A falling blood pressure and increasing pulse is a sign of hemorrhage, and in this client a uterine rupture needs to be a first consideration. The scenario does not indicate a hypertonic uterus, a placenta previa, or umbilical cord compression.

A woman at 38 weeks' gestation with a history of heroin use disorder has given birth to a newborn several hours ago. Upon assessment, the nurse determines that the newborn is experiencing withdrawal based on which findings? Select all that apply. a. flaccid extremities b. almost constant sneezing c. high-pitched shrill cry d. poor sucking reflex e. nasal stuffiness

b. almost constant sneezing c. high-pitched shrill cry d. poor sucking reflex e. nasal stuffiness The most common harmful effect of heroin and other narcotics on newborns is withdrawal, or neonatal abstinence syndrome. This collection of symptoms may include irritability, hypertonicity, jitteriness, fever, excessive and often high-pitched cry, vomiting, diarrhea, feeding disturbances, respiratory distress, disturbed sleeping, excessive sneezing and yawning, nasal stuffiness, diaphoresis, fever, poor sucking, tremors, and seizures.

The clinic nurse teaches a pregestational type 1 diabetic client that constant insulin levels are very important during pregnancy. The nurse tells the client that the best way to maintain a constant insulin level is to use: a. regular insulin twice a day. b. an insulin pump. c. an insulin pen. d. an insulin drip.

b. an insulin pump. Because a pregnant client will have some periods of relative hyperglycemia and hypoglycemia no matter how carefully she maintains her diet and balances her exercise levels, an effective method to keep serum glucose levels constant is to administer insulin with a continuous pump during pregnancy.

A nurse is caring for a pregnant client with heart disease in a labor unit. Which intervention is most important in the first 48 hours postpartum? a. inspecting the extremities for edema b. assessing for cardiac decompensation c. ensuring that the client consumes a high fiber diet d. limiting sodium intake

b. assessing for cardiac decompensation The nurse should assess the client with heart disease for cardiac decompensation, which is most common from 28 to 32 weeks of gestation and in the first 48 hours postpartum. Limiting sodium intake, inspecting the extremities for edema, and ensuring that the client consumes a high-fiber diet are interventions during pregnancy not in the first 48 hours postpartum.

A nurse is client teaching with a 30-year-old gravida 1 who has sickle cell anemia. Providing education on which topic is the highest nursing priority? a. constipation prevention b. avoidance of infection c. administration of immunoglobulins d. consumption of a low-fat diet

b. avoidance of infection Prevention of crises, if possible, is the focus of treatment for the pregnant woman with sickle cell anemia. Maintaining adequate hydration, avoiding infection, getting adequate rest, and eating a balanced diet are all common-sense strategies that decrease the risk of a crisis. Fat intake does not need to be decreased and immunoglobulins are not normally administered. Constipation is not usually a result of sickle cell anemia.

A nurse working with a woman in preterm labor receives a telephone report for the fetal fibronectin test done 10 hours ago. The report indicates an absence of the protein, which the nurse knows indicates: a. infection is present. b. birth is unlikely within the 2 next weeks. c. birth is likely within the next 2 weeks. d. no infection is present.

b. birth is unlikely within the 2 next weeks. Fetal fibronectin is a protein that helps the placenta and fetal membranes adhere to the uterus during pregnancy. A negative result (absence of fetal fibronectin) is a reliable indicator that birth is unlikely within 2 weeks following the test. It does not diagnose infection.

A woman has been in active labor for over 20 hours, and the nurses are discussing this case with the health care provider. The nurses know to assess this woman for which potential complication from a prolonged labor? Select all that apply. a. exhaustion b. bleeding tendencies to the point of hemorrhage c. exaggerated pain d. maternal postpartal infection e. premature rupture of membranes

b. bleeding tendencies to the point of hemorrhage d. maternal postpartal infection The risk of maternal postpartal infection, hemorrhage, and infant mortality is higher in women who have a prolonged labor than in those who do not. Therefore, it is vital to recognize and prevent dysfunctional labor to the extent possible. In active labor, it is likely the membranes have already ruptured. Exhaustion occurs following labor and is not specific to prolonged labor. All labor results in pain, not just prolonged labor.

A client in labor is agitated and nervous about the birth of her child. The nurse explains to the client that fear and anxiety cause the release of certain compounds which can prolong labor. Which compounds is the nurse referring to in the explanation? a. relaxin b. catecholamines c. oxytocin d. prostaglandins

b. catecholamines Fear and anxiety cause the release of catecholamines, such as norepinephrine and epinephrine, which stimulate the adrenergic receptors of the myometrium. This in turn interferes with effective uterine contractions and results in prolonged labor. Estrogen promotes the release of prostaglandins and oxytocin. Relaxin is a hormone that is involved in producing backache by acting on the pelvic joints. Prostaglandins, oxytocin, and relaxin are not produced due to fear or anxiety in clients during labor.

A pregnant client with a history of heart disease has been admitted to a health care center reporting breathlessness. The client also reports shortness of breath and easy fatigue when doing ordinary activity. The client's condition is markedly compromised. The nurse would document the client's condition using the New York Heart Association (NYHA) classification system as which class? a. class I b. class III c. class IV d. class II

b. class III The nurse should classify the client's condition as belonging to class III of NYHA. In class III of NYHA classification, the client will be symptomatic with ordinary activity, and her condition is markedly compromised. The client is asymptomatic with all kinds of activity and is in uncompromised state in class I. The client is symptomatic with increased activity and is in slight compromised state in class II. The client is symptomatic when resting and is incapacitated in class IV.

Which intervention would be most important when caring for the client with breech presentation confirmed by ultrasound? a. noting the space at the maternal umbilicus b. continuing to monitor maternal and fetal status c. applying suprapubic pressure against the fetal back d. auscultating the fetal heart rate at the level of the umbilicus

b. continuing to monitor maternal and fetal status Once a breech presentation is confirmed by ultrasound, the nurse should continue to monitor the maternal and fetal status when the team makes decisions about the method of birth. The nurse usually plays an important role in communicating information during this time. Applying suprapubic pressure against the fetal back is the nursing intervention for shoulder dystocia and may not be required for breech presentation. Noting the space or dip at the maternal umbilicus and auscultating the fetal heart rate at the umbilicus level are assessments related to occipitoposterior positioning of the fetus.

A nurse is caring for a pregnant client in labor in a health care facility. The nurse knows that which sign marks the termination of the first stage of labor in the client? a. rupturing of fetal membranes b. dilation of cervix diameter to 10 cm c. diffuse abdominal cramping d. start of regular contractions

b. dilation of cervix diameter to 10 cm The first stage of labor terminates with the dilation of the cervix diameter to 10 cm. Diffused abdominal cramping and rupturing of the fetal membrane occurs during the first stage of labor. Regular contractions occur at the beginning of the latent phase of the first stage; they do not mark the end of the first stage of labor.

The nurse is caring for a client who is considered low-risk and in active labor. During the second stage, the nurse would evaluate the client's FHR at which frequency? a. every 10 minutes b. every 15 minutes c. every 5 minutes d. every 20 minutes

b. every 15 minutes It is recommended that the FHR be assessed during the second stage of labor every 15 minutes for the low-risk woman and every 5 minutes for the high-risk woman and during the pushing stage.

A client in week 38 of her pregnancy has an ultrasound performed at a routine office visit and learns that her fetus has not moved out of a breech position. Which intervention does the nurse anticipate for this client? a. vacuum extraction b. external cephalic version c. forceps birth d. trial labor

b. external cephalic version External cephalic version is the turning of a fetus from a breech to a cephalic position before birth. It may be done as early as 34 to 35 weeks, although the usual time is 37 to 38 weeks of pregnancy. A trial birth is performed when a woman has a borderline (just adequate) inlet measurement and the fetal lie and position are good and involves allowing labor to take its normal course as long as descent of the presenting part and dilatation of the cervix continue to occur. Forceps, which are not commonly used anymore, and vacuum extraction are used to facilitate birth when other complications are present, but they would be less likely to be used with a fetus in breech position.

A woman at 10 weeks gestation comes to the clinic for an evaluation. Which assessment finding should the nurse prioritize? a. report of frequent mild nausea b. fundal height measurement of 18 cm c. history of bright red spotting 6 weeks ago d. blood pressure of 120/84 mm Hg

b. fundal height measurement of 18 cm A fundal height of 18 cm is larger than expected and should be further investigated for gestational trophoblastic disease (hydatidiform mole). One of the presenting signs is the uterus being larger than expected for date. Mild nausea would be a normal finding at 10 weeks' gestation. Blood pressure of 120/84 would not be associated with hydatidiform mole and depending on the woman's baseline blood pressure may be within acceptable parameters for her. Bright red spotting might suggest a spontaneous abortion.

A client has been in labor for 10 hours, with contractions occurring consistently about 5 minutes apart. The resting tone of the uterus remains at about 9 mm Hg, and the strength of the contractions averages 21 mm Hg. The nurse recognizes which condition in this client? a. uncoordinated contractions b. hypotonic contractions c. hypertonic contractions d. Braxton Hicks contractions

b. hypotonic contractions With hypotonic uterine contractions, the number of contractions is unusually infrequent (not more than two or three occurring in a 10-minute period). The resting tone of the uterus remains less than 10 mm Hg, and the strength of contractions does not rise above 25 mm Hg. Hypertonic uterine contractions are marked by an increase in resting tone to more than 15 mm Hg. However, the intensity of the contraction may be no stronger than that associated with hypotonic contractions. In contrast to hypotonic contractions, these occur frequently and are most commonly seen in the latent phase of labor. Uncoordinated contractions can occur so closely together they can interfere with the blood supply to the placenta. Because they occur so erratically, such as one on top of another and then a long period without any, it may be difficult for a woman to rest between contractions or to breath effectively with contractions. Braxton Hicks contractions are sporadic contractions that occur in pregnancy before the onset of true labor.

During a prenatal visit a pregnant client asks the nurse how to tell whether the contractions she is having are true contractions or Braxton Hicks contractions. Which description should the nurse mention as characteristic of true contractions? a. begin and remain irregular b. increase in duration, frequency, and intensity c. felt first in lower back and sweep around to the abdomen in a wave d. often disappear with ambulation or sleep e. felt first abdominally and remain confined to the abdomen and groin f. begin irregularly but become regular and predictable

b. increase in duration, frequency, and intensity c. felt first in lower back and sweep around to the abdomen in a wave f. begin irregularly but become regular and predictable True contractions begin irregularly but become regular and predictable; are felt first in the lower back and sweep around to the abdomen in a wave; continue no matter what the woman's level of activity; increase in duration, frequency, and intensity; and achieve cervical dilatation. False (Braxton Hicks) contractions begin and remain irregular; are felt first abdominally and remain confined to the abdomen and groin; often disappear with ambulation or sleep; do not increase in duration, frequency, or intensity; and do not achieve cervical dilatation.

The nurse is reviewing the laboratory test results of a client in labor. Which finding would the nurse consider normal? a. increased blood coagulation time b. increased white blood cell count c. increased blood glucose levels d. decreased plasma fibrinogen levels

b. increased white blood cell count The nurse should identify increased white blood cell count as the hematological change occurring in a client during labor. The increase in the white blood cell count can be attributed to physical and emotional stress during labor. During labor there could be a decrease, and not increase, in the blood coagulation time. There is an increased, not decreased, plasma fibrinogen level during labor. Blood glucose levels are decreased during labor.

The nurse assesses a client in labor and finds that the fetal long axis is longitudinal to the maternal long axis. How should the nurse document this finding? a. attitude b. lie c. position d. presentation

b. lie The nurse is assessing fetal lie, the relationship of the fetal long axis to the maternal long axis. When the fetal long axis is longitudinal to the maternal long axis, the lie is said to be longitudinal. Presentation is the portion of the fetus that overlies the maternal pelvic inlet. Attitude is the relationship of the different fetal parts to one another. Position is the relationship of the fetal denominator to the different sides of the maternal pelvis.

A pregnant client has a history of asthma. After reveiwing the possible medications that may be prescribed during her pregnancy to control her asthma, the nurse determines additional teaching is needed when the client identifies which drug as being used? a. salmeterol b. misoprostol c. budesonide d. albuterol

b. misoprostol Three specific drugs recommended for use for controlling asthma during pregnancy are budesonide, albuterol, and salmeterol. Misoprostol is a prostaglandin that is used for treating postpartum hemorrhage but is contraindicated with asthma clients due to the risk of bronchial spasm and bronchoconstriction.

The five "Ps" of labor are: a. passenger, position, presentation, pushing, psych. b. passageway, passenger, position, powers, psych. c. passenger, posture, position, presentation, psych. d. passenger, position, powers, presentation, psych.

b. passageway, passenger, position, powers, psych. The five "Ps" are passageway (birth canal), passenger (fetus and placenta), position (maternal), powers (contractions), and psych (maternal psychological response).

During a difficult labor of an infant in the face presentation, the nurse notes the infant has a large amount of facial edema with bruising and ecchymosis. Which assessment would be the priority for this infant? a. palpation of the anterior fontanels b. patent airway c. ability to arch the eyebrows d. ability to swallow fluids

b. patent airway Babies born after a face presentation have a great deal of facial edema and may be purple from ecchymotic bruising. The nurse must observe the infant closely for a patent airway. Arching of eyebrows is not a priority. Ability to swallow and palpation of fontanels are routine assessments for all newborns, but they are not the priority.

A client in her fifth month of pregnancy is having a routine clinic visit. The nurse should assess the client for which common second trimester condition? a .mastitis b. physiological anemia c. metabolic alkalosis d. respiratory acidosis

b. physiological anemia Hemoglobin level and hematocrit decrease during pregnancy as the increase in plasma volume exceeds the increase in red blood cell production. Mastitis is an infection in the breast characterized by a swollen tender breast and flu-like symptoms. This condition is most commonly seen in breastfeeding clients. Alterations in acid-base balance during pregnancy result in a state of respiratory alkalosis, compensated by mild metabolic acidosis.

A nurse is educating a group of nursing students about the molding of the fetal skull during the birth process. What would the nurse include as the usual cause of molding? a. tight membranous attachments b. poorly ossified bones of the cranial vault c. well-ossified bones of the face d. rigid bones at the base of the skull

b. poorly ossified bones of the cranial vault Molding is an adaptive process in which there is overriding and movement of the bones of the cranial vault to adapt to the maternal pelvis. The poorly ossified bones of the cranial vault, along with loosely attached membranous attachments, allow for the process of molding in the fetal skull. The bones of the face and the base of the skull are completely ossified and united. Hence they cannot allow for movement or overriding. The membranous attachments are loosely bound to the cranial vault and not tightly, which allows for molding of the fetal skull.

After conducting a review class on the labor and birth process for a group of nurses working in the community clinic, the nurse determines that the teaching was successful when the group identifies which factors as affecting the labor process? Select all that apply. a. place b. powers c. passenger d. patience e. participation

b. powers c. passenger d. patience There are "five Ps" that affect the labor process. They are passageway, passenger, powers, position, and psychological response. There are an additional five that can also have an effect on the labor process. They include philosophy, partners, patience, client preparation, and pain control.

The nurse preceptor explains that several factors are involved with the "powers" that can cause dystocia. She focuses on the dysfunction that occurs when the uterus contracts so frequently and with such intensity that a very rapid birth will take place. This is known as which term? a. hypertonic contractions b. precipitous labor c. hypotonic contractions d. none of the above

b. precipitous labor When the expulsive forces of the uterus become dysfunctional, the uterus may either never fully relax (hypertonic contractions) placing the fetus in jeopardy, or relax too much (hypotonic contractions), causing ineffective contractions. Another dysfunction can occur when the uterus contracts so frequently and with such intensity that a very rapid birth will take place (precipitous labor).

A maternity nurse is aware that the fetal head is the presenting part in complete extension position. Which type of birth should the maternity nurse anticipate? a. a forceps-assisted vaginal birth b. prolonged labor and possible cesarean birth c. a normal labor and a spontaneous vaginal birth d. precipitous labor and birth

b. prolonged labor and possible cesarean birth The attitude of the fetal head is moderate flexion. If there are changes in the fetal attitude (the head), the presenting part is then a larger diameter to the maternal pelvis. This presentation could cause a long labor and possible cesarean birth.

A woman has presented to the emergency department with symptoms that suggest an ectopic pregnancy. Which finding would lead the nurse to suspect that the fallopian tube has ruptured? a. nausea b. referred shoulder pain c. breast tenderness d. vaginal spotting

b. referred shoulder pain Referred pain to the shoulder area indicates bleeding into the abdomen caused by phrenic nerve irritation when a tubal pregnancy ruptures. Vaginal spotting, nausea, and breast tenderness are typical findings of early pregnancy and an unruptured ectopic pregnancy.

A nurse is explaining to a pregnant client about the changes occurring in the body in preparation for labor. Which hormone would the nurse include in the explanation as being responsible for causing the pelvic connective tissue to become more relaxed and elastic? a. progesterone b. relaxin c. oxytocin d. prolactin

b. relaxin As the pregnancy progresses, the hormones relaxin and estrogen cause the connective tissues to become more relaxed and elastic and cause the joints to become more flexible to prepare the mother's pelvis for birth.

During the assessment of a laboring client, the nurse learns that the client has cardiovascular disease (CVD). Which assessment would be priority for the newborn? a. urine output b. respiratory function c. temperature d. heart rate

b. respiratory function The nurse should identify respiratory distress syndrome as a major risk that can be faced by the offspring of a client with cardiovascular disease. While the other assessments are important, they are not priority.

A pregnant client is admitted to a maternity clinic after experiencing contractions. The assigned nurse observes that the client experiences pauses between contractions. The nurse knows that which event marks the importance of the pauses between contractions during labor? a. effacement and dilation of the cervix b. restoration of blood flow to uterus and placenta c. reduction in length of the cervical canal d. shortening of the upper uterine segment

b. restoration of blood flow to uterus and placenta The pauses between contractions during labor are important because they allow the restoration of blood flow to the uterus and the placenta. Shortening of the upper uterine segment, reduction in length of the cervical canal, and effacement and dilation of the cervix are other processes that occur during uterine contractions.

A multipara woman is experiencing a prolonged descent while trying to rest and increase her fluid intake. The nurse suggests that she change position. Which position is most effective for pushing to speed up the descent? Select all that apply. a. standing, leaning against a door facing b. semi-Fowler's position c. lithotomy position d. move into a squatting position e. supine with knees pulled up to chest

b. semi-Fowler's position A semi-Fowler's position, or a squatting, kneeling position are more effective pushing positions and may speed descent. The other positions will not help speed the descent.

When assessing a pregnant woman with vaginal bleeding, the nurse would suspect a threatened abortion based on which finding? a. passage of fetal tissue b. slight vaginal bleeding c. strong abdominal cramping d. cervical dilation

b. slight vaginal bleeding Slight vaginal bleeding early in pregnancy, no cervical dilation, and a closed cervical os are associated with a threatened abortion. Strong abdominal cramping is associated with an inevitable abortion. With an inevitable abortion, passage of the products of conception may occur. No fetal tissue is passed with a threatened abortion.

What is a nursing intervention that helps prevent the most frequent side effect from epidural anesthesia in a pregnant client? a. administrating IV naloxone b. starting an IV and hanging IV fluids c. administrating IV ephedrine d. maintaining the client in a supine position

b. starting an IV and hanging IV fluids Prehydration with IV fluids helps to prevent the most common side effect of epidural anesthesia, which is hypotension (20%). If the client develops hypotension or respiratory depression, then IV ephedrine or IV naloxone, respectively, can be administered, but neither is preventative. Maintaining the client in a supine position is recommended for a spinal headache, which can be a side effect of epidural anesthesia but is not the most common side effect and is not preventative.

What is the role of the nurse during the preconception counseling of a pregnant client with chronic hypertension? a. stressing regular walks and exercise b. stressing the positive benefits of a healthy lifestyle c. stressing the increased use of Vitamin D supplements d. stressing the avoidance of dairy products

b. stressing the positive benefits of a healthy lifestyle The nurse should stress the positive benefits of a healthy lifestyle during the preconception counseling of a client with chronic hypertension. The client need not avoid dairy products or increase intake of vitamin D supplements. It may not be advisable for a client with chronic hypertension to exercise without consultation.

A woman at 26 weeks' gestation is undergoing screening for diabetes with a 1-hour oral glucose challenge test. On the client's return visit, the nurse anticipates the need to schedule a 3-hour glucose challenge test based on which result of the previous test? a. 114 mg/dL b. 130 mg/dL c. 146 mg/dL d. 100 mg/dL

c. 146 mg/dL For a 1-hour glucose challenge test, a 75-g oral glucose load is given, without regard to the timing or content of the last meal. Blood glucose is measured 1 hour later; a level above 140 mg/dL is abnormal. If the result is abnormal, a 3-hour glucose tolerance test is done.

A woman in week 16 of her pregnancy calls her primary care provider's office to report that she has experienced abdominal cramping, cervical dilation, vaginal spotting, and the passing of tissue. The nurse instructs the client to bring the passed tissue to the hospital with her. What is the correct rationale for this instruction? a. to determine the stage of development of the fetus b. to determine whether gestational trophoblastic disease is present c. to determine whether infection is present d. to determine whether the fetus is viable

b. to determine whether gestational trophoblastic disease is present Gestational trophoblastic disease is abnormal proliferation and then degeneration of the trophoblastic villi. The embryo fails to develop beyond a primitive start. Abnormal trophoblast cells must be identified because they are associated with choriocarcinoma, a rapidly metastasizing malignancy. This is why it is important for any woman who begins to miscarry at home to bring any clots or tissue passed to the hospital with her. The presence of clear fluid-filled cysts changes the diagnosis from a simple miscarriage to gestational trophoblastic disease. The client is not instructed to bring in passed tissue to determine whether infection is present or the fetus is viable or to determine the stage of development of the fetus.

A young woman experiencing contractions arrives at the emergency department. After examining her, the nurse learns that the client is at 33 weeks' gestation. What treatment can the nurse expect this client to be prescribed? a. muscle relaxants b. tocolytic therapy c. anti-anxiety therapy d. bronchodilators

b. tocolytic therapy Tocolytic therapy is most likely prescribed if preterm labor occurs before the 34th week of gestation in an attempt to delay birth and thereby reduce the severity of respiratory distress syndrome and other complications associated with prematurity.

A client with a pendulous abdomen and uterine fibroid tumors had just begun labor and arrived at the hospital. After examining the client, the primary care provider informs the nurse that the fetus appears to be malpositioned in the uterus. Which fetal position or presentation should the nurse most expect in this woman? a. cephalic presentation b. transverse lie c. anterior fetal position d. occipitoposterior position

b. transverse lie A transverse lie, in which the fetus is more horizontal than vertical, occurs in women with pendulous abdomens, with uterine fibroid tumors that obstruct the lower uterine segment, with contraction of the pelvic brim, with congenital abnormalities of the uterus, or with hydramnios. Anterior fetal position and cephalic presentation are normal conditions. Occipitoposterior position tends to occur in women with android, anthropoid, or contracted pelves.

A nursing instructor highlights which risk factors associated with preterm labor? Select all that apply. a. weight of pregnant mother b. uterine or cervical abnormalities c. weight of fetus d. current multiple gestation pregnancy e. history of previous preterm birth

b. uterine or cervical abnormalities d. current multiple gestation pregnancy e. history of previous preterm birth The top risk factors for preterm labor include history of previous preterm birth, current multiple gestation pregnancy, and uterine or cervical abnormalities. The weight of the fetus or mother does not cause preterm labor.

A pregnant client has tested positive for hepatitis B virus. When discussing the situation with the client, the nurse explains that her infant should be vaccinated with an initial HBV vaccine dose at which time? a. within 24 hours of birth b. within 12 hours of birth c. within 36 hours of birth d. within 48 hours of birth

b. within 12 hours of birth If a woman tests positive for HBV, the newborn will receive HBV vaccine within 12 hours of birth. The second dose will be given at 1 month and the third dose at 6 months.

The health care provider has determined that the source of dystocia for a woman is related to the fetus size. The nurse understands that macrosomia would indicate the fetus would weigh: a. 4,500 g or more. b. 3,000 g or more. c. 4,000 g or more. d. 3,500 g or more.

c. 4,000 g or more. Macrosomia occurs when the fetus measures 4,000 g (8.13 lbs) or more at birth and complicates approximately 10% of all pregnancies. The excessive fetal size and abnormalities contribute to labor and birth dysfunctions.

The nurse tests the pH of fluid found on the vaginal exam and determines that the woman's membranes have ruptured based on which result? a. 5.5 b. 5.0 c. 6.5 d. 6.0

c. 6.5 Amniotic fluid is alkaline, so the membranes are probably ruptured if the pH ranges from 6.5 to 7.5.

A 35-year-old client is seen for her 2-week postoperative appointment after a suction curettage was performed to evacuate a hydatidiform mole. The nurse explains that the human chorionic gonadotropin (hCG) levels will be reviewed every 2 weeks and teaches about the need for reliable contraception for the next 6 months to a year. The client states, "I'm 35 already. Why do I have to wait that long to get pregnant again?" What is the nurse's best response? a. "Since you are at the end of your reproductive years, it is suggested that you don't try to have any more pregnancies." b. "After a curettage procedure, it is recommended that you give your body some time to build up its stores." c. "A contraceptive is used so that a positive pregnancy test resulting from a new pregnancy will not be confused with the increased level of hCG that occurs with a developing malignancy." d. "You may need chemotherapy, so we don't want to risk pregnancy."

c. "A contraceptive is used so that a positive pregnancy test resulting from a new pregnancy will not be confused with the increased level of hCG that occurs with a developing malignancy." Because of the risk of choriocarcinoma, the woman receives extensive treatment. Therapy includes baseline chest X-ray to detect lung metastasis, plus a physical exam (including a pelvic exam). Serum B-hCG levels weekly until negative results are obtained three consecutive times, then monthly for 6 to 12 months. The woman is cautioned to avoid pregnancy during this time because the increasing B-hCG levels associated with pregnancy would cause confusion as to whether cancer had developed. If after a year B-hCG seruim titers are within normal levels, a normal pregnancy can be achieved.

A pregnant client is diagnosed with syphilis. Which interviewing question would demonstrate respect for the client and therapeutic communication? a. "I noticed that you seem fidgety. Is there something wrong besides your STI?" b. "You should have thought about what diseases you could be exposed to. At least you are HIV negative." c. "I am sure it is frightening to you to be diagnosed with a disease that can affect your baby." d. "Why didn't you use protection when having intercourse with your partner?":

c. "I am sure it is frightening to you to be diagnosed with a disease that can affect your baby." The nurse needs to be supportive, empathic and accepting of the client, asking open-ended questions and acting calm and reassuring to her. By acknowledging her fears for her fetus, the nurse is demonstrating respect for her and conveying confidence that the client is trying to take care of her fetus.

A client at 11 weeks' gestation experiences pregnancy loss. The client asks the nurse if the bleeding and cramping that occurred during the miscarriage were caused by working long hours in a stressful environment. What is the most appropriate response from the nurse? a. "Something was wrong with the fetus." b. "Your spontaneous bleeding is not work-related." c. "I can understand your need to find an answer to what caused this. Let's talk about this further." d. "It is hard to know why a woman bleeds during early pregnancy."

c. "I can understand your need to find an answer to what caused this. Let's talk about this further." Talking with the client may assist her to explore her feelings. She and her family may search for a cause for a spontaneous early bleeding so they can plan for future pregnancies. Even with modern technology and medical advances, however, a direct cause cannot usually be determined.

A client is scheduled for a cesarean section under spinal anesthesia. After instruction is given by the anesthesiologist, the nurse determines the client has understood the instructions when the client states: a. "I will need to lie on my right side to reduce vena cava compression." b. "The anesthesia will numb both of my legs to a level above my breasts." c. "I may end up with a severe headache from the spinal anesthesia." d. "I can continue sitting up after the spinal is given."

c. "I may end up with a severe headache from the spinal anesthesia." Cerebrospinal fluid (CSF) leakage from the needle insertion site and irritation caused by a small amount of air that enters at the injection site and shifts the pressure of the CSF causes strain on the cerebral meninges, initiating headache pain.

The nurse is teaching a client who is diagnosed with preeclampsia how to monitor her condition. The nurse determines the client needs more instruction after making which statement? a. "I will count my baby's movements after each meal." b. "I will weigh myself every morning after voiding before breakfast." c. "If I have changes in my vision, I will lie down and rest." d. "If I have a severe headache, I'll call the clinic."

c. "If I have changes in my vision, I will lie down and rest." Changes in the visual field may indicate the client has moved from preeclampsia to severe preeclampisa and is at risk for developing a seizure due to changes in cerebral blood flow. The client would require immediate assessment and intervention. Gaining weight is not necessarily a sign of worsening preeclampsia. The other choices are instructions which the client may be given to follow.

The nurse is caring for a client in the transition phase of the labor process. Which client statement requires nursing action? a. "My mouth and lips are so dry." b. "I feel burning in my perineum." c. "My lips and fingers are tingling." d. "My contractions are really intense now."

c. "My lips and fingers are tingling." When the client reports that her lips and fingers are tinging, the nurse is correct to understand that she is hyperventilating. To correct hyperventilation, the nurse instructs the client to slow the breathing. A paper bag or cupped hands is the correct nursing action. All of the other statements are normal for the client in the transition phase of labor. The nurse would moisten the client's lips or provide a lip balm for dry mouth or lips.

A pregnant woman with diabetes is having a glycosylated hemoglobin (HbA1C) level drawn. Which result would require the nurse to revise the client's plan of care? a. 5.5% b. 6.0% c. 8.5% d. 7%

c. 8.5% An HbA1C level of more than 8% indicates poor control and the need for intervention, necessitating a revision in the woman's plan of care.

The nurse is teaching a prenatal class on potential problems during pregnancy to a group of expectant parents. The risk factors for placental abruption are discussed. What comment validates accurate learning by the parents? a. "Since I am over 30, I run a much higher risk of developing this problem." b. "If I develop this complication, I will have bright red vaginal bleeding," c. "Placental abruption is quite painful and I will need to let the doctor know if I begin to have abdominal pain." d. "I need a cesarean section if I develop this problem."

c. "Placental abruption is quite painful and I will need to let the doctor know if I begin to have abdominal pain." Placental abruption occurs when there is a spontaneous separation of the placenta from the uterine wall. It can occur anywhere on the placenta and will cause painful, dark red vaginal bleeding. If the abruption is small, the physician will try to deliver the fetus vaginally. But if severe bleeding occurs or there is fetal distress, a C-section is done. Women older than 35 are also at higher risk for developing placental abruption.

A pregnant women calls the clinic to report a small amount of painless vaginal bleeding. What response by the nurse is best? a. "Bleeding during pregnancy happens for many reasons, some serious and some harmless." b. "Lie on your left side and drink lots of water and monitor the bleeding." c. "Please come in now for an evaluation by your health care provider." d. "If the bleeding lasts more than 24 hours, call us for an appointment."

c. "Please come in now for an evaluation by your health care provider." Bleeding during pregnancy is always a deviation from normal and should be evaluated carefully. It may be life-threatening or it may be something that is not a threat to the mother and/or fetus. Regardless, it needs evaluated quickly and carefully. Telling the client it may be harmless or serious is a reassuring statement, but does not advice for urgent evaluation. Having the mother lay on her left side and drink water is indicated for cramping.

The nurse determines a client is 7 cm dilated. What is the best response when asked by the client's partner how long will she be in labor? a. "She is doing well and is in the second stage; it could be anytime now." b. "She is still in early latent labor and has much too long to go to tell when she will give birth." c. "She is in active labor; she is progressing at this point and we will keep you posted." d. "She is in the transition phase of labor, and it will be within 2 to 3 hours, though it might be sooner."

c. "She is in active labor; she is progressing at this point and we will keep you posted." At 7 cm dilated, she is considered in the active phase of labor. There is no science that can predict the length of labor. She is progressing in labor, and it is best not to give the family a specific time frame.

A primigravidia client at 38 weeks' gestation calls the clinic and reports, "My baby is lower and it is more difficult to walk." Which response should the nurse prioritize? a. "The baby moved down into the pelvis; this means you will be in labor within 24 hours, so wait for contractions then come to the hospital." b. "This is not normal unless you are in active labor; come to the hospital and be checked." c. "The baby has dropped into the pelvis; your body and baby are getting ready for labor in the next few weeks." d. "That is something we expect with a second or third baby, but because it is your first, you need to be checked."

c. "The baby has dropped into the pelvis; your body and baby are getting ready for labor in the next few weeks." The baby can drop into the pelvis, an event termed lightening, and can happen for up to 2 weeks before the woman goes into labor. This is normal and does not require intervention.

The nurse is caring for a client who is a primagravida. Which statement is best to improve the client's psyche? a. "You will be finished soon." b. "Your second pregnancy will be easier." c. "You are doing a great job" d. "You will be pushing very soon."

c. "You are doing a great job" When a mother feels confident in her ability to cope and finds ways to work with the contractions, the labor process is enhanced. Stating that the client will be finished soon or the second pregnancy will be easier does not provide the client with confidence. Stating that the client will be pushing soon may provide the client with anxiety as she has never pushed before.

A fetus is assessed at 2 cm above the ischial spines. How would the nurse document the fetal station? a. +2 b. +4 c. -2 d. 0

c. -2 When the presenting part is above the ischial spines, it is noted as a negative station. Since the measurement is 2 cm, the station would be -2. A fetus at 0 (zero) station indicates that the fetal presenting part is at the level of the ischial spines. Positive stations indicate that the presenting part is below the level of the ischial spines.

During labor, progressive fetal descent occurs. Place the stations listed in their proper sequence from first to last. All options must be used. a. +4 station b. 0 station c. -4 station d. -2 station e. +2 station

c. -4 station d. -2 station b. 0 station e. +2 station a. +4 station Progressive fetal descent (-5 to +4) is the expected norm during labor, moving downward from the negative stations to zero station to the positive stations in a timely manner.

The nurse is providing care to a client in labor. On examination, the nurse determines the fetus is at -1 station. The nurse interprets this as indicating that the fetus is: a. 1 cm below the pubic bone. b. 1 cm below the ischial spines. c. 1 cm above the ischial spines. d. 1 cm above the pubic bone.

c. 1 cm above the ischial spines. Station refers to the relationship of the presenting part to the level of the maternal pelvic ischial spines. Fetal station is measured in centimeters and referred to as a minus or plus, depending on its location above or below the ischial spines. When the presenting part is above the ischial spines, the distance is recorded as minus stations. When the presenting part is below the ischial spine, it is recorded as plus stations. Therefore this fetus is 1 cm above the ischial spines.

A G3, P2 woman arrives at the birthing center stating that she has been in labor for the past 18 hours. To rule out a potential protracted disorder the nurse is aware that which criterion for cervical dilation would be used? a. 1.0 cm per hour b. 1.75 cm per hour c. 1.5 cm per hour d. 1.25 cm per hour

c. 1.5 cm per hour Protracted disorders refer to a series of events including protracted active phase dilation and protracted descent. A laboring woman who has a slower than normal rate of cervical dilation is said to have a protraction labor pattern disorder. In terms of time, it describes a labor lasting more than 18 to 24 hours. Diagnostic criteria are 1.2 cm per hour for primiparous and 1.5 cm per hour for multiparous.

A postpartum mother has the following lab data recorded: a negative rubella titer. What is the appropriate nursing intervention? a. Assess the rubella titer of the baby. b. Notify the health care provider. c. Administer rubella vaccine before discharge. d. No action needed.

c. Administer rubella vaccine before discharge. Rubella is a virus, which when contracted during pregnancy has significant complications for the fetus. The illness is mild to the adult but can result in the infant being born deaf and blind. There is no cure; the CDC recommends all individuals be vaccinated against rubella. If the titer is negative, the mother does not have protection against rubella, and the next pregnancy would be at risk. She should receive the vaccination prior to discharge from the hospital. Assessing the rubella titer of the baby would not mean anything. The baby has not had rubella and has not received antibodies against rubella from the mother. Notifying the health care provider is not a priority, as most institutions have standing orders to administer the rubella vaccine if the mother's rubella titer is negative.

A primigravida client at 39 weeks' gestation calls the OB unit questioning the nurse about being in labor. Which response should the nurse prioritize? a. Emphasize that food and fluid should stop or be light. b. Arrange for the woman to come to the hospital for labor evaluation. c. Ask the woman to describe why she believes that she is in labor. d. Tell the woman to stay home until her membranes rupture.

c. Ask the woman to describe why she believes that she is in labor. The nurse needs further information to assist in determining if the woman is in true or false labor. The nurse will need to ask the client questions to seek further assessment and triage information. Having the client wait until membranes rupture may be dangerous, as the client may give birth before reaching the hospital. The client should continue fluid intake until it is determined whether or not the client is in labor. The client may be in false labor, and more information should be obtained before the client is brought to the hospital.

As a woman enters the second stage of labor, her membranes spontaneously rupture. When this occurs, what would the nurse do next? a. Test a sample of amniotic fluid for protein. b. Elevate her hips to prevent cord prolapse. c. Assess fetal heart rate for fetal safety. d. Ask her to bear down with the next contraction.

c. Assess fetal heart rate for fetal safety. Rupture of the membranes may lead to a prolapsed cord. Assessment of FHR detects this.

When caring for a client with premature rupture of membranes (PROM), the nurse observes an increase in the client's pulse. What should the nurse do next? a. Assess for cord compression. b. Monitor the client for preterm labor. c. Assess the client's temperature. d. Monitor the fetus for respiratory distress.

c. Assess the client's temperature. A temperature elevation or an increase in the pulse of a client with PROM would indicate infection. Increase in the pulse does not indicate preterm labor or cord compression. The nurse should monitor FHR patterns continuously, reporting any variable decelerations suggesting cord compression. Respiratory distress syndrome is one of the perinatal risks associated with PROM.

Which postoperative intervention should a nurse perform when caring for a client who has undergone a cesarean birth? a. Delay breastfeeding the newborn for a day. b. Ensure that the client does not cough or breathe deeply. c. Assess uterine tone to determine fundal firmness. d. Avoid early ambulation to prevent respiratory problems.

c. Assess uterine tone to determine fundal firmness. When caring for a client who has undergone a cesarean birth, the nurse should assess the client's uterine tone to determine fundal firmness. The nurse should assist with breastfeeding initiation and offer continued support. The nurse can also suggest alternate positioning techniques to reduce incisional discomfort while breastfeeding. Delaying breastfeeding may not be required. The nurse should encourage the client to cough, perform deep-breathing exercises, and use the incentive spirometer every 2 hours. The nurse should assist the client with early ambulation to prevent respiratory and cardiovascular problems.

The nurse is assessing a 35-year-old woman at 22 weeks' gestation who has had recent laboratory work. The nurse notes fasting blood glucose 146 mg/dL (8.10 mmol/L), hemoglobin 13 g/dL (130 g/L), and hematorcrit 37% (0.37). Based on these results which instruction should the nurse prioritize? a. Take daily iron supplements. b. The signs and symptoms of urinary tract infection. c. Check blood sugar levels daily. d. Include iron-enriched foods in the diet.

c. Check blood sugar levels daily. An elevated blood glucose is concerning for diabetes. A fasting blood glucose level of greater than 140 mg/dL (7.77 mmol/L) or random level of greater than 200 mg/dL (11.10 mmol/L) is concerning; this must be followed up to ensure the client is not developing gestational diabetes. The hemoglobin and hematocrit are within normal limits for this client. The values should be hemoglobin greater than 11 g/dL (110 g/L) and hematocrit greater than 33% (0.33). Values lower than that are possible indications of anemia and would necessitate further evaluation. An individual with higher than normal blood glucose levels is at risk for developing urinary tract infeciton. This will usually happen after the glucose levels are elevated. Anemia can be treated by increasing the consumption of iron-enriched foods and taking a daily iron supplement.

A gravida 1 client is admitted in the active phase of stage 1 labor with the fetus in the LOA position. The nurse anticipates noting which finding when the membranes rupture? a. Bloody fluid b. Greenish fluid c. Clear to straw-colored fluid d. Cloudy white fluid

c. Clear to straw-colored fluid The infant is in the correct position, and the client has been in labor. Expectation would be for normal amniotic fluid presentation of clear to straw-colored fluid. If there is blood, then the uterus is bleeding and there is an extreme emergency. If the fluid is greenish, there is meconium in the fluid. Cloudy, white fluid may indicate an infection is present.

A client in preterm labor is receiving magnesium sulfate IV and appears to be responding well. Which finding on assessment should the nurse prioritize? a. Bradycardia b. Elevated blood glucose c. Depressed deep tendon reflexes d. Tachypnea

c. Depressed deep tendon reflexes The nurse should assess the woman at least once hourly and report any dyspnea (not tachypnea), tachycardia (not bradycardia), productive cough, adventitious breath sounds, and absent or decreased deep tendon reflexes in a client receiving magnesium sulfate; these are all signs of possible magnesium toxicity. Elevated blood glucose is a potential adverse reaction if the woman is receiving terbutaline.

Which consideration is a priority when caring for a mother with strong contractions 1 minute apart? a. The station in which the fetus is located b. Maternal heart rate and blood pressure c. Fetal heart rate in relation to contractions d. Maternal request for pain medication

c. Fetal heart rate in relation to contractions The priority consideration is on the status of the fetus. Because each contraction temporarily interrupts blood flow to the placenta, there is a decrease in oxygen available. Therefore, a fetus cannot tolerate contractions lasting too long or too strong. All other options are important but not the priority.

A client tells that nurse in the doctor's office that her friend developed high blood pressure on her last pregnancy. She is concerned that she will have the same problem. What is the standard of care for preeclampsia? a. Monitor the client for headaches or swelling on the body. b. Take one aspirin every day. c. Have her blood pressure checked at every prenatal visit. d. Take low-dose antihypertensive prophylactically.

c. Have her blood pressure checked at every prenatal visit. Preeclampsia and eclampsia are common problems for pregnant clients and require regular blood pressure monitoring at all prenatal visits. Antihypertensives are not prescribed unless the client is already hypertensive. Monitoring for headaches and swelling is a good predictor of a problem but doesn't address prevention— nor does it predict who will have hypertension. Taking aspirin has shown to reduce the risk in women who have moderate to high risk factors, but has shown no effect on those women with low risk factors.

Which assessment findings indicate a distressed fetus? Select all that apply. a. Fetal heart rate baseline of 140 b. Moderate fetal heart rate variability c. Late deceleration patterns d. Persistent bradycardia e. Moderate fetal heart rate variability

c. Late deceleration patterns d. Persistent bradycardia e. Moderate fetal heart rate variability The nurse evaluates the fetal monitor for reassuring patterns and/or signs of fetal distress. Absent accelerations, late deceleration patterns and persistent bradycardia indicate client hypoxia. A fetal heart rate baseline of 140 and moderate variability is a reassuring sign.

A woman with class II heart disease is experiencing an uneventful pregnancy and is now prescribed bed rest at 36 weeks' gestation by her health care provider. The nurse should point out that this is best accomplished with which position? a. Lie flat on her back. b. Stay in high Fowler's position. c. Lie in a semi-recumbent position. d. Use pillows and wedges to stay in a fully recumbent position.

c. Lie in a semi-recumbent position. Semi-recumbent position is the best position for circulation of the mother and fetus. Lying flat on the back can induce supine hypotensive syndrome and fully recumbent impedes other circulation. The high Fowler's position would not be comfortable for sleeping, as well as possibly impede the blood flow through the hips and lower abdomen.

What important instruction should the nurse give a pregnant client with tuberculosis? a. Avoid direct sunlight. b. Wear light, cotton clothes. c. Maintain adequate hydration. d. Avoid red meat.

c. Maintain adequate hydration. The nurse should instruct the pregnant client with tuberculosis to maintain adequate hydration as a health-promoting activity. The client need not avoid direct sunlight or red meat, or wear light clothes; these have no impact on the client's condition.

General anesthesia is not used frequently in obstetrics because of the risks involved. There are physiologic changes that occur during pregnancy that make the risks of general anesthesia higher than it is in the general population. What is one of those risks? a. The client is less sensitive to inhalation anesthetics. b. Fetal hypersensitivity to anesthetic is possible. c. Neonatal depression is possible. d. The client is more sensitive to preanesthetic medications.

c. Neonatal depression is possible. General anesthesia is not used frequently in obstetrics because of the risks involved. The pregnant woman is at higher risk for aspiration. It requires more skill to intubate a pregnant woman because of physiologic changes in the trachea and thorax. In addition, general anesthetic agents cross the placenta and can result in the birth of a severely depressed neonate who requires full resuscitation.

A 26-year-old primigravida has brought her doula to the birthing center for support during her labor and birth. The doula has been helping her through the past 16 hours of labor. The laboring woman is now 6 cm dilated. She continues to report severe pain in her back with each contraction. The client finds it comforting when her doula uses the ball of her hand to put counterpressure on her lower back. What is the likely cause of the woman's back pain? a. Fetal macrosomia b. Breech presentation c. Occiput posterior position d. Nongynecoid pelvis

c. Occiput posterior position A labor complicated by occiput posterior position is usually prolonged and characterized by maternal perception of increased intensity of back discomfort. The lay term for this type of labor is "back labor."

A woman at 34 weeks' gestation presents to labor and delivery with vaginal bleeding. Which finding from the obstetric examination would lead to a diagnosis of placental abruption? a. Fetus is in a breech position b. Sonogram shows the placenta covering the cervical os c. Onset of vaginal bleeding was sudden and painful d. Uterus is soft between contractions

c. Onset of vaginal bleeding was sudden and painful Sudden onset of abdominal pain and vaginal bleeding with a rigid uterus that does not relax are signs of a placental abruption. The other findings are consistent with a diagnosis of placenta previa.

The nurse is caring for a woman undergoing cervical dilation. Which assessment finding would alert the nurse to the complication of vasa previa? a. Rapid cervical dilation and effacement b. Failure to dilate as labor and effacement progresses c. Painless bleeding at the beginning of cervical dilation d. Meconium stained amniotic fluid noted

c. Painless bleeding at the beginning of cervical dilation Painless bleeding at the beginning of cervical dilation (due to vessel tearing) is a sign of vasa previa. Rapid cervical dilation and effacement are indicative of precipitous birth. Meconium staining may be noted with a breech birth.

A 32-year-old gravida 3 para 2 at 36 weeks' gestation comes to the obstetric department reporting abdominal pain. Her blood pressure is 164/90 mm/Hg, her pulse is 100 beats per minute, and her respirations are 24 per minute. She is restless and slightly diaphoretic with a small amount of dark red vaginal bleeding. What assessment should the nurse make next? a. Measure fundal height. b. Obtain a voided urine specimen, and determine blood type. c. Palpate the fundus, and check fetal heart rate. d. Check deep tendon reflexes.

c. Palpate the fundus, and check fetal heart rate. The classic signs of abruptio placentae are pain, dark red vaginal bleeding, a rigid, board-like abdomen, hypertonic labor, and fetal distress.

A client is admitted to the health care facility. The fetus has a gestational age of 42 weeks and is suspected to have cephalopelvic disproportion. Which should the nurse do next? a. Perform artificial rupture of membranes. b. Administer oxytocin intravenously at 4 mU/minute. c. Prepare the client for a cesarean birth. d. Place the client in lithotomy position for birth.

c. Prepare the client for a cesarean birth. Cephalopelvic disproportion is associated with postterm pregnancy. This client will not be able to vaginally give birth and should be prepared for a cesarean birth. Lithotomy position, artificial rupture of membranes, and oxytocin are interventions for a vaginal birth.

The maternal health nurse is caring for a group of high-risk pregnant clients. Which client condition will the nurse identify as being the highest risk for pregnancy? a. Secondary hypertension b. Repaired atrial septal defect c. Pulmonary hypertension d. Loud systolic murmur

c. Pulmonary hypertension Pulmonary hypertension is considered the greatest risk to a pregnancy because of the hypoxia that is associated with the condition. The remaining conditions represent potential cardiac complications that may increase the client's risk in pregnancy; however, these do not present the greatest risk in pregnancy.

A pregnant woman comes to the emergency department stating she thinks she is in labor. Which assessment finding concerning the pain will the nurse interpret as confirmation that this client is in true labor? a. Lasts about 20 to 25 seconds b. Slows when the woman changes position c. Radiates from the back to the front d. Occurs in an irregular pattern

c. Radiates from the back to the front Contractions that begin in the back and then radiate to the front are typical of true labor. Contractions that slow when a woman walks or changes position suggest false labor, as do irregular contractions. Contractions lasting 30 seconds or less commonly suggest Braxton Hicks contractions and are associated with false labor.

The nurse has been asked to present information to a group of civic leaders concerning women's health issues. In preparing the information, the nurse includes what goal from Healthy People 2020 related to women in labor? a. Ensure all couples receive preconceptional genetic counseling. b. Encourage women with previous cesareans to always have a cesarean. c. Reduce the rate of cesarean births among low-risk births. d. Ensure care during labor includes immunizations.

c. Reduce the rate of cesarean births among low-risk births. Healthy People 2020 has two goals related to cesarean births in the United States. They are to reduce the rate of cesarean births among low-risk women and reduce the rate of cesarean births among women who have had a prior cesarean birth. Immunizations and genetic counseling are not associated with women in labor.

The following hourly assessments are obtained by the nurse on a client with preeclampsia receiving magnesium sulfate: 97.3oF (36.2oC), HR 88, RR 1, BP 148/110 mm Hg. What other priority physical assessments by the nurse should be implemented to assess for potential toxicity? a. Lung sounds b. Magnesium sulfate level c. Reflexes d. Oxygen saturation

c. Reflexes Reflex assessment is part of the standard assessment for clients on magnesium sulfate. The first change when developing magnesium toxicity may be a decrease in reflex activity. The health care provider needs to be notified immediately. A change in lung sounds and oxygen saturation are not indicative of magnesium sulfate toxicity. Hourly blood draws to gain information on the magnesium sulfate level are not indicated.

A nurse is providing education to a woman at 28 weeks' gestation who has tested positive for gestational diabetes mellitus (GDM). What would be important for the nurse to include in the client teaching? a. Her baby is at increased risk for type 1 diabetes mellitus. b. She is at increased risk for type 1 diabetes mellitus after her baby is born. c. She is at increased risk for type 2 diabetes mellitus after her baby is born. d. Her baby is at increased risk for neonatal diabetes mellitus.

c. She is at increased risk for type 2 diabetes mellitus after her baby is born. The woman who develops GDM is at increased risk for developing type 2 diabetes mellitus after pregnancy.

A 15-year-old adolescent arrives at the office with a report of flu symptoms, including nausea and vomiting and recent weight loss. A pregnancy test is done and is positive. The client begins crying and tells the nurse her mother will be furious with her. What can the nurse do to assist this adolescent at this point? a. Recommend some adoption agencies for her to talk to in the near future. b. Contact the mother of the adolescent to be sure the child gets prenatal care. c. Support her by respecting her right to privacy and confidentiality. d. Tell the adolescent that this is too big of a problem for her to make decisions about and she needs to listen to her mother.

c. Support her by respecting her right to privacy and confidentiality. The nurse needs to be an advocate for the adolescent and respect her privacy and confidentiality. It would be advisable for the nurse to encourage the adolescent to talk to her mother or some other support person for help. The nurse has no right to contact the adolescent's mother or to share any information with her. Also, the nurse should not mention adoption at this point to the adolescent. That would be a topic for later discussion.

A client and her husband have prepared for a natural birth; however, as the client progresses to 8 cm dilation, she can no longer endure the pain and begs the nurse for an epidural. What is the nurse's best response? a. Suggest a less extreme alternative such as a sedative. b. Gently remind the client of her goal of a natural birth and encourage and help her. c. Support the client's decision and call the obstetrician. d. Ask the husband to gently remind her of their goal of natural birth and to encourage and help her.

c. Support the client's decision and call the obstetrician. Pain is subjective and its level is only what the client experiences. The nurse should support the desire of the client. Sedatives would be counterproductive as they may slow the labor process. It would be inappropriate to negate her feelings and remind her of earlier goals; that is the job of the support person and should be left up him or her to decide what to say and when to say it.

A client is entering her 42nd week of gestation and is being prepared for induction of labor. The nurse recognizes that the fetus is at risk for which condition? a. dystocia b. infection c. macrosomia d. hemorrhage

c. macrosomia Fetal risks associated with a prolonged pregnancy include macrosomia, shoulder dystocia, brachial plexus injuries, low Apgar scores, postmaturity syndrome, cephalopelvic disproportion, uteroplacental insufficiency, meconium aspiration, and intrauterine infection. Amniotic fluid volume begins to decline by 40 weeks of gestation, possibly leading to oligohydramnios. Hemorrhage, infection, and dystocia are risk to the mother not the fetus.

The nurse is assisting with a G2P1, 24-year-old client who has experienced an uneventful pregnancy and is now progressing well through labor. Which action should be prioritized after noting the fetal head has retracted into the vagina after emerging? a. Use Zavanelli maneuver. b. Apply pressure to the fundus. c. Use McRoberts maneuver. d. Attempt to push in one of the fetus's shoulders.

c. Use McRoberts maneuver. McRoberts maneuver intervention is used with a large baby who may have shoulder dystocia and requires assistance. The legs are sharply flexed by a support person or nurse, and the movement will help to open the pelvis to the widest diameter possible. Zavanelli maneuver is performed when the practitioner pushes the fetal head back in the birth canal and performs an emergency cesarean birth. Fundal pressure is contraindicated with shoulder dystocia. It is out of the province of the LVN to attempt birth of the fetus by pushing one of the fetus' shoulders in a clockwise or counterclockwise motion.

The nurse is caring for a client experiencing a prolonged second stage of labor. The nurse would place priority on preparing the client for which intervention? a. a cesarean birth b. a precipitous birth c. a forceps and vacuum-assisted birth d. artificial rupture of membranes

c. a forceps and vacuum-assisted birth A forceps-and-vacuum-assisted birth is required for the client having a prolonged second stage of labor. The client may require a cesarean birth if the fetus cannot be delivered with assistance. A precipitous birth occurs when the entire labor and birth process occurs very quickly. Artificial rupture of membranes is done during the first stage of labor.

A woman with cardiac disease has come to the office for prenatal counseling. According to the functional classification system developed by the Criteria Committee of the New York Heart Association, the woman would be cautioned against pregnancy based on which category? a. class III b. class I c. class IV d. class II

c. class IV A woman with class IV disease should avoid pregnancy. These individuals can be symptomatic at rest or with any physical activity resulting in inability to carry on any physical activity without discomfort. Symptoms or heart failure or the anginal syndrome may be present even at rest. The pregnancy may put too much stress on the woman's body increasing her risk of serious complications or even death.

A pregnant woman comes to the birthing center, stating she is in labor and does not know far along her pregnancy is because she has not had prenatal care. A primary care provider performs an ultrasound that indicates oligohydramnios. When the client's membranes rupture, meconium is in the amniotic fluid. What does the nurse suspect may be occurring with this client? a. placental abruption b. complications of preterm labor c. complications of a postterm pregnancy d. complications of placenta previa

c. complications of a postterm pregnancy A postterm pregnancy carries risks for increased perinatal mortality, particularly during labor. Oligohydramnios and meconium staining of the amniotic fluid are common complications. Oligohydramnios increases the incidence of cord compression, which can lead to fetal distress during labor. Thick, meconium-stained fluid increases the risk for meconium aspiration syndrome.

The nurse is assessing a pregnant client with a known history of congestive heart failure who is in her third trimester. Which assessment findings should the nurse prioritize? a. shortness of breath, bradycardia, and hypertension b. regular heart rate and hypertension c. dyspnea, crackles, and irregular weak pulse d. increased urinary output, tachycardia, and dry cough

c. dyspnea, crackles, and irregular weak pulse The nurse should be alert for signs of cardiac decompensation due to congestive heart failure which include crackles in the lungs from fluid, difficulty breathing, and weak pulse from heart exhaustion. The heart rate would not be regular, and a cough would not be dry. The heart rate would increase rather than decrease.

When teaching a group of nursing students about the stages of labor, the nurse explains that softening, thinning, and shortening of the cervical canal occur during the first stage of labor. Which term is the nurse referring to in the explanation? a. dilatation b. molding c. effacement d. crowning

c. effacement The nurse is explaining about effacement, which involves softening, thinning, and shortening of the cervical canal. Dilatation refers to widening of the cervical os from a few millimeters in size to approximately 10 cm wide. Crowning refers to a point in the maternal vagina from where the fetal head cannot recede back after the contractions have passed. Molding is a process in which there is overriding and movement of the bones of the cranial vault, so as to adapt to the maternal pelvis.

A pregnant woman is being evaluated for HELLP. The nurse reviews the client's diagnostic test results. Which result would the nurse interpret as helping to confirm this diagnosis? a. elevated hematocrit b. elevated platelet count c. elevated LDH d. elevated white blood cells

c. elevated LDH HELLP (hemolysis, elevated liver enzymes, low platelet count) syndrome is a variant of the preeclampsia/eclampsia syndrome. The diagnosis is based on laboratory test results, including: low hematocrit, elevated LDH, elevated AST, elevated ALT, elevated BUN, elevated bilirubin level, elevated uric acid and creatinine levels, and low platelet count. White blood cell counts are not used to evaluate for HELLP.

A client in the first stage of labor is admitted to a health care center. The nurse caring for the client instructs her to rock on a birth ball. The nurse informs her that this causes the release of certain natural substances, which reduces the pain. To which substance is the nurse referring? a. prostaglandins b. progesterone c. endorphins d. relaxin

c. endorphins The nurse is referring to the release of endorphins, which are natural analgesic substances released by the movement of the client on the birth ball. The nurse should encourage the client to rock or sit on the birth ball. This causes the release of endorphins. The client's movement on the birth ball does not produce prostaglandins, progesterone, or relaxin. Prostaglandins are local hormones that bring about smooth muscle contractions in the uterus. Progesterone is a hormone involved in maintaining pregnancy. Relaxin is a hormone that causes backache during pregnancy by acting on the pelvic joints.

Which medication would the nurse prepare to administer if prescribed as treatment for an unruptured ectopic pregnancy? a. promethazine b. ondansetron c. methotrexate d. oxytocin

c. methotrexate Methotrexate, a folic acid antagonist that inhibits cell division in the developing embryo, is most commonly used to treat ectopic pregnancy. Oxytocin is used to stimulate uterine contractions and would be inappropriate for use with an ectopic pregnancy. Promethazine and ondansetron are antiemetics that may be used to treat hyperemesis gravidarum.

A nurse performs an initial assessment of a laboring woman and reports the following findings to the primary care provider: fetal heart rate is 152 bpm, cervix is 100% effaced and 5 cm dilated, membranes are intact, and presenting part is well applied to the cervix and at -1 station. The nurse recognizes that the client is in which stage of labor? a. second b. first, latent c. first, active d. third

c. first, active Because the cervix is dilating (5 cm) and has fully effaced (100%), the woman appears to be in active labor, which is characterized by cervical dilation of 4 to 7 cm. Regular uterine contractions are effective in facilitating fetal descent through the pelvis because the presenting part is well applied on the cervix and at -1 station. Second stage of labor begins when the cervix is 10 cm dilated. The first latent phase is characterized by the onset of regular contractions and cervical dilation of 0 to 4 cm. Third stage of labor is from birth of the infant to completed delivery of the placenta.

The nurse is determining how often contractions occur measuring from the beginning of the one contraction to the beginning of the next contraction. The nurse documents this finding as: a. peak. b. intensity. c. frequency. d. duration.

c. frequency. Frequency refers to how often the contractions occur and is measured from the beginning of one contraction to the beginning of the next contraction. Duration refers to how long a contraction lasts and is measured from the beginning of one contraction to the end of that same contraction. Intensity refers to the strength of the contraction determined by manual palpation or measured by an internal intrauterine pressure catheter. The peak or acme of a contraction is the highest intensity of a contraction.

A client has come to the office for a prenatal visit during her 22nd week of gestation. On examination, it is noted that her blood pressure has increased to 138/90 mm Hg. Her urine is negative for proteinuria. The nurse recognizes which factor as the potential cause? a. preeclampsia b. chronic hypertension c. gestational hypertension d. HELLP

c. gestational hypertension Gestational hypertension is characterized by hypertension without proteinuria after 20 weeks of gestation resolving by 12 weeks postpartum. It is defined as systolic blood pressure of greater than 140 mm Hg and/or diastolic of greater than 90 mm Hg on at least two occasions at least 6 hours apart after the 20th week of gestation, in women known to be normotensive prior to this time and prior to pregnancy. HELLP is an acronym that refers to hemolysis, elevated liver enzymes, and low platelets. Preeclampsia may result if hypertension is not controlled or advances to a more severe state.

A pregnant client has been admitted with reports of brownish vaginal bleeding. On examination there is an elevated hCG level, absent fetal heart sounds, and a discrepancy between the uterine size and the gestational age. The nurse interprets these findings to suggest which condition? a. placenta previa b. abruption of placenta c. gestational trophoblastic disease e. ectopic pregnancy

c. gestational trophoblastic disease The client is most likely experiencing gestational trophoblastic disease or a molar pregnancy. In gestational trophoblastic disease or molar pregnancy, there is an abnormal proliferation and eventual degeneration of the trophoblastic villi. The signs and symptoms of molar pregnancy include brownish vaginal bleeding, elevated hCG levels, discrepancy between the uterine size and the gestational age, and absent fetal heart sounds. Abruption of placenta is characterized by premature separation of the placenta. Ectopic pregnancy is a condition where there is implantation of the blastocyst outside the uterus. In placenta previa the placental attachment is at the lower uterine segment.

A nurse is taking a history of a client at 5 weeks' gestation in the prenatal clinic; however, the client is reporting dark brown vaginal discharge, nausea, and vomiting. Which diagnosis should the nurse suspect? a. pregnancy-induced depression b. placenta previa c. gestational trophoblastic disease d. hyperemesis gravidarum

c. gestational trophoblastic disease This client has risk factors of a "molar" pregnancy: nausea and vomiting at an early gestational week and dark brown vaginal discharge. The early nausea/vomiting can be due to a high HCG level, which is a sign of gestational trophoblastic disease. There is only one sign/symptom of hyperemesis gravidarum. Placenta previa is marked by bright red bleeding and tends to happen later in gestation. There are no data to support any psychosis at this stage.

Cytomegalovirus infection can result in different congenital anomalies. It can also be transmitted via different routes. When discussing this infection with a pregnant woman, the nurse integrates understanding that permanent fetal disability can occur with which type of transmission of CMV? a. after birth transmission b. with any transmission c. in utero transmission d. during birth transmission

c. in utero transmission There are three time periods during which mother-to-child transmission can occur; however, permanent disability occurs only in association with in utero infection. Such disability can result from maternal infection during any point in the pregnancy, but more severe disabilities are usually associated with maternal infection during the first trimester.

A nurse sees a pregnant client at the clinic. The client is close to her due date. During the visit the nurse would emphasize that the client get evaluated quickly should her membranes rupture spontaneously based on the understanding of which possibility? a. increased risk of breech presentation b. potential rapid birth of fetus c. increased risk of infection d. potential placenta previa

c. increased risk of infection After the amniotic sac has ruptured, the barrier to infection is gone, and an ascending infection is possible. In addition, there is a danger of cord prolapse. The spontaneous rupture does not hasten labor, although it might signal the beginning of labor. The client may have placenta previa with the membranes intact.

A woman in labor who is receiving an opioid for pain relief is to receive promethazine. The nurse determines that this drug is effective when the woman demonstrates which finding? a. increased feelings of control b. decreased sedation c. less anxiety d. increased cervical dilation

c. less anxiety Promethazine is used in combination with an opioid to decrease nausea and vomiting and lessen anxiety. It may also be used to increase sedation. It does not affect the progress of labor. Benzodiazepines are used to calm a woman who is out of control, allowing her to relax enough to participate effectively during labor.

A pregnant client has tested positive for cytomegalovirus. What can this cause in the newborn? a. bicuspid valve stenosis b. hypertension c. microcephaly d. clubbed fingers and toes

c. microcephaly Signs that are likely to be present in the 10 percent of newborns who are symptomatic at birth include microcephaly, seizures, IUGR, hepatosplenomegaly, jaundice, and rash.

When caring for a client requiring a forceps-assisted birth, the nurse would be alert for: a. increased risk for cord entanglement. b. increased risk for uterine rupture. c. potential lacerations and bleeding. d. damage to the maternal tissues.

c. potential lacerations and bleeding. Forcible rotation of the forceps can cause potential lacerations and bleeding. Cervical ripening increases the risk for uterine rupture in a client attempting vaginal birth after undergoing at least one previous cesarean birth. There is an increased risk for cord entanglement in multiple pregnancies. Damage to the maternal tissues happens if the cup slips off the fetal head and the suction is not released.

After assessing a client's progress of labor, the nurse suspects the fetus is in a persistent occiput posterior position. Which finding would lead the nurse to suspect this condition? a. contractions most forceful in the middle of uterus rather than the fundus b. fetal buttocks as the presenting part c. reports of severe back pain d. lack of cervical dilation past 2 cm

c. reports of severe back pain Reports of severe back pain are associated with a persistent occiput posterior position due to the pressure of the fetal head on the woman's sacrum and coccyx. Cervical dilation that has not progressed past 2 cm is associated with dysfunctional labor. A breech position is one in which the fetal presenting part is the buttocks or feet. Contractions that are more forceful in the midsection of the uterus rather than in the fundus suggest hypertonic uterine dysfunction.

A client is diagnosed with peripartum cardiomyopathy (PPCM). Which therapy would the nurse expect to administer to the client? a. ginger therapy b. methadone therapy c. restricted sodium intake d. monoamine oxidase inhibitors

c. restricted sodium intake The client with peripartum cardiomyopathy should be prescribed restricted sodium intake to control the blood pressure. Monoamine oxidase inhibitors are given to treat depression in pregnancy not peripartum cardiomyopathy. Methadone is a drug given for the treatment of a substance use disorder during pregnancy. Complimentary therapies like ginger therapy help in the alleviation of hyperemesis gravidarum not peripartum cardiomyopathy.

A woman's perception of pain can differ according to all of the following except: a. her expectations and preparation for labor. b. psychosocial, physiologic, and cultural influences. c. the presentation, lie, and attitude of the fetus. d. fear, anxiety, and self-efficacy. e. the length of her labor.

c. the presentation, lie, and attitude of the fetus. Fetal position can influence a client's perception of pain. Fetal attitude does not influence a client's perception of pain.

A woman at 8 weeks' gestation is admitted for ectopic pregnancy. She is asking why this has occurred. The nurse knows that which factor is a known risk factor for ectopic pregnancy? a. multiple gestation pregnancy b. high number of pregnancies c. use of IUD for contraception d. use of oral contraceptives

c. use of IUD for contraception Use of an IUD with progesterone has a known increased risk for development of ectopic pregnancies. The nurse needs to complete a full history of the client to determine if she had any other risk factors for an ectopic pregnancy. Adhesions, scarring, and narrowing of the tubal lumen may block the zygote's progress to the uterus. Any condition or surgical procedure that can injure a fallopian tube increases the risk. Examples include salpingitis, infection of the fallopian tube, endometriosis, history of prior ectopic pregnancy, any type of tubal surgery, congenital malformation of the tube, and multiple elective abortions. Conditions that inhibit peristalsis of the tube can result in tubal pregnancy. Hormonal factors may play a role because tubal pregnancy occurs more frequently in women who take fertility drugs or who use progesterone intrauterine contraceptive devices (IUDs). A high number of pregnancies, multiple gestation pregnancy, and the use of oral contraceptives are not known risk factors for ectopic pregnancy.

The nurse is monitoring a client who just received IV sedation. Which instruction should the nurse prioritize with the client and her partner? a. Sit on the edge of the bed with her feet dangling before ambulating. b. Ambulate only with assistance from the nurse or caregiver. c. Remain in bed for at least 30 minutes. d. Ambulate within 15 minutes to prevent spinal headache.

d. Ambulate within 15 minutes to prevent spinal headache. The client may have decreased sensory ability from the medication. She needs assistance to ambulate for safety. She will be largely unable to move, so she should remain in bed unless absolutely necessary.

The health care provider is evaluating a high-risk woman for a continuous internal monitoring. It would be most appropriate to meet which criterion? a. insertion by any staff b. cervical dilation of 1 cm c. the presenting fetal part not visible d. rupture of membranes

d. rupture of membranes The insertion of the spiral electrode should be inserted only by a skilled practitioner. Ruptured membranes, cervical dilation of at least 2 cm, and the presenting fetal part low enough to allow placement of the scalp electrode are all necessary.

A nurse is teaching a woman diagnosed with gestational diabetes about meal planning and nutrition. The nurse determines that additional teaching is needed based on which client statement? a. "It's okay to eat small meals or snacks throughout the day." b. "Having a bedtime snack is good for me." c. "I should get most of my calories from good complex carbs." d. "I need to avoid any fat with my meals."

d. "I need to avoid any fat with my meals." Recommendations for nutrition and diet with gestational diabetes include: eating three meals a day plus three snacks to promote glycemic control with 40% of calories from good-quality complex carbohydrates, 35% of calories from protein sources, and 25% of calories from unsaturated fats; eating small frequent feedings throughout the day; having bedtime snacks; and including protein and fat at each meal.

A mother is talking to the nurse and is concerned about managing her asthma while she is pregnant. Which response to the nurse's teaching indicates that the woman needs further instruction? a. "It is fine for me to use my albuterol inhaler if I begin to feel tight." b. "I will monitor my peak expiratory flow rate regularly to help me predict when an asthma attack is coming on." c. "I need to be aware of my triggers and avoid them as much as possible." d. "I need to begin taking allergy shots like my friend to prevent me from having an allergic reaction this spring."

d. "I need to begin taking allergy shots like my friend to prevent me from having an allergic reaction this spring." A pregnant woman with a history of asthma needs to be proactive, taking her inhalers and other asthma medications to prevent an acute asthma attack. She needs to understand that it is far more dangerous to not take the medications and have an asthma attack. She also needs to monitor her peak flow for decreases, be aware of triggers, and avoid them if possible. However, a pregnant woman should never begin allergy shots if she has not been taking them previously, due to the potential of an adverse reaction.

A nurse is talking to a newly pregnant woman who had a mitral valve replacement in the past. Which statement by the client reveals an understanding about the preexisting condition? a. "I don't have to worry about this because I had the problem fixed before I became pregnant." b. "I know my baby will be fine, but I am worried about having a personal complication." c. "I know I will be fine, but I worry about the fetus." d. "I understand that my fetus and I both are at risk for complications."

d. "I understand that my fetus and I both are at risk for complications." When a woman enters pregnancy with a preexisting condition, both she and her fetus can be at risk of developing complications.

The nurse is teaching a client with gestational diabetes about complications that can occur either following birth or at delivery for her baby. Which statement by the mother indicates that further teaching is needed by the nurse? a. "I may need an amniocentesis during the third trimester to see if my baby's lungs are ready to be born." b. "Beginning at 28 weeks' gestation, I will start counting with my baby's movements every day." c. "My baby may be very large and I may need a cesarean section to have him." d. "If my blood sugars are elevated, my baby's lungs will mature faster, which is good."

d. "If my blood sugars are elevated, my baby's lungs will mature faster, which is good." Elevated blood sugars delay the maturation of fetal lungs, not increase maturation time, resulting in potential respiratory distress in newborns born to diabetic mothers. Doing "kick counts", as the fetal movement monitoring is often called, is standard practice, as is the possibility of an amniocentesis to determine lung maturity during the third trimester. Health care personnel should also prepare the mother for the potential of a cesarean section delivery if the infant is too large.

A woman has been in labor for the past 8 hours, and she has progressed to the second stage of labor. However, after 2 hours with no further descent, the provider diagnoses an "arrested descent." The woman asks, "Why is this happening?" Which response is the best answer to this question? a. "Maybe your baby has developed hydrocephaly and the head is too swollen." b. "It is likely that your body has not secreted enough hormones to soften the ligaments so your pelvic bones can shift to allow birth of the baby." c. "Maybe your uterus is just tired and needs a rest." d. "More than likely you have cephalopelvic disproportion (CPD) where baby's head cannot make it through the canal."

d. "More than likely you have cephalopelvic disproportion (CPD) where baby's head cannot make it through the canal." Arrest of descent results when no descent has occurred for 2 hours in a nullipara or 1 hour in a multipara. The most likely cause for arrest of descent during the second stage is CPD. Rest should allow the uterine contractions to be more efficient. The hormones secreted during pregnancy allow ligaments to soften so bones can shift to allow birth. Ultrasound would have previously been diagnosed prior to the onset of labor.

After a regular prenatal visit, a pregnant client asks the nurse to describe the differences between abruptio placentae and placenta previa. Which statement should the nurse include in the teaching? a. "Placenta previa causes painful, dark red bleeding during pregnancy due to an abnormally implanted placentae that is too close to or covers the cervix; abruptio placenta is associated with bright red painless bleeding caused by premature separation of the placenta from the wall of the uterus before the end of labor." b. "Placenta previa causes painful, dark red bleeding during pregnancy due to an abnormally implanted placenta that is too close to or covers the fundus; abruptio placentae is associated with right red painless bleeding caused by premature separation of the placenta from the wall of the uterus before the end of labor." c. "Placenta previa causes painless, bright red bleeding during pregnancy due to an abnormally implanted placenta that is too close to or covers the fundus; abruptio placentae is associated with dark red painful bleeding caused by premature separation of the placenta from the wall of the uterus before the end of labor." d. "Placenta previa causes painless, bright red bleeding during pregnancy due to an abnormally implanted placenta that is too close to or covers the cervix; abruptio placentae is associated with dark red painful bleeding caused by premature separation of the placenta from the wall of the uterus before the end of labor."

d. "Placenta previa causes painless, bright red bleeding during pregnancy due to an abnormally implanted placenta that is too close to or covers the cervix; abruptio placentae is associated with dark red painful bleeding caused by premature separation of the placenta from the wall of the uterus before the end of labor." Placenta previa is a condition of pregnancy in which the placenta is implanted abnormally in the lower part of the uterus and is the most common cause of painless bright red bleeding in the third trimester. Abruptio placenta is the premature separation of a normally implanted placenta that pulls away from the wall of the uterus either during pregnancy or before the end of labor.

A woman's baby is HIV positive at birth. She asks the nurse if this means the baby will develop AIDS. Which statement would be the nurse's best answer? a. "HIV antibodies do not cross the placenta; this means the baby will develop AIDS." b. "She already has AIDS. That's what being HIV positive means." c. "HIV is transmitted at birth; having a cesarean birth prevented transmission." d. "The antibodies may be those transferred across the placenta; the baby may not develop AIDS."

d. "The antibodies may be those transferred across the placenta; the baby may not develop AIDS." Infants born of HIV-positive women test positive for HIV antibodies at birth because these have crossed the placenta. An accurate disease status cannot be determined until the antibodies fade at about 18 months. Testing positive for HIV antibodies does not mean the infant has AIDS. Having a cesarean birth does decrease the risk of transmitting the virus to the infant at birth; it does not prevent the transmission of the disease. HIV antibodies do cross the placenta, which is why babies born of HIV positive mothers are HIV positive.

A client in her first trimester arrives at the emergency room with reports of severe cramping and vaginal spotting. On examination, the health care provider informs her that no fetal heart sounds are evident and orders a dilatation and curettage. The client looks frightened and confused and states that she does not believe in abortion. Which statement by the nurse is best? a. "You have experienced an incomplete miscarriage and must have the placenta and any other tissues cleaned out." b. "I know that it is sad but the pregnancy must be terminated to save your life." c. "The choice is up to you but the healthcare provider is recommending an abortion." d. "Unfortunately, the pregnancy is already lost. The procedure is to clear the uterus to prevent further complications."

d. "Unfortunately, the pregnancy is already lost. The procedure is to clear the uterus to prevent further complications." The nurse should not inform the client what she must do but supply information about what has happened and teach the client about the treatments which are used to correct the situation. A threatened miscarriage becomes an imminent (inevitable) miscarriage if uterine contractions and cervical dilation occur. A woman who reports cramping or uterine contractions is asked to seek medical attention. If no fetal heart sounds are detected and an ultrasound reveals an empty uterus or nonviable fetus, her health care provider may perform a dilatation and curettage (D&C) or a dilation and evacuation (D&E) to ensure all products of conception are removed. Be certain the woman has been told the pregnancy was already lost and all procedures, such as suction curettage, are to clear the uterus and prevent further complications such as infection, not to end the pregnancy. This scenario does not involve an elective abortion or an incomplete miscarriage.

The nurse is noting a collection of blood under the scalp on a newborn being discharged to home. The nurse is correct to prepare teaching instructions of which topic? a. Developmental delay b. A caput succedaneum c. Wrapping of the head d. A cephalohematoma

d. A cephalohematoma Blood collection under the scalp of the newborn from birth trauma is called a cephalohematoma. Instructions for care include simple observation of the area. The cephalohematoma will subside in a couple of weeks and may take a couple of months to completely go away. There is no brain damage associated with a cephalohematoma. A caput succedaneum is swelling, without blood collection, of the soft tissue of the head.

The nurse is caring for a client in the transition stage of labor. In which scenario would the nurse predict the use of forceps may be used to assist in delivery? a. The fetus is descending too slowly b. Reduce risk of complications c. To lessen the mother's pain d. Abnormal position of the fetal head

d. Abnormal position of the fetal head Forceps are mechanical devices which can be used to help deliver the fetus. Although no longer used routinely they are still used in certain situations to assist with the deliver. One of those situations is when the fetus is in an abnormal position and the health care provider attempts to reposition the fetus to facilitate birth. The use of forceps is not to lessen the mother's pain or to speed up the process. The use of forceps is not without risk of complications to include perineal lacerations and injury to the fetus if done incorrectly.

The nurse is analyzing the readout on the EFM and determines the FHR pattern is reassuring based on which recording? a. Decrease in variability for 15 seconds b. Deceleration followed by acceleration of 15 bpm c. Increase in variability by 27 bpm d. Acceleration of at least 15 bpm for 15 seconds

d. Acceleration of at least 15 bpm for 15 seconds A reassuring active fetal heart rate is a change in baseline by increase of 15 bpm for 15 seconds. This is a positive and reassuring periodic change in fetal heart rates as a response to fetal movement. Normal variability is noted to occur within 6 to 25 bpm from the baseline FHR. There should be no decelerations.

A nurse is caring for a client with hyperemesis gravidarum. Which nursing action is the priorityfor this client? a. Administer total parenteral nutrition. b. Administer an antiemetic. c. Set up for a percutaneous endoscopic gastrostomy. d. Administer IV NS with vitamins and electrolytes.

d. Administer IV NS with vitamins and electrolytes. The first choice for fluid replacement is generally NS with vitamins and electrolytes added. If the client does not improve after several days of bed rest, "gut rest," IV fluids, and antiemetics, then total parenteral nutrition or percutaneous endoscopic gastrostomy tube feeding is instituted to prevent malnutrition.

Which statement is true regarding analgesia versus anesthesia? a. Decreased FHR variability is a common side effect when regional anesthesia is used. b. Hypotension is the most common side effect when systemic analgesia is used. c. Regional anesthesia should be given with caution close to the time of birth because it crosses the placenta and can cause respiratory depression in the newborn. d. Analgesia only reduces pain, but anesthesia partially or totally blocks all pain in a particular area.

d. Analgesia only reduces pain, but anesthesia partially or totally blocks all pain in a particular area. Systemic analgesia should be used with caution near the time of birth because it can cause respiratory depression, in addition to decreased FHR variability. Hypotension is a common side effect of regional anesthesia.

The nurse identifies a nursing diagnosis of risk for injury related to possible effects of oxytocin therapy. Which action would the nurse perform to ensure a positive outcome for the client? a. Administer hydration and sedation frequently. b. Turn down oxytocin administration by half. c. Start administering tocolytic therapy. d. Assess contractions by using external monitor.

d. Assess contractions by using external monitor. In a client with the risk for injury, continuous assessment of contractions using external monitor and palpation to ensure the presence of a low resting tone will assist in collecting information about labor and the need for further intervention. Turning down oxytocin administration by half is required if hyperstimulation occurs not to prevent it. Tocolytic therapy is generally employed when preterm labor has been definitively diagnosed. Administering hydration and sedation frequently and bedrest are employed to halt preterm labor since these stop uterine activity by increasing intravascular volume and uterine blood flow.

A client's membranes have just ruptured. Her fetus is presenting breech. Which action should the nurse do immediately to rule out prolapse of the umbilical cord in this client? a. Administer amnioinfusion. b. Place the woman in Trendelenburg position. c. Administer oxygen at 10 L/min by face mask. d. Assess fetal heart sounds.

d. Assess fetal heart sounds. To rule out cord prolapse, always assess fetal heart sounds immediately after rupture of the membranes whether this occurs spontaneously or by amniotomy, as the fetal heart rate will be unusually slow or a variable deceleration pattern will become apparent when cord prolapse has occurred. The other answers refer to therapeutic interventions to implement once cord prolapse has been confirmed.

A client has been in labor for 10 hours and is 6 cm dilated. She has already expressed a desire to use nonpharmacologic pain management techniques. For the past hour, she has been lying in bed with her doula rubbing her back. Now, she has begun to moan loudly, grit her teeth, and bear down with each contraction. She rates her pain as 8 out of 10 with each contraction. What should the nurse do first? a. Assist the client in ambulating to the bathroom. b. Instruct the client to do slow-paced breathing. c. Prepare the client for an epidural. d. Assess for labor progression.

d. Assess for labor progression. Performing breathing exercises, ambulating, changing position, and emptying the bladder all can help the client experience a reduction in pain. However, the best first step is to assess the client for labor progress before assisting her otherwise. Bearing down can be a sign that the client is 10 cm dilated.

At which point along the birth canal must the fetal head extend for successful passage? a. At the level of the ischial spines b. At the level of the iliac crest c. At the level of the pelvic inlet d. At the level of the symphysis pubis

d. At the level of the symphysis pubis Fetal extension occurs late in the labor process as the fetus extends through the final portion of the passageway. The fetal head must extend at the symphysis pubis for successful passage. The next step is the head being born. The pelvic inlet, iliac crest and ischial spines are high in the birth canal.

When educating the post-term pregnant client, what should the nurse be sure to include to prevent fetal complications? a. Increase fluid intake to prevent dehydration. b. Monitor bowel movements. c. Be sure to measure 24-hour urine output daily. d. Be sure to monitor fetal movements daily.

d. Be sure to monitor fetal movements daily. The nurse should be sure to teach the post-term client to monitor fetal movements daily.

A nurse is preparing a nursing care plan for a client who is admitted at 22 weeks' gestation with advanced cervical dilatation to 5 cm, cervical insufficiency, and a visible amniotic sac at the cervical opening. Which primary goal should the nurse prioritize at this point? a. Notification of social support for loss of pregnancy b. Give birth vaginally c. Education on causes of cervical insufficiency for the future d. Bed rest to maintain pregnancy as long as possible

d. Bed rest to maintain pregnancy as long as possible At 22 weeks' gestation, the fetus is not viable. The woman would be placed on total bed rest with every attempt made to halt any further progression of dilatation of the cervix. The nurse would not want this fetus to be born vaginally at this stage of gestation. It is not the nurse's responsibility to notify the client's social support of a possible loss of the pregnancy. It is not appropriate at this time to educate the mother on causes of cervical insufficiency for future pregnancies.

A nurse is providing care to a woman in labor. When reviewing the woman's medical record, the nurse notes that fetal position is documented as LSA. The nurse interprets this to mean that which part of the fetus is presenting? a. Acromion process b. Occiput c. Chin d. Buttocks

d. Buttocks The second letter of LSA denotes the presenting part. In this case, it is "S" which is for sacrum or buttocks. "O" refers to the occiput; "M" would be used to refer to the chin. "A" would be used to refer to the acromion process.

A client arrives at the clinic in labor. The nurse assesses a bulging perineum and prepares for the birth. Place the nurse's actions in sequence. All options must be used. a. Palpate for a nuchal cord b. Use bulb to suction mouth and nose c. Support perineum with one hand d. Call for assistance e. Deliver the head f. Put on gloves

d. Call for assistance f. Put on gloves c. Support perineum with one hand e. Deliver the head a. Palpate for a nuchal cord b. Use bulb to suction mouth and nose The nurse assesses a bulging perineum noting that birth is imminent. The nurse would call for assistance, put on gloves, support the perineum, deliver the head, palpate for a nuchal cord, and use bulb to suction mouth and nose.

There are four essential components of labor. The first is the passageway. It is composed of the bony pelvis and soft tissues. What is one component of the passageway? a. Uterus b. Perineum c. False pelvis d. Cervix

d. Cervix The cervix and vagina are soft tissues that form the part of the passageway known as the birth canal.

The nurse is monitoring a laboring client with continuous fetal monitoring and notes a decrease in FHR with variable deceleration to 75 bpm. Which intervention should the nurse prioritize? a. Increase her IV fluids. b. Notify the primary care provider. c. Administer oxygen. d. Change the position of the client.

d. Change the position of the client. Variable decelerations often indicate a type of cord compression. The initial response is to change the position and try to release the cord compression. If this does not work, apply oxygen while using the call light to alert others. If this continues, her fluid status needs to be assessed before increasing her IV rate.

The nurse is comforting and listening to a young couple who just suffered a miscarriage. When asked why this happened, which reason should the nurse share as a common cause? a. The age of the mother b. Maternal smoking c. Lack of prenatal care d. Chromosomal abnormality

d. Chromosomal abnormality The most common cause for the loss of a fetus in the first trimester is associated with a genetic defect or chromosomal abnormality. There is nothing that can be done, and the mother should feel no fault. The nurse needs to encourage the parents to speak with a health care provider for further information and questions related to genetic testing. Early pregnancy loss is not associated with maternal smoking, lack of prenatal care, or the age of the mother.

The nurse is documenting the length of time in the second stage of labor. Which data will the nurse use to complete the documentation? a. Admission time and time of fetal birth b. Time of mucous plug expulsion and full cervical dilation c. Effacement time and time when contractions are regular d. Complete cervical dilation and time of fetal birth

d. Complete cervical dilation and time of fetal birth The second stage of labor begins with complete cervical dilation of 10 cm and ends with delivery of the neonate. All other options are incorrect time frames.

Between her regularly scheduled visits, a woman in her first trimester of pregnancy who is taking iron supplements for anemia calls the nurse at her obstetrician's office reporting constipation. She reports that she has never had this problem before and asks for some advice about how to get relief. What is the best advice the nurse can give her? a. Increase the iron supplements, fluid intake, and consumption of high-fiber foods; exercise more. b. Stop taking iron supplements for a few days, exercise more, drink more fluids, eat high-fiber, low-iron foods until the constipation is relieved, then resume the iron supplement. c. Take the iron supplement every other day, increase fluid intake and consumption of high-fiber foods; exercise more. d. Continue taking iron supplements but increase fluids and high-fiber foods; exercise more.

d. Continue taking iron supplements but increase fluids and high-fiber foods; exercise more. Constipation is a common side effect of iron supplementation. The diagnosis of anemia indicates a true need for the iron supplementation; she needs to increase fluid and fiber to relieve the constipation associated with the iron preparations. The nurse should not advise this client to stop taking her iron supplements, even for a few days. The nurse should not advise the client to increase her iron supplementation, nor take the supplements on an every other day basis. These supplements are ordered by the primary care provider based on the client's hematologic status.

A G2P1 woman with type 1 diabetes is determined to be at 8 weeks' gestation by her health care provider. The nurse should point out which factor will help the client maintain glycemic control? a. Vitamin supplements b. Oral hypoglycemic agents c. Plenty of rest d. Exercise

d. Exercise The three main facets to glycemic control for the woman with pregestational diabetes are diet, exercise, and insulin. An individual with type 1 diabetes uses insulin and not oral hypoglycemic agents. Vitamin supplements may assist with helping to keep the woman healthy but not necessarily through glycemic control. It will be important for the woman to get enough rest throughout the pregnancy but this will not assist with glycemic control.

The nurse is preparing a postpartum nursing care plan for a single HIV-positive primigravida client. The nurse should prioritize in the plan how to acquire which resource? a. Car seat b. Diapers c. Breast pump d. Formula

d. Formula It is possible to transmit HIV via breastfeeding, and formula is the only option for feeding. The nurse needs to provide positive information and offer to make a referral or get assistance for clients who may be in financial need. In this case, acquiring adequate amounts of formula would be the priority. The diapers and a car seat are also necessary but would follow the formula. The client would not need a breast pump since she cannot give the milk to her baby.

At 31 weeks' gestation, a 37-year-old woman with a history of preterm birth reports cramps, vaginal pain, and low, dull backache accompanied by vaginal discharge and bleeding. Assessment reveals cervix 2.1 cm long; fetal fibronectin in cervical secretions, and cervix dilated 3 to 4 cm. Which interactions should the nurse prepare to assist with? a. Bed rest and hydration at home b. Careful monitoring of fetal kick counts c. An emergency cesarean birth d. Hospitalization, tocolytic, and corticosteroids

d. Hospitalization, tocolytic, and corticosteroids At 31 weeks' gestation, the goal would be to maintain the pregnancy as long as possible if the mother and fetus are tolerating continuation of the pregnancy. Stopping the contractions and placing the client in the hospital allow for monitoring in a safe place if the woman continues and gives birth. Administration of corticosteroids may help to develop the lungs and prepare for early preterm birth. Sending the woman home is contraindicated in the scenario described. An emergency cesarean birth is not indicated at this time. Monitoring fetal kick counts is typically done with a post-term pregnancy.

A nurse is teaching a couple about patterned breathing during their birth education. Which technique should the nurse suggest for slow-paced breathing? a. Hold breath for 5 seconds after every 3 breaths. b. Inhale and exhale through the mouth at a rate of 4 breaths every 5 seconds. c. Punctuated breathing by a forceful exhalation through pursed lips every few breaths. d. Inhale slowly through nose and exhale through pursed lips.

d. Inhale slowly through nose and exhale through pursed lips. For slow-paced breathing, the nurse should instruct the woman to inhale slowly through her nose and exhale through pursed lips. In shallow or modified-pace breathing, the woman should inhale and exhale through her mouth at a rate of 4 breaths every 5 seconds. In pattern-paced breathing, the breathing is punctuated every few breaths by a forceful exhalation through pursed lips. Holding the breath for 5 seconds after every three breaths is not recommended in any of the three levels of patterned breathing.

A woman who is 10 weeks' pregnant calls the physician's office reporting "morning sickness" but, when asked about it, tells the nurse that she is nauseated and vomiting all the time and has lost 5 pounds. What interventions would the nurse anticipate for this client? a. Since morning sickness is a common problem for pregnant women, the nurse will suggest the woman drink more fluids and eat crackers. b. The nurse will encourage the woman to lie down and rest whenever she feels ill. c. An ultrasound will be done to reassess the correctness of gestational dates. d. Lab work will be drawn to rule out acid-base imbalances.

d. Lab work will be drawn to rule out acid-base imbalances. Morning sickness that lasts all day and is severe is called hyperemesis gravidarum. It is much more serious than "morning sickness" and can lead to significant weight loss and electrolyte imbalance. Lab work needs to be drawn to determine the extent of electrolyte loss and acid-base balance. An ultrasound is performed but it is done to determine if the mother is experiencing a molar pregnancy. Treatment for hyperemesis gravidarum requires much more care than just rest, drinking fluids and eating crackers.

The nurse is preparing information for a client who has just been diagnosed with gestational diabetes. Which instruction should the nurse prioritize in this information? a. Plan daily menus with dietitian b. Report any signs of possible urinary tract infection c. Long term therapy goals d. Maintain a daily blood glucose log

d. Maintain a daily blood glucose log Control of the blood glucose throughout the pregnancy is the primary goal to help decrease potential complications to both the mother and fetus. The mother should keep a daily log of her blood glucose levels and bring this log to each visit for the nurse to evaluate. The other choices of reporting possible signs of a UTI and working with a dietitian to plan menus would also be important but would follow stressing the blood glucose control. It would be inappropriate to discuss long-term goals at this time. This would be handled at a later time and would depend on the mother's situation.

A laboring client has been pushing without delivering the fetal shoulders. The primary care provider determines the fetus is experiencing shoulder dystocia. What intervention can the nurse assist with to help with the birth? a. positioning the woman prone b. fundal pressure c. Lamaze position d. McRoberts maneuver

d. McRoberts maneuver The McRoberts maneuver is frequently successful and often tried first. It requires assistance from two people. Two nurses place the client in the lithotomy position, while each holds a leg and sharply flexes the leg toward the woman's shoulders. This opens the pelvis to its widest diameters and allows the anterior shoulder to deliver in almost half of the cases.

The skull is the most important factor in relation to the labor and birth processes. The fetal skull must be small enough to travel through the bony pelvis. What feature of the fetal skull helps to make this passage possible? a. Vertex presentation b. Caput succedaneum c. Cephalohematoma d. Molding

d. Molding The cartilage between the bones allows the bones to overlap during labor, a process called molding that elongates the fetal skull, thereby reducing the diameter of the head.

The nurse is caring for a pregnant client with fallopian tube rupture. Which intervention is the priority for this client? a. Monitor the fetal heart rate (FHR). b. Monitor the mass with transvaginal ultrasound. c. Monitor the client's beta-hCG level. d. Monitor the client's vital signs and bleeding.

d. Monitor the client's vital signs and bleeding. A nurse should closely monitor the client's vital signs and bleeding (peritoneal or vaginal) to identify hypovolemic shock that may occur with tubal rupture. Beta-hCG level is monitored to diagnose an ectopic pregnancy or impending abortion. Monitoring the mass with transvaginal ultrasound and determining the size of the mass are done for diagnosing an ectopic pregnancy. Monitoring the FHR does not help to identify hypovolemic shock.

A pregnant woman diagnosed with diabetes should be instructed to perform which action? a. Discontinue insulin injections until 15 weeks gestation. d. Notify the primary care provider if unable to eat because of nausea and vomiting. c. Prepare foods with increased carbohydrates to provide needed calories. d. Ingest a smaller amount of food prior to sleep to prevent nocturnal hyperglycemia.

d. Notify the primary care provider if unable to eat because of nausea and vomiting. During pregnancy, the insulin levels change in response to the production of HPL. The client needs to alert her provider if she is not able to eat or hold down appropriate amounts of nutrition. The client is at risk for episodes of hypoglycemia during the first trimester. She should never discontinue insulin therapy without her provider's directions. The increase of carbohydrates needs to be balanced with protein, and smaller meals would result in hypoglycemia rather than hyperglycemia.

A woman presents at Labor and Delivery very upset. She reports that she has not felt her baby moving for the last 6 hours. The nurse listens for a fetal heart rate and cannot find a heartbeat. An ultrasound confirms fetal death and labor induction is started. What intervention by the nurse would be appropriate for this mother at this time? a. Call the hospital chaplain to talk to the parents. b. Explain to her that there was probably something wrong with the infant and that is why it died. c. Recommend that she not hold the infant after it is delivered so as to not upset her more. d. Offer to take pictures and footprints of the infant once it is delivered.

d. Offer to take pictures and footprints of the infant once it is delivered. When parents are faced with a fetal death, they need comfort and support without being intrusive. Taking pictures, footprints and gathering other mementos are very important in helping the family deal with the death better. The mother is encouraged to hold the infant after delivery and name it. Telling the parents that the infant was probably defective is hurtful and not supportive to them. Calling the hospital chaplain is something that can be offered but should not be done without the parent's approval.

The nurse is leading a discussion with a group of pregnant women who have diabetes. The nurse should point out which situation can potentially occur during their pregnancy? a. Hypotension of pregnancy b. Post-term birth c. Small for gestation age infant d. Polyhydramnios

d. Polyhydramnios Polyhydramnios is an increase, or excess, in amniotic fluid and is a pregnancy-related complication associated with diabetes. An infant who is small for gestational age is not associated with a mother who had diabetes prior to pregnancy. Other pregnancy-related complications associated with pregestational DM include hypertensive disorders, preterm birth, and shoulder dystocia

A woman in labor suddenly reports sharp fundal pain accompanied by slight dark red vaginal bleeding. The nurse should prepare to assist with which situation? a. Preterm labor that was undiagnosed b. Placenta previa obstructing the cervix c. Possible fetal death or injury d. Premature separation of the placenta

d. Premature separation of the placenta Premature separation of the placenta begins with sharp fundal pain, usually followed by dark red vaginal bleeding. Placenta previa usually produces painless bright red bleeding. Preterm labor contractions are more often described as cramping. Possible fetal death or injury does not present with sharp fundal pain. It is usually painless.

A client at 27 weeks' gestation is admitted to the OB unit after reporting headaches and edema of her hands. Review of the prenatal notes reveals BP consistently above 136/90 mm Hg. The nurse anticipates the health care provider will order magneisum sulfate to accomplish which primary goal? a. Decrease blood pressure b. Decrease protein in urine c. Reverse edema d. Prevent maternal seizures

d. Prevent maternal seizures The primary therapy goal for any preeclamptic client is to prevent maternal seizures. Use of magnesium sulfate is the drug therapy of choice for severe preeclampsia and is only used to manage and attempt to prevent progression to eclampsia. Magnesium sulfate therapy does not have as its primary goal a decrease in blood pressure, a decrease in protein in the urine, nor the reversal of edema.

Which action would be most appropriate for the woman who experiences dysfunctional labor in the first stage of labor? a. Tell her not to feel anxious or discouraged about what is happening. b. Limit talking to things the woman asks questions about. c. Hold all explanations until after the birth to conserve the woman's energy. d. Provide ongoing communication about what is happening.

d. Provide ongoing communication about what is happening. Dysfunctional labor at any point is frustrating to women. Maintaining open lines of communication at least keeps the woman well informed about what is happening.

A nurse is coaching a woman during the second stage of labor. Which action should the nurse encourage the client to do at this time? a. Begin pushing as soon as the cervix has dilated to 8 cm. b. Hold the breath while pushing during contractions. c. Pant while pushing. d. Push with contractions and rest between them.

d. Push with contractions and rest between them. Make sure the woman pushes with contractions and rests between them. Holding the breath during a contraction could cause a Valsalva maneuver or temporarily impede blood return to her heart because of increased intrathoracic pressure, which could then also interfere with blood supply to the uterus. It is important for women to understand they should not bear down with their abdominal muscles to push until the cervix is fully dilated, which is 10 cm, not 8 cm. Panting limits the ability to push and is to be encouraged only when it is desirable to delay labor, such as when a nuchal cord is present.

The nurse would prepare a client for amnioinfusion when which action occurs? a. Fetal presenting part fails to rotate fully and descend in the pelvis. b. The fetus shows abnormal fetal heart rate patterns. c. Maternal pushing is compromised due to anesthesia. d. Severe variable decelerations occur and are due to cord compression.

d. Severe variable decelerations occur and are due to cord compression. Indications for amnioinfusion include severe variable decelerations resulting from cord compression, oligohydramnios (decreased amniotic fluid), postmaturity, preterm labor with rupture of the membranes, and thick meconium fluid. Failure of the fetal presenting part to rotate fully, descend in the pelvis, abnormal fetal heart rate patterns or acute pulmonary edema, and compromised maternal pushing sensations from anesthesia are indications for forceps-assisted birth, and not for amniofusion.

A 39-year-old multigravida with diabetes presents at 32 weeks' gestation reporting she has not felt movement of her fetus. Assessment reveals the fetus has died. The nurse shares with the mother that the institution takes pictures after the birth and asks if she would like one. What is the best response if the mother angerily says no and starts crying? a. Console her with the fact that she has other children. b. Tell her that once she gets over her shock and grief, she will probably be happy to have the photos. c. Apologize and tell her that the photos will be destroyed immediately. d. Tell her that the hospital will keep the photos for her in case she changes her mind.

d. Tell her that the hospital will keep the photos for her in case she changes her mind. Emotional care of the woman is complex, especially one who has suffered the loss of a child. The woman will need time to move through the stages of grief and the responses of grief vary from person to person. The mother may request the items later and they should be stored or kept for a year after the delivery. There is no need to apologize to the client. It would be inappropriate to console her with the fact that she has other children. It negates her feelings and is not supportive of the woman at this time.

The nurse is caring for a client at 39 weeks' gestation and whose fetal station is noted as a 0 (zero). The nurse is correct to document which? a. The fetus is floating high in the pelvis. b. The client is fully effaced. c. The fetus has descended down the birth canal. d. The fetus is in the true pelvis and engaged.

d. The fetus is in the true pelvis and engaged. When the fetus is at a 0 (zero) station, it is at the level of the ischial spines and said to be engaged. Determining the station does not mean that the client's cervix is fully effaced. If the fetus is floating high in the pelvis, its station is noted as a negative number. Descending into the pelvis or birth canal is documented as a positive number.

While monitoring the EFM tracing the nurse notes decelerations with each contraction. The nurse knows that for a deceleration to be classified as early it has to meet three criteria. What is one of these criteria? a. The FHR begins to accelerate as the contraction begins. b. The nadir of the deceleration falls midway between the acme of two contractions. c. The deceleration ends midway between two contractions. d. The nadir of the deceleration coincides with the acme of the contraction.

d. The nadir of the deceleration coincides with the acme of the contraction. Three criteria classify the deceleration as early: (1) the FHR begins to slow as the contraction starts; (2) the lowest point of the deceleration, the nadir, coincides with the acme (highest point) of the contraction; and (3) the deceleration ends by the end of the contraction.

The nurse is caring for a client who has been in labor for the past 8 hours. The nurse determines that the client has transitioned into the second stage of labor based on which sign? a. Emotions are calm and happy. b. Fetus is at -1 station. c. Frequency of contractions are 5 to 6 minutes. d. The urge to push occurs.

d. The urge to push occurs. Second stage of labor is the pushing stage; this is typically identified by the woman's urge to push or a feeling of needing to have a bowel movement. In the second stage the cervix can be 10 cm, dilated 100% and effaced. The station is usually 0 to +2. The emotional state may be altered due to pain and pressure. Contraction frequency is variable and not clearly indicative of a particular stage. The fetus can be at stage -1 for any length of time.

A primigravida 21-year-old client at 24 weeks' gestation has a 2-year history of HIV. As the nurse explains the various options for delivery, which factor should the nurse point out will influence the decision for a vaginal birth? a. Amniocentesis results at 34 weeks b. Prophylactic ART to infant at birth c. The mother's age d. The viral load

d. The viral load A woman who has HIV during pregnancy is at risk for transmitting the infection to the fetus during pregnancy or childbirth and to the newborn while breastfeeding. The type of birth, vaginal or cesarean, depends on several factors, including the woman's viral load, use of ART during pregnancy (not waiting until the birth), length of time membranes have been ruptured, and gestational age (not mother's age). With prenatal ART and prophylactic treatment of the newborn, there is a reduced risk of perinatal HIV transmission. The amniocentesis results would not be a factor in preventing the spread of HIV to the infant and may actually lead to the fetus being infected through the puncture site and bleeding into the amniotic sac.

The laboring client who is at 3 cm dilation and 25% effaced is asking for analgesia. The nurse explains the analgesia usually is not administered prior to the establishment of the active phase. What is the appropriate rationale for this practice? a. This can lead to maternal hypertension. b. This would cause fetal depression in utero. c. The effects would wear off before birth. d. This may prolong labor and increase complications.

d. This may prolong labor and increase complications. Administration of pharmacologic agents too early in labor can stall the labor and lengthen the entire labor. The client should be offered nonpharmacologic options at this point until she is in active labor.

A young woman presents at the emergency department reporting lower abdominal cramping and spotting at 12 weeks' gestation. The primary care provider performs a pelvic examination and finds that the cervix is closed. What does the care provider suspect is the cause of the cramps and spotting? a. Habitual abortion b. Ectopic pregnancy c. Cervical insufficiency d. Threatened abortion

d. Threatened abortion Spontaneous abortion occurs along a continuum: threatened, inevitable, incomplete, complete, missed. The definition of each category is related to whether or not the uterus is emptied, or for how long the products of conception are retained.

Hypertonic labor is labor that is characterized by short, irregular contractions without complete relaxation of the uterine wall in between contractions. Hypertonic labor can be caused by an increased sensitivity to oxytocin. What would the nurse do for a client who is in hypertonic labor because of oxytocin augmentation? a. Increase the methotrexate. b. Increase the oxytocin. c. Turn off the methotrexate. d. Turn off the oxytocin.

d. Turn off the oxytocin. A labor complicated by occiput posterior position is usually prolonged and characterized by maternal perception of increased intensity of back discomfort. The lay term for this type of labor is "back labor."

The nurse is caring for a client who prefers resting on her back during the labor process. To facilitate client wishes, which nursing action is required? a. Elevate the knee gatch b. Place the tocometer low on abdomen c. Raise the head of the bed d. Utilize a wedge under one hip

d. Utilize a wedge under one hip Changing positions frequently can help during the labor process. By placing a wedge under the client's hip, it decreases the likelihood of hypotension and allows the nurse to protect the fetus from decreased oxygenation and meet client wishes. This option is the only one in which the staff is meeting client wishes. The head of the bed may be elevated as needed. Depending upon the location of the fetus, the tocometer is placed where fetal heart tones are able to be heard. Rarely is the knee gatch elevated as it may slow blood flow.

At what time is the laboring client encouraged to push? a. When the fetal head can be seen b. When she feels the urge to push c. When the health care provider has arrived d. When the cervix is fully dilated

d. When the cervix is fully dilated To avoid birth trauma, the client is not encouraged to push until the cervix is fully dilated. This is determined on vaginal exam. Once it is noted, there is no need to wait until the fetal head can be seen. The urge to push may be present without full cervix dilation. Labor is not stopped until the health care provider arrives. A nurse can deliver the fetus.

A pregnant client has been admitted with reports of brownish vaginal bleeding. On examination there is an elevated hCG level, absent fetal heart sounds, and a discrepancy between the uterine size and the gestational age. The nurse interprets these findings to suggest which condition? a. abruption of placenta b. placenta previa c. ectopic pregnancy d. gestational trophoblastic disease

d. gestational trophoblastic disease The client is most likely experiencing gestational trophoblastic disease or a molar pregnancy. In gestational trophoblastic disease or molar pregnancy, there is an abnormal proliferation and eventual degeneration of the trophoblastic villi. The signs and symptoms of molar pregnancy include brownish vaginal bleeding, elevated hCG levels, discrepancy between the uterine size and the gestational age, and absent fetal heart sounds. Abruption of placenta is characterized by premature separation of the placenta. Ectopic pregnancy is a condition where there is implantation of the blastocyst outside the uterus. In placenta previa the placental attachment is at the lower uterine segment.

The nurse is transcribing messages from the answering service. Which phone message should the nurse return first? a. an 18-year-old, 38-week G2P1 client with intermittent cramping; the client's last blood pressure was 98/50 mm Hg, and proteinuria was 1+ b. a 20-year-old, 31-week G1P0 client with malaise and rhinitis; the client's last blood pressure was 120/80 mm Hg, and she had no proteinuria c. a 25-year-old, 31-week G1P0 client with blood pressure of 100/80 mm Hg and left flank pain; the client's last blood pressure was 100/77 mm Hg and she had no proteinuria d. a 35-year-old, 21-week G3P2 client with blood pressure of 160/110 mm Hg, blurred vision, and whose last blood pressure was 143/99 mm Hg and urine dipstick showed a +2 proteinuria

d. a 35-year-old, 21-week G3P2 client with blood pressure of 160/110 mm Hg, blurred vision, and whose last blood pressure was 143/99 mm Hg and urine dipstick showed a +2 proteinuria The nurse should call the at-risk 35-year-old client first. She is 21 weeks and has symptoms (blurred vision) of preeclampsia. She also had an increase of protein in her urine (2+) and a 15% increase in her BP. The nurse will need more information to determine if the 38-week client may be in the early stages of labor, and if the 31-week client with flank pain has a kidney infection. The client with malaise and rhinitis will need to talk to the nurse last to find out what over-the-counter medication she is able to take.

A nurse is caring for a client who just experienced a miscarriage in her first trimester. When asked by the client why this happened, which is the best response from the nurse? rejection of the embryo through an immune response a. implantation abnormality b. lack of sufficient progesterone produced by c. the corpus luteum d. abnormal fetal development

d. abnormal fetal development The most frequent cause of miscarriage in the first trimester of pregnancy is abnormal fetal development, due either to a teratogenic factor or to a chromosomal aberration. In other miscarriages, immunologic factors may be present or rejection of the embryo through an immune response may occur. Another common cause of early miscarriage involves implantation abnormalities. Miscarriage may also occur if the corpus luteum on the ovary fails to produce enough progesterone to maintain the decidua basalis.

A nurse assesses a client in labor and suspects hypotonic uterine dysfunction. Which intervention would the nurse expect to include in the plan of care for this client? a. providing a comfortable environment with dim lighting b. preparing the woman for an amniotomy c. encouraging the woman to assume a hands-and-knees position d. administering oxytocin

d. administering oxytocin Oxytocin would be appropriate for the woman experiencing hypotonic uterine dysfunction (problem with the powers). Comfort measures minimize the woman's stress and promote relaxation so that she can work more effectively with the forces of labor. An amniotomy may be used with hypertonic uterine dysfunction to augment labor. A hands-and-knees position helps to promote fetal head rotation with a persistent occiput posterior position.

The nurse caring for a client in preterm labor observes abnormal fetal heart rate (FHR) patterns. Which nursing intervention should the nurse perform next? a. fetal scalp stimulation b. application of vibroacoustic stimulation c. tactile stimulation d. administration of oxygen by mask

d. administration of oxygen by mask The client should be administered oxygen by mask because the abnormal FHR pattern could be due to inadequate oxygen reserves in the fetus. Because the client is in preterm labor, it is not advisable to apply vibroacoustic stimulation, tactile stimulation, or fetal scalp stimulation.

After teaching a review class to a group of perinatal nurses about various methods for cervical ripening, the nurse determines that the teaching was successful when the group identifies which method as surgical? a. breast stimulation b. prostaglandin c. laminaria d. amniotomy

d. amniotomy Amniotomy is considered a surgical method of cervical ripening. Breast stimulation is considered a nonpharmacologic method for ripening the cervix. Laminaria is a hygroscopic dilator that mechanically causes cervical ripening. Prostaglandins are pharmacologic methods for cervical ripening.

A woman whose fetus in in the occiput posterior position is experiencing increased back pain. Which is the best way for the nurse to help alleviate this back pain? a. applying a heating pad to the back b. performing acupuncture on the back c. applying ice to the back d. applying counter pressure to the back

d. applying counter pressure to the back Counter pressure applied to the lower back with a fisted hand sometimes helps the woman cope with "back labor" associated with occiput-posterior positioning. The others are not recommended or used techniques for a woman in labor with back pain.

A nurse is caring for a pregnant client with asthma. Which intervention would the nurse perform first? a. assessing for feeling nauseated b. monitoring temperature frequently c. monitoring frequency of headache d. assessing oxygen saturation

d. assessing oxygen saturation The nurse should evaluate for signs of a respiratory complication with asthma clients. The nurse need not monitor the client's temperature, frequency of headache, or feelings of nausea because these conditions are not related to asthma.

A woman is admitted to the labor suite with contractions every 5 minutes lasting 1 minute. She is post-term and has oligohydramnios. What does this increase the risk of during birth? a. fetal hydrocephalus b. macrosomia c. shoulder dystocia d. cord compression

d. cord compression Oligohydramnios and meconium staining of the amniotic fluid are common complications of post-term pregnancy. Oligohydramnios increases the incidence of cord compression, which can lead to fetal distress during labor.

As a woman enters the second stage of labor, which would the nurse expect to assess? a. expressions of satisfaction with her labor progress b. reports of feeling hungry and unsatisfied c. falling asleep from exhaustion d. feelings of being frightened by the change in contractions

d. feelings of being frightened by the change in contractions The nature of contractions changes so drastically— the urge to push is very strong—that this can be frightening.

If a fetus were not receiving enough oxygen during labor because of uteroplacental insufficiency, which pattern would the nurse anticipate seeing on the monitor? a. fetal baseline rate increasing at least 5 mm Hg with contractions b. variable decelerations, too unpredictable to count c. a shallow deceleration occurring with the beginning of contractions d. fetal heart rate declining late with contractions and remaining depressed

d. fetal heart rate declining late with contractions and remaining depressed Lack of blood supply to the fetus because of poor placental filling prevents the fetal heart rate from recovering immediately following a contraction.

An OB/GYN care provider has just finished evaluating the 100th client. If the nurse could review all the documentation from each client thus far, which types of pelvis would the nurse predict the care provider has seen the most and the least? a. anthropoid and gynecoid, respectively b. gynecoid and android, respectively c. android and platypelloid, respectively d. gynecoid and platypelloid, respectively

d. gynecoid and platypelloid, respectively The gynecoid is the typical female pelvis shape, although only about half of all females have this type of pelvis. The platypelloid pelvis is the least common type of pelvis in women. Women with anthropoid pelvic shapes are able to give birth vaginally approximately one third of the time and are somewhat rare. An android pelvis is similar to a male pelvis and is seen in 16% of nonwhite women and one-third of white women.

A nurse is caring for a pregnant adolescent client, who is in her first trimester, during a visit to the maternal child clinic. Which important area should the nurse address during assessment of the client? a. options for birth control in the future b. whether sex was consensual c. sexual development of the client d. knowledge of child development

d. knowledge of child development The nurse should address the client's knowledge of child development during assessment of the pregnant adolescent client. The nurse need not address the sexual development of the client or whether sex was consensual. This would not be an opportune time to discuss birth control methods to be used after the pregnancy.

Early in labor, a pregnant client asks why contractions hurt so much. Which answer should the nurse provide? a. blocking of nerve transmission via mechanical irritation of nerve fibers b. release of endorphins in response to contractions c. distraction of the brain cortex by other stimuli d. lack of oxygen to the muscle fibers of the uterus due to compression of blood vessels

d. lack of oxygen to the muscle fibers of the uterus due to compression of blood vessels During contractions, blood vessels constrict, reducing the blood supply to uterine and cervical cells, resulting in anoxia to muscle fibers. This anoxia can cause pain in the same way blockage of the cardiac arteries causes the pain of a heart attack. Endorphins are naturally occurring opiate-like substances that reduce pain, not cause it. Distraction and mechanical irritation of nerve fibers are also methods of reducing pain, not causes of pain.

A primary care provider prescribes oral tocolytic therapy for a woman with preterm labor. The nurse explains to the client about the drugs that may be used. The nurse determines that the client needs additional teaching when she states which drug might be used? a. bethamethasone b. nifedipine c. indomethacin d. magnesium sulfate

d. magnesium sulfate Magnesium sulfate is only given intravenously for preterm labor. Nifedipine and indomethacin are given orally for preterm labor. Betamethasone promotes fetal lung maturity by stimulating surfactant production.

A woman with an artificial mitral valve develops heart failure at the 20th week of pregnancy. Which measure would the nurse stress with her during the remainder of the pregnancy? a. beginning a low-impact aerobics program b. discontinuing her prepregnancy anticoagulant c. maintaining a high fluid intake d. obtaining enough rest

d. obtaining enough rest As the blood volume doubles during pregnancy, heart failure can occur. The pregnant woman needs to obtain adequate rest to prevent overworking the heart. Fluid may need to be restricted.

A pregnant client has developed iron-deficiency anemia and has been prescribed 200 mg of elemental iron per day. The nurse should encourage the client to take this medication with which substance? a. a full meal b. water c. milk d. orange juice

d. orange juice Iron is absorbed best from an acid medium. Advise women, therefore, to take iron supplements with orange juice or a vitamin C supplement, which supplies ascorbic acid.

A client in labor has been admitted to the labor and birth suite. The nurse assessing her notes that the fetus is in a cephalic presentation. Which description should the nurse identify by the term presentation? a. relation of the fetal presenting part to the maternal ischial spine b. relation of the different fetal body parts to one another c. relationship of the presenting part to the maternal pelvis d. part of the fetal body entering the maternal pelvis first

d. part of the fetal body entering the maternal pelvis first The term presentation is the part of the fetal body that is entering the maternal pelvis first. Relationship of the presenting part to the sides of the maternal pelvis is called the position. Attitude is the term that describes the relation of the different fetal body parts to one another. Relation of the fetal presenting part to maternal ischial spine is termed the station.

A nurse is caring for a client undergoing treatment for ectopic pregnancy. Which symptom is observed in a client if rupture or hemorrhaging occurs before the ectopic pregnancy is successfully treated? a. fetal distress b. painless bright red vaginal bleeding c. tetanic contractions d. phrenic nerve irritation

d. phrenic nerve irritation The symptoms if rupture or hemorrhaging occurs before successfully treating the pregnancy are lower abdomen pain, feelings of faintness, phrenic nerve irritation, hypotension, marked abdominal tenderness with distension, and hypovolemic shock. Painless bright red vaginal bleeding occurring during the second or third trimester is the clinical manifestation of placenta previa. Fetal distress and tetanic contractions are not the symptoms observed in a client if rupture or hemorrhaging occurs before successfully treating an ectopic pregnancy.

The nurse is admitting a G3 P2 client at 38 weeks' gestation who arrived reporting painless bleeding from the vagina leading to the diagnosis of placenta previa. When questioned by the client as to what caused this, which most likely factor should the nurse point out in her answer? a. morbidly obese b. living in coastal areas c. maternal age more than 30 years d. previous cesarean birth

d. previous cesarean birth The risk of placenta previa is greatly increased when a woman has had a previous cesarean delivery due to the scarring of the endometrial lining. Maternal age over 35 years, and not just more than 30 years, is considered another risk factor. Placenta previa is more common among those living in high altitudes not among those living in coastal areas. Obesity is not recognized as a potential risk for this condition. Other risk factors can include uterine insult or injury, cocaine use, prior placenta previa, infertility treatment, multiple gestations, previous induced surgical abortion, smoking previous myomectomy to remove fibroids, short interval between pregnancies, and hypertension or diabetes.

A 19-year-old female presents in advanced labor. Examination reveals the fetus is in frank breech position. The nurse interprets this finding as indicating: a. one arm is presenting. b. the fetus is sitting cross-legged above the cervix. c. one leg is presenting. d. the buttocks are presenting first with both legs extended up toward the face.

d. the buttocks are presenting first with both legs extended up toward the face. In a frank breech position, the buttocks present first with both legs extended up toward the face. The full or complete breech occurs when the fetus sits crossed-legged above the cervix. In a footling or incomplete breech one or both legs are presenting.


Related study sets

Chapter 15:Oncology: Nursing Management in Cancer Care

View Set

Chapter 14 Final Exam Political Science

View Set

Core Concepts Practice Questions

View Set

Slumdog Millionaire Show Questions

View Set

Unit 5-8 EXCEL, Unit 9-11 EXCEL, Unit 1-4 EXCEL

View Set